You are on page 1of 99

JD3A INSURANCE

COMPILATION OF CASE
DIGESTS
CASE NO. 1. VICENTE G. HENSON, JR., PETITIONER, V. UCPB GENERAL INSURANCE
CO., INC., RESPONDENT., G.R. NO. 223134, AUGUST 14, 2019 – DE GUZMAN, YVONNE
CHRISTELLE

FACTS:
From 1989 to 1999, National Arts Studio and Color Lab6 (NASCL) leased the front
portion of the ground floor of a 2-storey building, then owned by petitioner.
In 1999, NASCL gave up its initial lease and instead, leased the right front portion of the
ground floor and the entire second floor of the said building, and made renovations with the
building's piping assembly. Meanwhile, Copylandia Office Systems Corp. (Copylandia) moved in
to the ground floor. On May 9, 2006, a water leak occurred in the building and damaged
Copylandia's various equipment, causing injury to it in the amount of P2,062,640.00. As the said
equipment were insured with UCPB General Insurance Co. (UCPB), Copylandia filed a
claim with the former. Eventually, the two parties settled on November 2, 2006 for the amount
of P1,326,342.76.
This resulted in UCPB's subrogation to the rights of Copylandia over all claims and
demands arising from the said incident.
On May 20, 2010, UCPB, as subrogee to Copylandia's rights, demanded from, inter
alia, NASCL for the payment of the aforesaid claim, but to no avail. Thus, it filed a complaint for
damages against NASCL, among others, before the RTC.

ISSUE: Whether by virtue of the payment of subrogee-insurer it is able to acquire, all rights of
the subrogor-insured against the debtor?

RULING:
Subrogation‘s legal effects under Article 2207 of the Civil Code are primarily between the
subrogee-insurer and the subrogor-insured: by virtue of the former‘s payment of indemnity to
the latter, it is able to acquire, by operation of law, all rights of the subrogor-insured against the
debtor. The debtor is a stranger to this juridical tie because it only remains bound by its original
obligation to its creditor whose rights, however, have already been assumed by the subrogee. In
Vector‘s case, American Home was able to acquire ipso jure all the rights Caltex had against
Vector under their contract of affreightment by virtue of its payment of indemnity. If at all,
subrogation had the effect of obliging Caltex to respect this assumption of rights in that it must
now recognize that its rights against the debtor, i.e., Vector, had already been transferred to
American Home as the subrogee-insurer. In other words, by operation of Article 2207 of the
Civil Code, Caltex cannot deny American Home of its right to claim against Vector. However,
the subrogation of American Home to Caltex‘s rights did not alter the original obligation between
Caltex and Vector.

CASE NO. 2. PAZ LOPEZ DE CONSTANTINO V. ASIA LIFE INSURANCE COMPANY, 87


PHIL 248, AUGUST 31, 1950 – CHAWI, DECIMAY

FACTS:
FIRST CASE:
In consideration of the sum of P176.04 as annual premium duly paid to it, the Asia Life
Insurance Company (a foreign corporation incorporated under the laws of Delaware, U. S. A.) ,
issued on September 27, 1941, its Policy No. 93912 for P3,000, whereby it insured the life of
Arcadio Constantino for a term of twenty years. The first premium covered the period up to

Page 1 of 99
September 26, 1942. The plaintiff Paz Lopez de Constantino was regularly appointed
beneficiary.
All premium payments are due in advance and any unpunctuality in making any such payment
shall cause this policy to lapse unless and except as kept in force by the Grace Period condition
or under Option 4 below. (Grace of 31 days.)"
After that first payment, no further premiums were paid. The insured died on September 22,
1944.

It is admitted that the defendant, being an American corporation, had to close its branch office in
Manila by reason of the Japanese occupation, i. e. from January 2 1942, until the year 1945.
SECOND CASE:
On August 1, 1938, the defendant Asia Life Insurance Company issued its Policy No. 78145
(Joint Life 20-Year Endowment Participating with Accident Indemnity), covering the lives of the
spouses Tomas Ruiz and Agustina Peralta, for the sum of P3,000, The annual premium
stipulated in the policy was regularly paid from August 1, 1938, up to and including September
30, 1941. Effective August 1, 1941 the mode of payment of premiums was changed from annual
to quarterly, so that quarterly premiums were paid, the last having been delivered on November
18, 1941, said payment covering the period up to January 31, 1942. No further payments were
handed to the insurer. Upon the Japanese occupation, the insured and the insurer became
separated by the lines of war, and it was impossible and illegal for them to deal with each other.
Because the insured had borrowed on the policy an amount of P234.00 in January, 1941, the
cash surrender value of the policy was sufficient to maintain the policy in force only up to
September 7, 1942. Tomas Ruiz died on February 16, 1945. The plaintiff Agustina Peralta is his
beneficiary. Her demand for payment met with defendant‘s refusal, grounded on non-payment
of the premiums.
Plaintiffs maintain that, as beneficiaries, they are entitled to receive the proceeds of the policies
minus all sums due for premiums in arrears. They allege that non-payment of the premiums was
caused by the closing of defendant‘s offices in Manila during the Japanese occupation and the
impossible circumstances created by war.
Defendant on the other hand asserts that the policies had lapsed for non-payment of premiums,
in accordance with the contract of the parties and the law applicable to the situation.
ISSUE: Whether the beneficiary in a life insurance policy may recover the amount thereof
although the insured died after repeatedly failing to pay the stipulated premiums, such failure
having been caused by the last war in the Pacific
HELD: (NO)
When the life insurance policy provides that non-payment of premiums will cause its
forfeiture, war does not excuse non-payment, and does not avoid forfeiture.
Rejecting the Connecticut Rule, and the New York Rule, the court adopts the United
States Rule about the effects of war upon non-payment of premiums.
CONNECTICUT RULE:

Page 2 of 99
"There are two elements in the consideration for which the annual premium is paid —
First, the mere protection for the year, and, second, the privilege of renewing the contract for
each succeeding year by paying the premium for that year at the time agreed upon. According
to this view of the contract, the payment of premiums is a condition precedent, the non-
performance of which, even when performance would be illegal, necessarily defeats the right to
renew the contract.‖
NEW YORK RULE:
"War between states in which the parties reside merely suspends the contracts of life
insurance, and that, upon tender of all premiums due by the insured or his representative after
the war has terminated, the contract revives and becomes fully operative."
UNITED STATES RULE:
The contract is not merely suspended, but is abrogated by reason of nonpayment of
premiums, since the time of the payments is peculiarly of the essence of the contract. It
additionally holds that it would be unjust to allow the insurer to retain the reserve value of the
policy, which is the excess of the premiums paid over the actual risk carried during the years
when the policy had been in force. This rule was announced in the well-known Statham 6 case
which, in the opinion of Professor Vance, is the correct rule.
Non-payment at the day involves absolute forfeiture if such be the terms of the contract,
as is the case here. Courts cannot with safety vary the stipulation of the parties by introducing
equities for the relief of the insured against their own negligence.
After perusing the Insurance Act, the non-payment of premiums is such a vital defense
of insurance companies that since the very beginning, said Act No. 2427 expressly preserved it,
by providing that after the policy shall have been in force for two years, it shall become
incontestable (i. e. the insurer shall have no defense) except for fraud, non-payment of
premiums, and military or naval service in time of war (sec. 184 [b], Insurance Act). And when
Congress recently amended this section (Rep. Act No. 171), the defense of fraud was
eliminated, while the defense of nonpayment of premiums was preserved. Thus the
fundamental character of the undertaking to pay premiums and the high importance of the
defense of non-payment thereof, was specifically recognized.

The periodic payment of premiums in life insurances policies is not an obligation of the
insured enforceable by action.
CASE NO. 3. RAFAEL ENRIQUEZ, AS ADMINISTRATOR OF THE ESTATE OF THE LATE
JOAQUIN MA. HERRER V. SUN LIFE ASSURANCE COMPANY OF CANADA, G.R. NO. L-
15895, NOVEMBER 29, 1920 – CAWIS, AUDREY

FACTS:

On September 24, 1917, Joaquin Herrer made an application to the Sun Life Assurance
Company of Canada through its office in Manila for a life annuity. 2 days later, he paid P6,000 to
the manager of the company's Manila office and was given a receipt. According to the
provisional receipt, 3 things had to be accomplished by the insurance company before there
was a contract:

Page 3 of 99
1. There had to be a medical examination of the applicant;

2. there had to be approval of the application by the head office of the company; and

3. this approval had in some way to be communicated by the company to the applicant

On November 26, 1917, the head office at Montreal, Canada gave notice of acceptance
by cable to Manila but this was not mailed. On December 4, 1917 the policy was issued at
Montreal. On December 18, 1917, Atty. Aurelio A. Torres wrote to the Manila office of the
company stating that Herrer desired to withdraw his application.

However, on December 19, 1917 the local office replied to Atty. Torres, stating that the
policy had been issued, and called attention to the notification of November 26, 1917

On the morning of December 21, 1917 Mr. Torres received the policy, but on December
20, 1917, Mr. Herrer died. Rafael Enriquez, as administrator of the estate of the late Joaquin
Ma. Herrer filed to recover from Sun Life Assurance Company of Canada through its office in
Manila for a life annuity.

Issue:

Whether Mr. Herrera received notice of acceptance of his application thereby perfecting
his life annuity.

Ruling:

NO. The Enriquez‘ shall have and recover from the Sun Life the sum of P6,000 with
legal interest from November 20, 1918, until paid, without special finding as to costs in either
instance.

According Art. 1319 of the Civil Code:

Consent is manifested by the meeting of the offer and the acceptance upon the thing
and the cause which are to constitute the contract. The offer must be certain and the
acceptance absolute. A qualified acceptance constitutes a counter-offer.

Acceptance made by letter or telegram does not bind the offerer except from the time it
came to his knowledge. The contract, in such a case, is presumed to have been entered into in
the place where the offer was made.

Therefore, it was not perfected because it has not been proved satisfactorily that the
acceptance of the application to withdraw ever came to the knowledge of the applicant

CASE NO. 4. RIZAL SURETY AND INSURANCE CORP V. CA, 336 SCRA 12 – PUA,
JOMARIE CASSANDRA

FACTS:

Rizal Surety issued a 1 million peso fire insurance policy with Transworld. This was
increased to 1.5 million. A four span building was part of the policy. A fire broke out and gutted
the building together with a two storey building behind it where gaming machines were stored.
The company filed its claims but to no avail. Hence, it brought a suit in court. It aimed to make
Rizal pay for almost 3 million including legal interest and damages. Rizal claimed that the policy
only covered damage on the four span building and not the two storey building.

Page 4 of 99
The trial court ruled in Transworld‘s favor and ordered Rizal to pay actual damages only.
The court of appeals increased the damages. The insurance company filed a Motion for
Reconsideration. The CA answered by modifying the imposition of interest. Not satisfied, the
insurance company petitioned to the Supreme Court.

ISSUE:

Whether Rizal Surety is liable for loss of the two-storey building considering that the fire
insurance policy sued upon covered only the contents of the four-span building.

HELD:

Yes. Petition is dismissed.

The policy had clauses on the building coverage that read:

"contained and/or stored during the currency of this Policy in the premises occupied by
them forming part of the buildings situated within own Compound"

"First, said properties must be contained and/or stored in the areas occupied by
Transworld and second, said areas must form part of the building described in the policy xxx"

This means that the policy didn‘t limit its coverage to what was stored in the four-span
building.

As to questions of fact, both the trial court and the Court of Appeals found that the so-
called "annex " was not an annex building but an integral part of the four-span building
described in the policy and consequently, the machines and spare parts stored were covered by
the fire insurance.

A report said: "Two-storey building constructed of partly timber and partly concrete
hollow blocks under g.i. roof which is adjoining and intercommunicating with the repair of the
first right span of the lofty storey building and thence by property fence wall."

Article 1377 provides that the interpretation of obscure words or stipulations in a contract
shall not favor the party who caused the obscurity.

In the case of Landicho vs. GSIS, the terms in an insurance policy, which are
ambiguous, equivocal, or uncertain are to be construed strictly and most strongly against the
insurer, and liberally in favor of the insured so as to effect the dominant purpose of indemnity or
payment to the insured.

CASE NO. 5. MILAGROS P. ENRIQUEZ VS. THE MERCANTILE INSURANCE CO., INC.,
G.R. 210950, AUGUST 15, 2018 – GATTOC, SHEKINAH

FACTS:

Petitioner Enriquez filed a replevin case against Asuten for the recovery of the Toyota
Hi-Ace van valued at P300,000.00. She applied for a bond in the amount of P600,000.00 with
respondent in Asuten's favor. The Regional Trial Court approved the bond and ordered the
sheriff to recover the van from Asuten and to deliver it to petitioner. While the van was in
petitioner's custody, the Regional Trial Court dismissed the case without prejudice for failure to
prosecute. Thus, it ordered the sheriff to restore the van to Asuten. When petitioner failed to

Page 5 of 99
produce the van, the Regional Trial Court directed respondent to pay Asuten the amount of the
bond.

There was no trial on the merits. The Regional Trial Court's dismissal for failure to
prosecute was a dismissal without prejudice to re-filing. In this particular instance, any writ of
seizure, being merely ancillary to the main action, becomes functus oficio. The parties returned
to the status quo as if no case for replevin had been filed. Thus, upon the dismissal of the case,
it was imperative for petitioner to return the van to Asuten.

ISSUES:

Whether petitioner Enriquez should be made liable for the full amount of the bond paid
by respondent as surety, in relation to a previous case for replevin filed by petitioner.

RULINGS:

YES. There was no trial on the merits. Basic is the principle that "a contract is law
between the parties" for as long as it is "not contrary to law, morals, good customs, public order,
or public policy." Under their Indemnity Agreement, petitioner held herself liable for any payment
made by respondent by virtue of the replevin bond.

Petitioner contends that the Indemnity Agreement was a contract of adhesion since
respondent made the extent of liability "so comprehensive and all-encompassing to the point of
being ambiguous." A contract of insurance is, by default, a contract of adhesion. It is prepared
by the insurance company and might contain terms and conditions too vague for a layperson to
understand; hence, they are construed liberally in favor of the insured. In Verendia v. Court of
Appeals:

Basically a contract of indemnity, an insurance contract is the law between the parties.
Its terms and conditions constitute the measure of the insurer's liability and compliance
therewith is a condition precedent to the insured's right to recovery from the insurer. As it is also
a contract of adhesion, an insurance contract should be liberally construed in favor of the
insured and strictly against the insurer company which usually prepares it.

It is clear from the antecedents that any losses which petitioner has suffered were due to
the consequences of her actions, or more accurately, her inactions. Civil Case No. 10846, which
she filed, was dismissed due to her failure to prosecute. The Regional Trial Court forfeited the
replevin bond which she had filed because she refused to return the property. She is now made
liable for the replevin bond because she failed to appeal its forfeiture.

CASE NO. 6. ALPHA INSURANCE AND SURETY CO. V. ARSENIA SONIA CASTOR, G.R.
NO. 198174, SEPTEMBER 2, 2013 – ALPINE CHIWERAN

FACTS:

On February 21, 2007, respondent Arsenia entered into a contract of insurance with petitioner
Alpha Insurance, involving her motor vehicle, a Toyota Revo DLX DSL. The contract of
insurance obligates the petitioner to pay the respondent the amount of Six Hundred Thirty
Thousand Pesos (P630,000.00) in case of loss or damage to said vehicle during the period
covered, which is from February 26, 2007 to February 26, 2008. On April 16, 2007, at about

Page 6 of 99
9:00 a.m., respondent instructed her driver, Jose Joel Salazar Lanuza (Lanuza), to bring the
above-described vehicle to a nearby auto-shop for a tune-up. However, Lanuza no longer
returned the motor vehicle to respondent and despite diligent efforts to locate the same, said
efforts proved futile.

Resultantly, respondent promptly reported the incident to the police and concomitantly notified
petitioner of the said loss and demanded payment of the insurance proceeds in the total sum of
P630,000.00. However, petitioner denied the insurance claim of respondent, stating among
others, that since Lanuza who stole the car is employed by Arsenia the loss is not covered per
provision of the Policy under Exceptions to Section-III.

ISSUE:

Whether the loss of respondent‘s vehicle is excluded under the insurance policy.

HELD:

No, the loss of respondent‘s vehicle is included under the insurance policy.

It is a basic rule in the interpretation of contracts that the terms of a contract are to be construed
according to the sense and meaning of the terms which the parties thereto have used. In the
case of property insurance policies, the evident intention of the contracting parties, i.e., the
insurer and the assured, determine the import of the various terms and provisions embodied in
the policy. However, when the terms of the insurance policy are ambiguous, equivocal or
uncertain, such that the parties themselves disagree about the meaning of particular provisions,
the policy will be construed by the courts liberally in favor of the assured and strictly against the
insurer.

Adverse to petitioner‘s claim, the words "loss" and "damage" mean different things in common
ordinary usage. The word "loss" refers to the act or fact of losing, or failure to keep possession,
while the word "damage" means deterioration or injury to property.

Therefore, petitioner cannot exclude the loss of respondent‘s vehicle under the insurance policy
under paragraph 4 of ―Exceptions to Section III,‖ since the same refers only to ―malicious
damage,‖ or more specifically, ―injury‖ to the motor vehicle caused by a person under the
insured‘s service. Paragraph 4 clearly does not contemplate ―loss of property,‖ as what
happened in the instant case.

Also, a contract of insurance is a contract of adhesion. So, when the terms of the insurance
contract contain limitations on liability, courts should construe them in such a way as to preclude
the insurer from non-compliance with his obligation.

CASE NO. 7. FORTUNE MEDICARE V. DAVID ROBERT U. AMORIN, G.R. NO. 195872,
MARCH 12, 2014 – ABRIGO, KAREN

FACTS:

David Robert U. Amorin was a cardholder/member of Fortune Medicare, Inc. (Fortune


Care), a corporation engaged in providing health maintenance services to its members. The
terms of Amorin's medical coverage were provided in a Corporate Health Program Contract
(Health Care Contract) which was executed on January 6, 2000 by Fortune Care and the House
of Representatives, where Amorin was a permanent employee. While on vacation in Honolulu,
Hawaii, USA, Amorin underwent an emergency surgery, specifically appendectomy, at the St.

Page 7 of 99
Francis Medical Center, causing him to incur professional and hospitalization expenses of
US$7,242.35 and US$1,777.79, respectively. He attempted to recover from Fortune Care the
full amount thereof upon his return to Manila, but the company merely approved a
reimbursement of ₱12,151.36, an amount that was based on the average cost of
appendectomy, net of medicare deduction, if the procedure were performed in an accredited
hospital in Metro Manila. Amorin received under protest the approved amount, but asked for its
adjustment to cover the total amount of professional fees which he had paid, and eighty percent
(80%) of the approved standard charges based on "American standard", considering that the
emergency procedure occurred in the U.S.A.

To support his claim, Amorin cited Section 3 (B), Article V on Benefits and Coverages of
the Health Care Contract for EMERGENCY CARE IN NON-ACCREDITED HOSPITAL which
says: “Whether as an in-patient or out-patient, Fortune Care shall reimburse the total
hospitalization cost including the professional fee (based on the total approved charges)
to a member who receives emergency care in a non-accredited hospital. The above
coverage applies only to Emergency confinement within Philippine Territory. However, if
the emergency confinement occurs in a foreign territory, Fortune Care will be obligated
to reimburse or pay eighty (80%) percent of the approved standard charges which shall
cover the hospitalization costs and professional fees.”

Fortune Care denied Amorin‘s request arguing that the Health Care Contract did not
cover hospitalization costs and professional fees incurred in foreign countries, as the contract‘s
operation was confined to Philippine territory. Further, it argued that its liability to Amorin was
extinguished upon the latter‘s acceptance from the company of the amount of ₱12,151.36. This
prompted Amorin to file a complaint for breach of contract with damages with the RTC of Makati
City.

RTC Ruling: The RTC of Makati dismissed Amorin‘s complaint citing Section 3, Article V
of the Health Care Contract, which the RTC explained: ―Taking the contract as a whole, the
Court is convinced that the parties intended to use the Philippine standard as basis as provided
in Section 3 of the Corporate Health Care Program Contract.‖

CA Ruling: Appeal Granted- Health Care Contract, being like insurance contracts, must
be liberally construed in favor of the subscriber. In case its provisions are doubtful or reasonably
susceptible of two interpretations, the construction conferring coverage is to be adopted and
exclusionary clauses of doubtful import should be strictly construed against the provider.
Second, the CA explained that there was nothing under Article V of the Health Care Contract
which provided that the Philippine standard should be used even in the event of an emergency
confinement in a foreign territory. Fortune Care‘s motion for reconsideration was denied, hence,
the filing of the present petition for review on certiorari.

ISSUE:

Whether Fortune Medicare is liable to reimburse or pay Amorin of the 80% of the
approved standard charges which shall cover the hospitalization costs and professional fees he
incurred in the USA.

SC RULING:

YES. Fortune Medicare is liable to pay Amorin 80% of the approved standard charges
which shall cover the hospitalization costs and professional fees he incurred in the USA. The
SC emphasized that for purposes of determining the liability of a health care provider to its

Page 8 of 99
members, jurisprudence holds that a health care agreement is in the nature of non-life
insurance, which is primarily a contract of indemnity. Once the member incurs hospital, medical
or any other expense arising from sickness, injury or other stipulated contingent, the health care
provider must pay for the same to the extent agreed upon under the contract.

To aid in the interpretation of health care agreements, the Court laid down the following
guidelines in Philamcare Health Systems v. CA: When the terms of insurance contract
contain limitations on liability, courts should construe them in such a way as to preclude
the insurer from non-compliance with his obligation. Being a contract of adhesion, the
terms of an insurance contract are to be construed strictly against the party which
prepared the contract – the insurer. The Court agrees with the CA that as may be gleaned
from the Health Care Contract, the parties thereto contemplated the possibility of emergency
care in a foreign country. As the contract recognized Fortune Care‘s liability for emergency
treatments even in foreign territories, it expressly limited its liability only insofar as the
percentage of hospitalization and professional fees that must be paid or reimbursed was
concerned, pegged at a mere 80% of the approved standard charges. The word "standard" as
used in the cited stipulation was vague and ambiguous, as it could be susceptible to different
meanings. The proper interpretation of the phrase "standard charges" could instead be
correlated with and reasonably inferred from the other provisions of Section 3(B). Thus, for
emergency care in non-accredited hospitals, this cited clause declared the standard in the
determination of the amount to be paid, without any reference to and regardless of the amounts
that would have been payable if the treatment was done by an affiliated physician or in an
affiliated hospital. For treatments in foreign territories, the only qualification was only as to the
percentage, or 80% of that payable for treatments performed in non-accredited hospital. Settled
is the rule that ambiguities in a contract are interpreted against the party that caused the
ambiguity. "Any ambiguity in a contract whose terms are susceptible to different interpretations
must be read against the party who drafted it.

CASE NO. 8. REPUBLIC OF THE PHILIPPINES, REPRESENTED BY THE COMMISSIONER


OF INTERNAL REVENUE V. SUNLIFE ASSURANCE COMPANY OF CANADA, G.R. NO.
158085, OCTOBER 14, 2005 – SOMERA, MARIA MONICA

―Mutual life insurance companies that are bona fide cooperatives are entitled to
exemption from the payment of taxes on life insurance premiums and documentary stamps.‖

FACTS:

Sun Life is a mutual life insurance company organized under the laws of Canada. It is
registered and authorized by the SEC and the Insurance Commission to engage in the business
in the Philippines as a mutual life insurance company. It made payments to the Commissioner
of Internal Revenue (CIR) P31.49M and P30M as premium tax and DST, respectively.

However, Sunlife subsequently filed an administrative claim for tax credits with the CIR
on ground of alleged erroneous payment. This is on the strength of the ruling of the SC in the
case of CIR v. Insular Life holding that mutual life insurance companies are purely cooperative
companies and are exempt from the payment of premium tax and DST.

The CIR, among others, raised the following defenses: 1) failure of Sunlife to register
with the Cooperative Development Authority (CDA), 2) failure to prove that its ownership was
vested to its member; 3) claimed that mutual life insurance companies earn profits on account of
it being subjected to regular corporate income tax under the 1997 Tax Code.

Page 9 of 99
The CIR failed to act on the claim. The CTA ruled in favor of Sunlife. The CA upheld
CTA‘s ruling.

ISSUES:
1) Whether Sunlife is a Cooperative.
2) Whether registration with the Cooperative Development Authority is a sine qua non
requirement to be entitled to tax exemption.
3) Whether respondent is exempted from payment of tax on life insurance premiums and
documentary stamp tax.

RULING:

1) Yes. The Tax Code defines a Cooperative as an association ―conducted by the


members thereof with the money collected from among themselves and solely for their own
protection and not for profit. The SC held that Sulife squarely falls within this definition.

First, it is managed by its members. Sunlife has been mutualized or converted from a
stock life insurance company to a non-stock mutual life insurance corporation, in conformance
with existing laws. Under its bylaws, ownership has been vested in its member-policyholders
who are entitled to vote. They also elect the board of trustees, the governing body of a non-
stock corporation.

Second, it is operated with money collected from its members. Sunlife is composed
entirely of members, all premiums obviously came from them. The member-policyholders were
both the ―insurers‖ and the ―insured‖ who contribute via premiums to create a fund for the
payment of the said members‘ indemnity and benefit claims.

Third, it is licensed for the mutual protection of its members, not for the profit of anyone.
A mutual life insurance company is conducted for the benefit of its member-policyholders, who
pay into its capital by way of premiums. To that extent, they are responsible for the payment of
all its losses. ―The cash paid in for premiums and the premium notes constitute their assets x x
x.‖ In the event that the company itself fails before the terms of the policies expire, the member-
policyholders do not acquire the status of creditors. Rather, they simply become debtors for
whatever premiums that they have originally agreed to pay the company, if they have not yet
paid those amounts in full, for ―[m]utual companies x x x depend solely upon x x x premiums.‖
Only when the premiums will have accumulated to a sum larger than that required to pay for
company losses will the member-policyholders be entitled to a ―pro rata division thereof as
profits.‖

2) No. First, the Tax Code does not require registration with the CDA. Second, the
provisions of the Cooperative Code of the Philippines do not apply. Third, not even the
Insurance Code requires registration with the CDA. The statutes prevailing at the time of
Sunlife‘s organization and mutualization were the Insurance Code and the Corporation Code,
which imposed no registration requirement with the CDA.

3) Yes. Having determined that Sunlilfe is a cooperative that does not have to be
registered with the CDA, it is entitled to exemption from both premium taxes and documentary
stamp taxes (DST).

Page 10 of 99
The Tax Code is clear. On the one hand, Section 121 of the Code exempts cooperative
companies from the 5 percent percentage tax on insurance premiums. On the other hand,
Section 199 also exempts from the DST, policies of insurance or annuities made or granted by
cooperative companies. Being a cooperative, respondent is thus exempt from
both types of taxes.

CASE NO. 9. WHITE GOLD MARINE SERVICES V. PIONEER INSURANCE, G.R. NO.
154514, JULY 28, 2005 – RAGMA, MYKA MARIE KHRISTINE

FACTS:

White Gold Marine Services, Inc. (White Gold) procured a protection and indemnity
coverage for its vessels from The Steamship Mutual Underwriting Association (Bermuda)
Limited (Steamship Mutual) through Pioneer Insurance and Surety Corporation (Pioneer).
Subsequently, White Gold was issued a Certificate of Entry and Acceptance. Pioneer also
issued receipts evidencing payments for the coverage. When White Gold failed to fully pay its
accounts, Steamship Mutual refused to renew the coverage.

Steamship Mutual thereafter filed a case against White Gold for collection of sum of
money to recover the latter‘s unpaid balance. White Gold on the other hand, filed a complaint
before the Insurance Commission claiming that Steamship Mutual violated Sections 186 and
187 of the Insurance Code, while Pioneer violated Sections 299, 300 and 301 in relation to
Sections 302 and 303, thereof.

The Insurance Commission dismissed the complaint. It said that there was no need for
Steamship Mutual to secure a license because it was not engaged in the insurance business.

Steamship Mutual contends that although Steamship Mutual is a P & I Club, it is not
engaged in the insurance business in the Philippines. It is merely an association of vessel
owners who have come together to provide mutual protection against liabilities incidental to
shipowning.

ISSUE:

Whether Steamship Mutual, a P&I Club, is engaged in the insurance business in the
Philippines.

HELD:

Yes. A P & I Club is ―a form of insurance against third party liability, where the third party
is anyone other than the P & I Club and the members.‖ By definition then, Steamship Mutual as
a P & I Club is a mutual insurance association engaged in the marine insurance business.

The test to determine if a contract is an insurance contract or not, depends on the nature
of the promise, the act required to be performed, and the exact nature of the agreement in the
light of the occurrence, contingency, or circumstances under which the performance becomes
requisite. It is not by what it is called.

Basically, an insurance contract is a contract of indemnity. In it, one undertakes for a


consideration to indemnify another against loss, damage or liability arising from an unknown or
contingent event.

Page 11 of 99
In particular, a marine insurance undertakes to indemnify the insured against marine
losses, such as the losses incident to a marine adventure. Section 99 of the Insurance Code
enumerates the coverage of marine insurance.

Relatedly, a mutual insurance company is a cooperative enterprise where the members


are both the insurer and insured. In it, the members all contribute, by a system of premiums or
assessments, to the creation of a fund from which all losses and liabilities are paid, and where
the profits are divided among themselves, in proportion to their interest. Additionally, mutual
insurance associations, or clubs, provide three types of coverage, namely, protection and
indemnity, war risks, and defense costs.

CASE NO. 10. PHILIPPINE HEALTH CARE PROVIDERS, INC. V. COMMISSIONER OF


INTERNAL REVENUE, G.R. NO. 167330, SEPTEMBER 18, 2009 – MENDOZA, JOYCE
ALLYSA

FACTS:

PhilHealth is a domestic corporation whose primary purpose is "[t]o establish, maintain,


conduct and operate a prepaid group practice health care delivery system or a health
maintenance organization to take care of the sick and disabled persons enrolled in the health
care plan and to provide for the administrative, legal, and financial responsibilities of the
organization." Individuals enrolled in its health care programs pay an annual membership fee
and are entitled to various preventive, diagnostic and curative medical services provided by its
duly licensed physicians, specialists and other professional technical staff participating in the
group practice health delivery system at a hospital or clinic owned, operated or accredited by it.

Respondent Commissioner of Internal Revenue [CIR] sent petitioner a formal demand


letter and the corresponding assessment notices demanding the payment of deficiency taxes,
including surcharges and interest, for the taxable years 1996 and 1997. The deficiency
[documentary stamp tax (DST)] assessment was imposed on petitioner‘s health care agreement
with the members of its health care program pursuant to Section 185 of the 1997 Tax Code.
Petitioner protested the assessment. As respondent did not act on the protest, petitioner
filed a petition for review in the Court of Tax Appeals (CTA) seeking the cancellation of the
deficiency VAT and DST assessments.
On April 5, 2002, the CTA rendered a decision partially granting the petitioners petition
for review. Respondent appealed the CTA decision to the CA insofar as it cancelled the DST
assessment. He claimed that petitioner‘s health care agreement was a contract of insurance
subject to DST under Section 185 of the 1997 Tax Code.
On August 16, 2004, the CA rendered its decision. It held that petitioner‘s health care
agreement was in the nature of a non-life insurance contract subject to DST. Petitioner moved
for reconsideration, but the CA denied it. Hence, petitioner filed this case.
ISSUE:
1. Whether or not petitioner as an HMO is engaged in an insurance business.
2. Whether or not petitioner is liable for the payment of DST on Health Care Agreement of
HMOS in accordance with Section 185.
HELD:

Page 12 of 99
1. No. In the application of Section 2(2) of PD 1460, the fact that no profit is derived from
the making of insurance contracts, agreements or transactions or that no separate or
direct consideration is received therefore, shall not be deemed conclusive to show that
the making thereof does not constitute the doing or transacting of an insurance
business.
Various courts in the United States, whose jurisprudence has a persuasive effect on our
decisions have determined that HMOs are not in the insurance business. One test that they
have applied is whether the assumption of risk and indemnification of loss (which are elements
of an insurance business) are the principal object and purpose of the organization or whether
they are merely incidental to its business. If these are the principal objectives, the business is
that of insurance. But if they are merely incidental and service is the principal purpose, then the
business is not insurance. The rule was enunciated in Jordan v. Group Health
Association wherein the Court of Appeals of the District of Columbia Circuit held that Group
Health Association should not be considered as engaged in insurance activities since it was
created primarily for the distribution of health care services rather than the assumption of
insurance risk.
Lastly, it is significant that petitioner, as an HMO, is not part of the insurance industry.
This is evident from the fact that it is not supervised by the Insurance Commission but by the
Department of Health. In fact, in a letter dated September 3, 2000, the Insurance Commissioner
confirmed that petitioner is not engaged in the insurance business. This determination of the
commissioner must be accorded great weight. It is well-settled that the interpretation of an
administrative agency which is tasked to implement a statute is accorded great respect and
ordinarily controls the interpretation of laws by the courts. The reason behind this rule was
explained in Nestle Philippines, Inc. v. Court of Appeals:
2. No. A Health Care Agreement Is not an insurance contract contemplated under Section
185 of the NIRC of 1997 It is incorrect to say that the health care agreement is not based
on loss or damage because, under the said agreement, petitioner assumes the liability
and indemnifies its member for hospital, medical and related expenses (such as
professional fees of physicians). The term "loss or damage" is broad enough to cover
the monetary expense or liability a member will incur in case of illness or injury.
It does not necessarily follow that a contract containing all the four elements mentioned
in Section 2 of the Insurance Code be an insurance contract. The primary purpose of the parties
in making the contract may negate the existence of an insurance contract.
There is nothing in petitioner's agreements that gives rise to a monetary liability on the
part of the member to any third party-provider of medical services which might in turn
necessitate indemnification from petitioner. The terms "indemnify" or "indemnity" presuppose
that a liability or claim has already been incurred. There is no indemnity precisely because the
member merely avails of medical services to be paid or already paid in advance at a pre-agreed
price under the agreements.
However, assuming that petitioner‘s commitment to provide medical services to its
members can be construed as an acceptance of the risk that it will shell out more than the
prepaid fees, it still will not qualify as an insurance contract because petitioner‘s objective is to
provide medical services at reduced cost, not to distribute risk like an insurer.
Taking into account that health care agreements are clearly not within the ambit of
Section 185 of the NIRC and there was never any legislative intent to impose the same on
HMOs like petitioner, the same should not be arbitrarily and unjustly included in its coverage.

Page 13 of 99
CASE NO. 11. MEDICARD PHILIPPINES, INC., V. CIR, G.R. NO. 222743, APRIL 5, 2017 –
ANTONIO, PHOEBE ANTONETTE

FACTS:

MEDICARD is a health maintenance organization (HMO) that provides prepaid health


and medical insurance coverage to its clients. Individuals enrolled in its health care programs
pay an annual membership fee and are entitled to various preventive, diagnostic and curative
medical services provided by duly licensed physicians, specialists, and other professional
technical staff participating in the group practice health delivery system at a hospital or clinic
owned, operated or accredited by it.

Upon finding some discrepancies between MEDICARD‘s Income Tax Returns (ITR) and
VAT Returns, the CIR issued a Letter Notice (LN) dated September 20, 2007. Subsequently, the
CIR also issued a Preliminary Assessment Notice (PAN) against MEDICARD for deficiency
VAT. MEDICARD received CIR‘s FAN dated December 10, 2007 for allegedly deficiency VAT
for taxable year 2006 including penalties. MEDICARD filed a protest arguing, among others,
that that the services it render is not limited merely to arranging for the provision of medical
and/or hospitalization services but include actual and direct rendition of medical and laboratory
services. On June 19, 2009, MEDICARD received CIR‘s Final Decision denying its protest. The
petitioner MEDICARD proceeded to file a petition for review before the CTA.

The CTA Division held that the determination of deficiency VAT is not limited to the
issuance of Letter of Authority (LOA) alone and that in lieu of an LOA, an LN was issued to
MEDICARD informing it if the discrepancies between its ITRs and VAT Returns and this
procedure is authorized under RMO No. 30-2003 and 42-2003. Also, the amounts that
MEDICARD earmarked and eventually paid to doctors, hospitals and clinics cannot be excluded
from the computation of its gross receipts because the act of earmarking or allocation is by itself
an act of ownership and management over the funds by MEDICARD which is beyond the
contemplation of RR No. 4-2007. Furthermore, MEDICARD‘s earnings from its clinics and
laboratory facilities cannot be excluded from its gross receipts because the operation of these
clinics and laboratory is merely an incident to MEDICARD‘s line of business as an HMO.
MEDICARD filed a Motion for Reconsideration but it was denied. Petitioner elevated the matter
to the CTA en banc. CTA en banc partially granted the petition only insofar as 10% VAT rate for
January 2006 is concerned but sustained the findings of the CTA Division.

ISSUES:

1. Whether the absence of the LOA is fatal. YES

2. Whether the amounts that MEDICARD earmarked and eventually paid to the
medical service providers should still form part of its gross receipts for vat purposes. NO

RULING:

1. The absence of an LOA violated MEDICARD's right to due process

An LOA is the authority given to the appropriate revenue officer assigned to


perform assessment functions. It empowers or enables said revenue officer to examine
the books of account and other accounting records of a taxpayer for the purpose of
collecting the correct amount of tax. A LOA is premised on the fact that the examination

Page 14 of 99
of a taxpayer who has already filed his tax returns is a power that statutorily belongs only
to the CIR himself or his duly authorized representatives.

In this case, there is no dispute that no LOA was issued prior to the issuance of a
PAN and FAN against MEDICARD. Therefore, no LOA was also served on MEDICARD.

2. The amounts earmarked and eventually paid by MEDICARD to the medical


service providers do not form part of gross receipts for VAT purposes.

In Philippine Health Care Providers, Inc. v. Commissioner of Internal Revenue,


the Court adopted the principal object and purpose object in determining whether the
MEDICARD therein is engaged in the business of insurance and therefore liable for
documentary stamp tax. The Court held therein that an HMO engaged in preventive,
diagnostic and curative medical services is not engaged in the business of insurance,
thus to summarize, the distinctive features of the cooperative are the rendering of
service, its extension, the bringing of physician and patient together, the preventive
features, the regularization of service as well as payment, the substantial reduction in
cost by quantity purchasing in short, getting the medical job done and paid for; not,
except incidentally to these features, the indemnification for cost after the services is
rendered. Except the last, these are not distinctive or generally characteristic of the
insurance arrangement.

In sum, the Court said that the main difference between an HMO and an
insurance company is that HMOs undertake to provide or arrange for the provision of
medical services through participating physicians while insurance companies simply
undertake to indemnify the insured for medical expenses incurred up to a pre-agreed
limit. In the present case, the VAT is a tax on the value added by the performance of the
service by the taxpayer. It is, thus, this service and the value charged thereof by the
taxpayer that is taxable under the NIRC.

CASE NO. 12. MITSUBISHI MOTORS PHILIPPINES SALARIED EMPLOYEES UNION


(MMPSEU), V. MITSUBISHI MOTORS PHILIPPINES CORPORATION, G.R. NO. 175773,
JUNE 17, 2013 – VILLAMAR, JOHN OLIVER DUKE

FACTS:

The Collective Bargaining Agreement (CBA) of the parties in this case provides that the
company shoulder the hospitalization expenses of the dependents of covered employees
subject to certain limitations and restrictions. With this, the employees covered by the CBA pay
part of the hospitalization insurance premium through the deduction of their salaries and in turn
the company will pay a portion of the hospitalization expenses.

The conflict arose when a portion of the hospitalization expenses of the covered
employees‘ dependents were paid/shouldered by the dependent‘s own health insurance. On
separate occasions, three members of MMPSEU, namely, Ernesto Calida (Calida), Hermie
Juan Oabel (Oabel) and Jocelyn Martin (Martin), filed claims for reimbursement of
hospitalization expenses of their dependents, however, MMPC paid only a portion of their
hospitalization insurance claims and not the full amount. MMPC only paid for the amounts not
covered by other insurance companies and those covered by official receipts.

Page 15 of 99
The union insists that the covered employees are entitled to the whole and undiminished
amount of said hospital expenses, which should not be reduced by the amounts paid by the
insurance companies. MMPC denied the claims contending that double insurance would result
if the said employees would receive from the company the full amount of hospitalization
expenses despite having already received payment of portions thereof from other health
insurance providers, and that employees should not be allowed to profit from their dependent‘s
loss.

Unable to resolve the problem themselves, MMPSEU referred the dispute to the
National Conciliation and Mediation Board and requested for preventive mediation.

ISSUE:

Whether or not MMPSEU members can collect the full amount of the hospital bills of their
dependents even if a portion was already covered by other insurance companies

HELD:

The MMPC maintains that under the CBA, member-employees are entitled to full
reimbursement of medical expenses incurred by their dependents regardless of any amounts
paid by the latter‘s health insurance provider. Otherwise, non-recovery will constitute unjust
enrichment on the part of MMPC. It avers that recovery from both the CBA and other insurance
companies is allowed under their CBA and not prohibited by law nor by jurisprudence.

The MMPSEU relies on the collateral source rule, however, it only applies in order to
place the responsibility for losses on the party causing them. Thus, it finds no application to
cases involving no-fault insurances under which the insured is indemnified for losses by
insurance companies. Here, it is clear that MMPC is a no-fault insurer.

Moreover, the condition that payment should be direct to the hospital and doctor implies
that MMPC is only liable to pay medical expenses actually shouldered by the employees‘
dependents. It follows that MMPC‘s liability is limited, that is, it does not include the amounts
paid by other health insurance providers

CASE NO. 13. PHILAMCARE V. CA, GR 125678 – CABALO, CASHMIR

FACTS:

Ernani Trinos, deceased husband of respondent Julita Trinos, applied for a health care
coverage with petitioner Philamcare Health Systems, Inc. In the standard application form, he
answered NO to the following question: Have you or any of your family members ever consulted
or been treated for high blood pressure, heart trouble, diabetes, cancer, liver disease, asthma or
peptic ulcer? (If yes, give details)

The healthcare agreement (HCA) was approved for a period of one year and then was
subsequently renewed on the following dates; March 1988 –March 1,March 1-1990 ; March
1,1990 – June 1 ,1990 and the coverage was increased to a maximum sum of P75000.00 per
disability. Under the HCA, Ernani was entitled to the following benefits:

Page 16 of 99
(a)Hospitalization benefits whether ordinary or emergency, etc

(b) Outpatient benefits such annual physical examinations, preventive health care and other
outpatient services.

Moreover, Ernani subsequently suffered a heart attack and was confined at the Manila Medical
Center (MMC) for one month beginning March 9, 1990 (within or during the period of coverage)
and as such, Julita (Ernani‘s wife) filed a claim of the benefits from the Philamcare Health
System however, it was denied by the petitioner due to their claim that the health care card was
void on the grounds that Ernani committed concealment of his medical records. Julita then was
compelled to pay the hospitalization expense amounting to P76, 000.00

After his husband –Ernani was discharge MMC, he was attended by a physical therapist at
home. Later he was admitted at the Chinese General Hospital and was discharge after. On April
13, 1990, Ernani was feeling very weak and died on the same day. On July 24, 1990,
responded Julita, instituted a case before the RTC, an action for damages against the petitioner
and its president Dr. Benito Reverente.

RTC rendered the judgement in favor of Julita Trinos.The.defendant – Philamcare was ordered
to reimburse the medical and hospital bills in the amount of P76, 000.00 plus interest. They
were further required to pay moral damages (P 10,000) exemplary damages (P 10,000) and
attorney‘s fee plus cost of suit.

On appeal, CA affirmed the decision of RTC but deleted all awards of damages and absolved
petitioner Reverente. Petioner‘s motion for reconsideration was also denied.

ISSUE:

1. WON a health care agreement (HCA) are considered insurance contract

2. WON concealment of material facts rendered the HCA void (Invalidation Agreement)

3.Granting there is concealment, what could have Philamcare done

HELD:

Denied , affirmed CA‘s decision.

1. Yes it is an insurance contract

*Section 2 (1) of the Insurance Code defines a contract of insurance as an agreement whereby
one undertakes for a consideration to indemnify another against loss, damage or liability arising
from an unknown or contingent event. An insurance contract exists where the following
elements concur:

1. The insured has an insurable interest;

2. The insured is subject to a risk of loss by the happening of the designated peril;

3. The insurer assumes the risk;

4. Such assumption of risk is part of a general scheme to distribute actual losses among a large
group of persons bearing a similar risk; and

Page 17 of 99
5. In consideration of the insurer‘s promise, the insured pays a premium

In the case at bar, the insurable interest of respondent‘s husband in obtaining the health care
agreement was his own health. The health care agreement was in the nature of non-life
insurance, which is primarily a contract of indemnity. Once the member incurs hospital, medical
or any other expense arising from sickness, injury or other stipulated contingent, the health care
provider must pay for the same to the extent agreed upon under the contract

2. No, there was no concealment on the part of Ernani.

The answer assailed by petitioner was in response to the question relating to the medical history
of the applicant. This largely depends on opinion rather than fact, especially coming from
respondent‘s husband who was not a medical doctor. Where matters of opinion or judgment are
called for, answers made in good faith and without intent to deceive will not avoid a policy even
though they are untrue.

3. Under Section 27 of the Insurance Code, "a concealment entitles the injured party to
rescind a contract of insurance." The right to rescind should be exercised previous to the
commencement of an action on the contract. In this case, no rescission was made. Besides, the
cancellation of health care agreements as in insurance policies require the concurrence of the
following conditions:

1. Prior notice of cancellation to insured;

2. Notice must be based on the occurrence after effective date of the policy of one or more of
the grounds mentioned;

3. Must be in writing, mailed or delivered to the insured at the address shown in the policy;

4. Must state the grounds relied upon provided in Section 64 of the Insurance Code and upon
request of insured, to furnish facts on which cancellation is based.

None of the above pre-conditions was fulfilled in this case. When the terms of insurance
contract contain limitations on liability, courts should construe them in such a way as to preclude
the insurer from non-compliance with his obligation. Being a contract of adhesion, the terms of
an insurance contract are to be construed strictly against the party which prepared the contract
– the insurer. By reason of the exclusive control of the insurance company over the terms and
phraseology of the insurance contract, ambiguity must be strictly interpreted against the insurer
and liberally in favor of the insured, especially to avoid forfeiture. This is equally applicable to
Health Care Agreements. The phraseology used in medical or hospital service contracts, such
as the one at bar, must be liberally construed in favor of the subscriber, and if doubtful or
reasonably susceptible of two interpretations the construction conferring coverage is to be
adopted, and exclusionary clauses of doubtful import should be strictly construed against the
provider

CASE NO. 14. GULF RESORTS, INC. V. PHILIPPINE CHARTER INSURANCE


CORPORATION, GR NO. 156167, MAY 16, 2005 – PASCUA, MA. BETINA GRACE

Facts:

Gulf Resorts, Inc. at Agoo, La Union was originally insured with American Home
Assurance Company (AHAC) which includes loss or damage to shock to any of the property
insured by this Policy occasioned by or through or in consequence of earthquake.

Page 18 of 99
After the earthquake, petitioner advised respondent that it would be making a claim
under its Insurance Policy No. 31944 for damages on its properties. Petitioner avers that,
pursuant to its earthquake shock endorsement rider, Insurance Policy No. 31944 covers all
damages to the properties within its resort caused by earthquake. Respondent contends that
the rider limits its liability for loss to the two swimming pools of petitioner.

The Regional Trial Court favoured AHAC ruling that the endorsement rider means that
only the two swimming pools were insured against the earthquake shock. CA affirmed RTC‘s
decision.

Issue:

Whether petitioner can claim for all its properties and not just the two (2) swimming
pools.

Ruling:

The petition is devoid of merit.

It is basic that all the provisions of the insurance policy should be examined and
interpreted in consonance with each other. All its parts are reflective of the true intent of the
parties. The policy cannot be construed piecemeal. Certain stipulations cannot be segregated
and then made to control; neither do particular words nor phrases necessarily determine its
character.

Petitioner cannot focus on the earthquake shock endorsement to the exclusion of the
other provisions. All the provisions and riders, taken and interpreted together, indubitably show
the intention of the parties to extend earthquake shock coverage to the two swimming pools
only.

A careful examination of the premium recapitulation will show that it is the clear intent of
the parties to extend earthquake shock coverage only to the two swimming pools. Section 2(1)
of the Insurance Code defines a contract of insurance as an agreement whereby one
undertakes for a consideration to indemnify another against loss, damage or liability arising from
an unknown or contingent event. Thus, an insurance contract exists where the following
elements concur:

1. The insured has an insurable interest;

2. The insured is subject to a risk of loss by the happening of the designated peril;

3. The insurer assumes the risk;

4. Such assumption of risk is part of a general scheme to distribute actual losses among a large
group of persons bearing a similar risk; and

5. In consideration of the insurer's promise, the insured pays a premium.

An insurance premium is the consideration paid an insurer for undertaking to indemnify


the insured against a specified peril. In the subject policy, no premium payments were made
with regard to earthquake shock coverage, except on the two swimming pools. There is no
mention of any premium payable for the other resort properties with regard to earthquake shock.

Page 19 of 99
CASE NO. 15. FILIPINO MERCHANTS INSURANCE CO., INC. VS. COURT OF APPEALS
AND CHOA TIEK SENG, 179 SCRA 638, G.R. NO. 85141, NOVEMBER 28, 1989 – CAPULE,
STELLA MARIE

FACTS:

Choa Tiek Seng, consignee of the shipment of fishmeal loaded, seeks to recover from
insurance company which represent damages to shipment. Consignee insured in "all risks
policy" 600 metric tons of fishmeal from Bangkok, Thailand to Manila against all risks under
warehouse to warehouse terms. The fishmeal bags were unloaded from the ship at Manila unto
the arrastre contractor, E. Razon, Inc. and Filipino Merchants‘ surveyor ascertained and certified
that the discharge of 105 bags were in bad condition. Choa filed claim against the Filipino
Merchants Insurance Co. (insurer) ; however, the latter refused to pay the claim and brought a
third party complaint against Compagnie Maritime Des Chargeurs Reunis and/or E. Razon, Inc.

The RTC rendered a judgment against Filipino Merchants and ordered them to pay
reimbursement from Compagnie Maritime Des Chargeurs Reunis and third part defendant, E.
Razon, Inc. The CA affirmed the decision with modification regarding the adjudication of the
third-party complaint.

Filipino Merchants filed an appeal and contended that the CA erred in not holding that
Chao had no insurable interest in the subject cargo, hence, the marine insurance policy taken
out by private respondent is null and void.

ISSUES:

1. Whether Filipino Merchants Co. should be held liable.

2. Whether Choa Tiek Seng as a consignee of the shipment has insurable interest.

RULING:

1. Yes. An "all risks policy" should be read literally as meaning all risks whatsoever and
covering all losses by an accidental cause of any kind. The terms "accident" and "accidental",
as used in insurance contracts, have not acquired any technical meaning. The very nature of
the term "all risks" must be given a broad and comprehensive meaning as covering any loss
other than a willful and fraudulent act of the insured. This is pursuant to the very purpose of an
"all risks" insurance to give protection to the insured in those cases where difficulties of logical
explanation or some mystery surround the loss or damage to property.

The insured under an "all risks insurance policy" has the initial burden of proving that the
cargo was in good condition when the policy attached and that the cargo was damaged when
unloaded from the vessel; thereafter, the burden then shifts to the insurer to show the exception
to the coverage. 10 As we held in Paris-Manila Perfumery Co. vs. Phoenix Assurance Co., Ltd.
11 the basic rule is that the insurance company has the burden of proving that the loss is
caused by the risk excepted and for want of such proof, the company is liable.

There being no showing that the loss was caused by any of the excepted perils, the
insurer is liable under the policy.

2. Yes. Anent the issue of insurable interest, we uphold the ruling of the respondent court
that private respondent, as consignee of the goods in transit under an invoice containing the
terms under "C & F Manila," has insurable interest in said goods.

Page 20 of 99
Section 13 of the Insurance Code defines insurable interest in property as every interest
in property, whether real or personal, or any relation thereto, or liability in respect thereof, of
such nature that a contemplated peril might directly damnify the insured

Herein private respondent, as vendee/consignee of the goods in transit has such


existing interest therein as may be the subject of a valid contract of insurance. His interest over
the goods is based on the perfected contract of sale. The perfected contract of sale between
him and the shipper of the goods operates to vest in him an equitable title even before delivery
or before be performed the conditions of the sale.

CASE NO. 16. ANG KA YU V PHOENIX ASSURANCE CO. LTD. 1 CAR 704, SEPTEMBER
28, 1961 – DELA CRUZ, JEREMY KIMBERLY

FACTS:

Ang Ka Yu, herein petitioner, was engaged in the business of dyeing and washing
clothes. This would require his clients to deliver and deposit to the petitioner such clothes that
would require his service. He acquired from Phoenix Assurance Co. Ltd., herein respondent, a
policy insuring the effects of his business. When the clothes he had in his possession were lost,
Ang Ka Yu sought to recover from Phoenix Assurance. However, the latter refused the claim
and denied liability on the ground that the petitioner was a mere possessor of said items, and
therefore did not have any insurable interest in the same.

ISSUE:

Whether or not a mere possessor has insurable interest over the property.

HELD:

Yes. A person having a mere right or possession of property may insure it to its full value
and in his own name, even when he is not responsible for its safekeeping. The reason is that
even if a person is not interested in the safety and preservation of material in his possession
because they belong to third parties, said person still has insurable interest, because he stands
either to benefit from their continued existence or to be prejudiced by their destruction.

A person is said to have an insurable interest in the subject matter insured where he has
a relation or connection with, or concern in it that he will derive pecuniary benefit or advantage
from its preservation and will suffer pecuniary loss or damage from its destruction, termination,
or injury by the happening of the event insured against.

CASE NO. 17. UCPB GENERAL, INSURANCE CO., INC., PETITIONER, VS. ASGARD
CORRUGATED BOX MANUFACTURING CORPORATION, RESPONDENT., G.R. NO.
244407, JANUARY 26, 2021 – MANGAOANG, NICOLE JOYCE

FACTS: Asgard and Milestone Paper Products, Inc. entered into a Toll Manufacturing
Agreement (TMA). Asgard undertook to perform toll-manufacturing of paper products for
Milestone.

Milestone installed new equipment for the manufacturing plant and paper mill. After months of
managing and operating the business, Milestone accepted Asgard's invitation, after months of
managing and operating the business.

Page 21 of 99
On August 7, 2009, Asgard and Milestone took out an insurance policy UCPB Insurance. Upon
payment of insurance premium, UCPB issued Industrial All Risk Policy to Milestone and/or
Market Link and/or Nova Baile and/or Asgard to insure Asgard's machinery and equipment of
every kind.

But on July 15, 2010, Milestone pulled out its stocks, machinery, and equipment from Asgard's
plant in Novaliches, Quezon City for relocation to Milestone's own premises in Laguna. In the
course thereof, it caused damage to Asgard's complete line of Isowa corrugating machine and
accessories as well as its printer-slotter-stacker. Asgard notified UCPB Insurance about the loss
and filed an insurance claim under the Policy.

UCPB Insurance denied the claim explaining that the Policy had no cross liability cover, and the
malicious damage was committed by Milestone, one of the name insured, and not committed by
a third party.

ISSUE/s:
● Whether Milestone had insurable interest over Asgard's corrugating machines at the
time of the loss or damage.
○ Whether UCPB Insurance is liable to Asgard for its insurance claim in the
amount of Php 147,000,000.00.

RULING:

NO.

The Insurance Code (Republic Act No. 10607) provides:

Section 89. An insurer is not liable for a loss caused by the willful act or through the
connivance of the insured; but he is not exonerated by the negligence of the insured, or
of the insurance agents or others.

Such damage/loss is not an insurable risk because the occurrence of the loss was subject to the
control of one of the parties and not merely caused by the negligence of the insured.

Under the heading "The Insurer Shall Not Be Liable For" or the exception clause written in the
policy, the Insurer (UCPB Insurance) shall not be liable for: (f) Loss or damage arising out of
the willful act or gross negligence of the insured or of his representatives; (Emphasis
supplied). Since the damage or loss caused by Milestone to Asgard's corrugating machines was
willful or intentional, UCPB Insurance is not liable under the Policy. To permit Asgard to recover
from the Policy for a loss caused by the willful act of the insured is contrary to public policy, i.e.,
denying liability for willful wrongs.

Even Asgard described the act done by Milestone as malicious; therefore, it is intentional and
not accidental.

● NO. The RTC merely relied on the quotation (Exh. "K" and series) and the 98 Philippine
Business Bank checks (Exhs. "R" to "KKKKK"). The basis of indemnity under the Policy
is the replacement cost of the property insured. To put its damaged corrugating
machines back in operation, Asgard claimed that the damaged parts had to be replaced.

Page 22 of 99
The party who alleges a fact has the burden of proving it. Since it is Asgard claiming for actual
damages or insurance claim against UCPB Insurance, it bears the burden of proof to
substantiate its claim.

Asgard did not explain the extent of the damage of its corrugating machines. It failed to show
whether the damaged machines are integral part of the stocks, machinery and equipment pulled
out by Milestone; whether these damaged machines are independent machinery, i.e., it can
operate on its own without the parts installed by Milestone; and whether the stocks, machinery
and equipment installed by Milestone are detachable, or it will not cause any damage when
detached. The Court cannot just rely on the Taiphil quotation to determine the amount of actual
loss, the PNB checks issued and deposited to Taiphil's account as proof of payment, or the
pictures of the damaged machines.

What was presented was a mere "quotation" not reliable and competent evidence.

CASE NO. 18. VICENTE ONG LIM SING, JR. VS. FEB LEASING & FINANCE CORP., G.R.
NO. 168115, JUNE 8, 2007 – GACAYAN, HAZEL FAYE

FACTS:

On March 9, 1995, FEB Leasing and Finance Corporation (FEB) entered into a lease of
equipment and motor vehicles with JVL Food Products (JVL). On the same date, Vicente Ong
Lim Sing, Jr. (Lim) executed an Individual Guaranty Agreement with FEB to guarantee the
prompt and faithful performance of the terms and conditions of the aforesaid lease agreement.
Corresponding Lease Schedules with Delivery and Acceptance Certificates over the equipment
and motor vehicles formed part of the agreement. Under the contract, JVL was obliged to pay
FEB an aggregate gross monthly rental of P170,494.00.

JVL defaulted in the payment of the monthly rentals. As of July 31, 2000, the amount in
arrears, including penalty charges and insurance premiums, amounted to P3,414,468.75. On
August 23, 2000, FEB sent a letter to JVL demanding payment of the said amount. However,
JVL failed to pay.

On December 6, 2000, FEB filed a Complaint with the Regional Trial Court of Manila,
docketed as Civil Case No. 00-99451, for sum of money, damages, and replevin against JVL,
Lim, and John Doe.

JVL and Lim admitted the existence of the lease agreement but asserted that it is in reality
a sale of equipment on installment basis, with FEB acting as the financier. JVL and Lim claimed
that this intention was apparent from the fact that they were made to believe that when full
payment was effected, a Deed of Sale will be executed by FEB as vendor in favor of JVL and
Lim as vendees. FEB purportedly assured them that documenting the transaction as a lease
agreement is just an industry practice and that the proper documentation would be effected as
soon as full payment for every item was made. They also contended that the lease agreement is
a contract of adhesion and should, therefore, be construed against the party who prepared it like
the FEB.

The RTC held that it is a sale on installment. There is no chattel mortgage on the thing sold,
but it appears that the plaintiff elected to exact fulfillment of the obligation. For the vehicles
returned, the plaintiff can only recover the unpaid balance of the price. However, with respect to

Page 23 of 99
the unreturned units and machineries still in the possession of the defendants, the defendants
are jointly and severally liable

The CA then reversed the decision of RTC. The transaction between the parties is that of a
financial lease agreement under RA 8556. It ordered JVL and Lim jointly and severally to pay
P3, 414,468.75.

ISSUES:

1. Whether or not JVL as the lessee have an insurable interest over the leased items.
2. Whether or not JVL and Lim should jointly and severally be liable for the insured financial
lease

HELD:

(1) Yes. The stipulation in Section 14 of the lease contract, that the equipment shall be
insured at the cost and expense of the lessee against loss, damage, or destruction from fire,
theft, accident, or other insurable risk for the full term of the lease, is a binding and valid
stipulation. Petitioner, as a lessee, has an insurable interest in the equipment and motor
vehicles leased. Section 17 of the Insurance Code provides that the measure of an insurable
interest in property is the extent to which the insured might be damnified by loss or injury
thereof. It cannot be denied that JVL will be directly damnified in case of loss, damage, or
destruction of any of the properties leased. It has also been held that the test of insurable
interest in property is whether the assured has a right, title or interest therein that he will be
benefited by its preservation and continued existence or suffer a direct pecuniary loss from its
destruction or injury by the peril insured against.

(2) Yes. While affirming that the subject lease agreement is a contract of adhesion, the
Court believes that such a contract is not void per se. It is as binding as any ordinary contract. A
party who enters into an adhesion contract is free to reject the stipulations entirely. If the terms
thereof are accepted without objection, then the Court serves as the law between the parties.
Petitioner‘s claim that the real intention of the parties was a contract of sale of personal property
on installment basis is more likely a mere afterthought in order to defeat the rights of the
respondent. The lease contract with corresponding Lease Schedules with Delivery and
Acceptance Certificates is, in point of fact, a financial lease within the purview of RA 8556.

CASE NO. 19. RIZAL COMMERCIAL BANKING CORPORATION VS. CA, 289 SCRA 292
(1998) – LAUSAN, PENNELOPE

FACTS:

When GOYU & Sons, Inc. (GOYU) obtained a credit facility from Rizal Commercial
Banking Corporation (RCBC) where it was granted the amount of P30 million was initially it
executed a mortgage contract in favor of the bank wherein it was expressly stipulated that
GOYU will insure all the subject properties with an insurance company approved by the bank
and to endorse and deliver the policies to the bank. Upon GOYU's application, its credit facility
increase to P50 million, then to P90 million, and finally to P117 million

GOYU, through Alchester Insurance Agency, Inc., obtained in its name a total of 10
insurance policies from Malayan Insurance Company, Inc. (MICO), sister company of RCBC,

Page 24 of 99
and endorsed them in favor of RCBC. Copies of the endorsements were sent and received by
GOYU, RCBC and MICO. GOYU continued to enjoy the benefits of the credit facilities extended
to it by the bank.

When GOYU‘s factory buildings were gutted by fire, GOYU and RCBC filed separate
claims for indemnity with MICO. However, both were denied on the ground that the insurance
policies were either attached pursuant to writs of attachments/garnishments issued by various
courts or that the insurance proceeds were also claimed by other creditors of GOYU alleging
better rights to the proceeds than the insured.

GOYU then filed a complaint for specific performance and damages disowning the
endorsements or lack of authority of Alchester to prepare and issue said endorsements in favor
of RCBC. RCBC, one of GOYU's creditors, also filed with MICO its formal claim over the
proceeds of the insurance policies, but said claims were also denied for the same reasons that
Alchester denied GOYU's claims.

The trial court, as well as the Court of Appeals, concluded that the endorsements are
defective and held that RCBC has no right over the insurance proceeds. A judgment, therefore,
was rendered ordering, among others, MICO to pay fire loss claim of GOYU while ordering
MICO and RCBC to pay damages. GOYU was ordered to pay its loan obligations with RCBC
with interests.

ISSUE:

Whether RCBC, as mortgagee, has any right over the insurance policies taken by
GOYU, the mortgagor, in case of the occurrence of loss.

HELD:

YES.

It is settled that a mortgagor and a mortgagee have separate and distinct insurable
interests in the same mortgaged property, such that each one of them may insure the same
property for his own sole benefit. There is no question that GOYU could insure the mortgaged
property for its own exclusive benefit. In the present case, although it appears that GOYU
obtained the subject insurance policies naming itself as the sole payee, the intentions of the
parties as shown by their contemporaneous acts, must be given due consideration in order to
better serve the interest of justice and equity.

GOYU continued to enjoy the benefits of the credit facilities extended to it by RCBC.
After the occurrence of the loss insured against, it was too late for GOYU to disown the
endorsements for any imagined or contrived lack of authority of Alchester to prepare and issue
said endorsements. If there had not been actually an implied ratification of said endorsements
by virtue of GOYU‘s inaction in this case, GOYU is at the very least estopped from assailing
their operative effects. Under the peculiar circumstances obtaining in this case, the Court is

Page 25 of 99
bound to recognize RCBC‘s right to the proceeds of the insurance policies if not for the actual
endorsement of the policies, at least on the basis of the equitable principle of estoppel.

GOYU cannot seek relief under Section 53 of the Insurance Code which provides that
the proceeds of insurance shall exclusively apply to the interest of the person in whose name or
for whose benefit it is made. It having been sufficiently established that it was the intention of
the parties to designate RCBC as the party for whose benefit the insurance policies were taken
out. Just as plain too is the intention of the parties to constitute RCBC as the beneficiary of the
various insurance policies obtained by GOYU. The intention of the parties will have to be given
full force and effect in this particular case. The insurance proceeds may, therefore, be
exclusively applied to RCBC, which under the factual circumstances of the case, is truly the
person or entity for whose benefit the policies were clearly intended. Moreover, the law‘s evident
intention to protect the interests of the mortgagee upon the mortgaged property is expressed in
Article 2127 of the Civil Code.

Finally, it is basic and fundamental that the first mortgagee has superior rights over
junior mortgagees or attaching creditors. Section 53 of the Insurance Code ordains that the
insurance proceeds of the endorsed policies shall be applied exclusively to the proper interest of
the person for whose benefit it was made. In this case, to the extent of GOYU‘s obligation with
RCBC, the interest of GOYU in the subject policies had been transferred to RCBC effective as
of the time of the endorsement. These policies may no longer be attached by the other creditors
of GOYU.

CASE NO. 20. ARMANDO GEAGONIA V. CA AND COUNTRY BANKERS INSURANCE


CORP., G.R. NO. 114427, FEBRUARY 6, 1995 – SOMERA, MARIA MONICA

FACTS:

Petitioner, as the owner of Norman‘s Mart, obtained insurance from private respondent
CBIC. The insurance policy contained the following condition:

"3. The insured shall give notice to the Company of any insurance or insurances already
effected, or which may subsequently be effected, covering any of the property or
properties consisting of stocks in trade, goods in process and/or inventories only hereby
insured, and unless such notice be given and the particulars of such insurance or
insurances be stated therein or endorsed in this policy pursuant to Section 50 of the
Insurance Code, by or on behalf of the Company before the occurrence of any loss or
damage, all benefits under this policy shall be deemed forfeited, provided however, that
this condition shall not apply when the total insurance or insurances in force at the time
of the loss or damage is not more than P200,000.00."

The building subject of fire insurance was razed by fire. Consequently, Petitioner
claimed before CBIC for the proceeds. CBIC refused alleging that Petitioner did not inform of a
previous insurance obtained by its creditor Cebu Tesing Textiles over the same property and in
violation of Condition 3.

Page 26 of 99
ISSUES:

1) Whether the non-disclosure of other insurance policies violate condition 3 of the policy,
so as to deny Geagonia from recovering on the policy.
2) Whether the violation of Condition 3 of the policy renders the policy void.

RULING:

1) Condition 3 of Country Bankers's Policy is a condition which is not proscribed by law. Its
incorporation in the policy is allowed by Section 75 of the Insurance Code, Such a condition is a
provision which invariably appears in fire insurance policies and is intended to prevent an
increase in the moral hazard. It is commonly known as the additional or "other insurance" clause
and has been upheld as valid and as a warranty that no other insurance exists. Its violation
would thus avoid the policy. However, in order to constitute a violation, the other insurance must
be upon the same subject matter, the same interest therein, and the same risk.

The fire insurance policies issued by the PFIC name Geagonia as the assured and
contain a mortgage clause which reads: "Loss, if any, shall be payable to MESSRS. TESING
TEXTILES, Cebu City as their interest may appear subject to the terms of the policy." This is
clearly a simple loss payable clause, not a standard mortgage clause. The Court concludes that
(a) the prohibition in Condition 3 of the subject policy applies only to double insurance, and (b)
the nullity of the policy shall only be to the extent exceeding P200,000.00 of the total policies
obtained. The first conclusion is supported by the portion of the condition referring to other
insurance "covering any of the property or properties consisting of stocks in trade, goods in
process and/or inventories only hereby insured," and the portion regarding the insured's
declaration on the subheading CO-INSURANCE that the co-insurer is Mercantile Insurance Co.,
Inc. in the sum of P50,000.00.

A double insurance exists where the same person is insured by several insurers
separately in respect of the same subject and interest. Since the insurable interests of a
mortgagor and a mortgagee on the mortgaged property are distinct and separate; the two
policies of the PFIC do not cover the same interest as that covered by the policy of Country
Bankers, no double insurance exists. The non-disclosure then of the former policies was not
fatal to Geagonia's right to recover on Country Bankers' policy.

2) No. Unlike the "other insurance" clauses involved in General Insurance and Surety Corp. vs.
Ng Hua, 106 Phil. 1117 [1960], or in Pioneer Insurance & Surety Corp. vs. Yap, 61 SCRA 426
[1974] which reads "The insured shall give notice to the company of any insurance or
insurances already effected, or which may subsequently be effected covering any of the
property hereby insured, and unless such notice be given and the particulars of such insurance
or insurances be stated in or endorsed on this Policy by or on behalf of the Company before the
occurrence of any loss or damage, all benefits under this Policy shall be forfeited"; or in the
1930 case of Santa Ana vs. Commercial Union Assurance Co., Phil. 329, 334 [1930], which
provided "that any outstanding insurance upon the whole or a portion of the objects thereby
assured must be declared by the insured in writing and he must cause the company to add or
insert it in the policy, without which such policy shall be null and void, and the insured will not be
entitled to indemnity in case of loss," Condition 3 in Country Bankers' policy does not absolutely
declare void any violation thereof. It expressly provides that the condition "shall not apply when
the total insurance or insurances in force at the time of the loss or damage is not more than
P200,000.00."

Page 27 of 99
By stating within Condition 3 itself that such condition shall not apply if the total
insurance in force at the time of loss does not exceed P200,000.00, Country Bankers was
amenable to assume a co-insurer's liability up to a loss not exceeding P200,000.00. What it had
in mind was to discourage over-insurance.

CASE NO. 21. SPOUSES CHA VS. CA, G.R. NO. 124520, AUGUST 18, 1997 – CAPULE,
STELLA MARIE

FACTS:

Petitioner-spouses Nilo Cha and Stella Uy-Cha, as lessees, entered into a lease contract with
private respondent CKS Development Corporation (hereinafter CKS), as lessor. One of the
stipulations of the one (1) year lease contract states:

x x x. The LESSEE shall not insure against fire the chattels,


merchandise, textiles, goods and effects placed at any stall or store
or space in the leased premises without first obtaining the written
consent and approval of the LESSOR. If the LESSEE obtain(s) the
insurance thereof without the consent of the LESSOR then the
policy is deemed assigned and transferred to the LESSOR for its
own benefit; x x x

Notwithstanding the above stipulation in the lease contract, the Cha spouses insured against
loss by fire their merchandise inside the leased premises for P500,000.00 with the United
Insurance Co., Inc. without the written consent of private respondents CKS. On the day that the
lease contract was to expire, fire broke out inside the leased premises.

When CKS learned of the insurance earlier procured by the Cha spouses (without its consent),
it wrote the insurer (United) a demand letter asking that the proceeds of the insurance contract
(between the Cha spouses and United) be paid directly to CKS, based on its lease contract with
Cha spouses. United refused to pay CKS. Hence, the latter filed a complaint against the Cha
spouses and United.

RTC rendered a judgment ordering United to pay CKS the amount of P335,063.11 and the
defendant spouses to pay damages and attorneys fees. On appeal, CA affirmed the trial court
decision with modification deleting the awards for damages and attorneys fees.

ISSUE:

Whether or not the aforequoted paragraph 18 of the lease contract entered into between CKS
and the Cha spouses is valid insofar as it provides that any fire insurance policy obtained by the
lessee (Cha spouses) over their merchandise inside the leased premises is deemed assigned
or transferred to the lessor (CKS) if said policy is obtained without the prior written of the latter.

RULING:

No. Sec. 18 of the Insurance Code provides:

Sec. 18. No contract or policy of insurance on property shall


be enforceable except for the benefit of some person having
an insurable interest in the property insured.

Page 28 of 99
A non-life insurance policy such as the fire insurance policy taken by petitioner-spouses over
their merchandise is primarily a contract of indemnity. Insurable interest in the property insured
must exist at the time the insurance takes effect and at the time the loss occurs. The basis of
such requirement of insurable interest in property insured is based on sound public policy: to
prevent a person from taking out an insurance policy on property upon which he has no
insurable interest and collecting the proceeds of said policy in case of loss of the property. In
such a case, the contract of insurance is a mere wager which is void under Section 25 of the
Insurance Code.

Therefore, respondent CKS cannot, under the Insurance Code a special law be validly a
beneficiary of the fire insurance policy taken by the petitioner-spouses over their merchandise.
This insurable interest over said merchandise remains with the insured, the Cha spouses.

The automatic assignment of the policy to CKS under the provision of the lease contract
previously quoted is void for being contrary to law and/or public policy. The proceeds of the fire
insurance policy thus rightfully belong to the spouses. The insurer (United) cannot be compelled
to pay the proceeds of the fire insurance policy to a person (CKS) who has no insurable interest
in the property insured.

CASE NO. 22. GREAT PACIFIC VS. CA, 89 SCRA 543 – VILLAMAR, JOHN OLIVER DUKE

FACTS:

In 1957, Ngo Hing filed an application with petitioner Great Pacific Life Assurance
Company for a twenty-year endowment policy in the amount of 50,000 pesos on the life of his
one-year-old daughter Helen Go.

Respondent supplied essential data which Modragon, Branch Manager of the Pacific Line in
Cebu City wrote on the form with his handwriting. Mondragon then types the data on the
application form to which was subsequently signed by Ngo Hing. go Hing paid the annual
premium sum and after payment of the insurance premium, the binding deposit receipt was
issued to private respondent Ngo Hing.

Mondragon, received a letter stating that the said life insurance application for 20-year
endowment plan is not available for minors below seven years old, but Pacific Life can consider
the same under the Juvenile Triple Action Plan, and advised that if the offer is acceptable, the
Juvenile Non-Medical Declaration be sent to the company. Mondragon did not inform Ngo Hing
of the non-acceptance of the insurance plan but instead he wrote back to Pacific Life strongly
recommending the approval of the 20-year endowment insurance plan to children, pointing out
that since 1954 the customers, especially the Chinese, were asking for such coverage.

After a few days, Helen Go died of influenza with complications of bronchopneumonia.

ISSUE:

1. Whether the binding deposit receipt (Exhibit E) constituted a temporary contract of the
life insurance in question

2. Whether private respondent Ngo Hing concealed the state of health and physical condition
of Helen Go, which rendered void the aforesaid Exhibit E.

Page 29 of 99
HELD:

1. NO.

The receipt was intended to be merely a provisional insurance contract. Its perfection
was subject to compliance of the following conditions: (1) that the company shall be satisfied
that the applicant was insurable on standard rates; (2) that if the company does not accept the
application and offers to issue a policy for a different plan, the insurance contract shall not be
binding until the applicant accepts the policy offered; otherwise, the deposit shall be refunded;
and (3) that if the company disapproves the application, the insurance applied for shall not be in
force at any time, and the premium paid shall be returned to the applicant.

The receipt is merely an acknowledgment that the latter's branch office had received
from the applicant the insurance premium and had accepted the application subject for
processing by the insurance company. There was still approval or rejection the same on the
basis of whether or not the applicant is "insurable on standard rates." Since Pacific Life
disapproved the insurance application of respondent Ngo Hing, the binding deposit receipt in
question had never become in force at any time. The binding deposit receipt is conditional and
does not insure outright. This was held in Lim v Sun. The deposit paid by private respondent
shall have to be refunded by Pacific Life.

1. YES.

This Court is of the firm belief that private respondent had deliberately concealed the
state of health and physical condition of his daughter Helen Go. Where private respondent
supplied the required essential data for the insurance application form, he was fully aware that
his one-year-old daughter is typically a mongoloid child. Such a congenital physical defect could
never be ensconced nor distinguished.

Nonetheless, private respondent, in apparent bad faith, withheld the fact material to the
risk to be assumed by the insurance company.

We are thus constrained to hold that no insurance contract was perfected between the parties
with the noncompliance of the conditions provided in the binding receipt, and concealment, as
legally defined, having been committed by herein private respondent.

CASE NO. 23. DEVELOPMENT BANK OF THE PHILIPPINES V. COURT OF APPEALS AND
THE ESTATE OF THE LATE JUAN B. DANS, G.R. NO. L-109937 – DE GUZMAN, YVONNE
CHRISTELLE

Facts:

Juan B. Dans applied for a loan of P500,000.00 with the Development Bank of the
Philippines (DBP). As the principal mortgagor, Dans, then 76 years of age, was advised by DBP
to obtain a mortgage redemption insurance (MRI) with the DBP Mortgage Redemption
Insurance Pool.

Page 30 of 99
A loan, in the reduced amount of P300,000.00, was approved by DBP. From the
proceeds of the loan, DBP deducted the amount of P1,476.00 as payment for the MRI premium.

On August 20, 1987, the MRI premium of Dans, less the DBP service fee of 10 percent,
was credited by DBP to the savings account of the DBP MRI Pool.

On September 3, 1987, Dans died of cardiac arrest. The DBP, relayed this information to
the DBP MRI Pool, however the latter notified that Dans was not eligible for MRI coverage,
being over the acceptance age limit of 60 years at the time of application.

DBP apprised Candida Dans of the disapproval of her late husband's MRI application.
The DBP offered to refund the premium. Candida refused and demanded the face value of the
MRI on the amount equivalent to the load. The administratrix, filed a complaint against DBP
and the insurance pool for "Collection of Sum of Money with Damages.

Issue/s:

1. Whehter the DBP MRI Pool is liable?

2. Whther DBP shall be held liable

Held:

1. The MRI coverage shall take effect: (1) when the application shall be approved by the
insurance pool; and (2) when the full premium is paid during the continued good health of the
applicant. These two conditions, being joined conjunctively, must concur.

Undisputably, the power to approve MRI applications is lodged with the DBP MRI Pool. The
pool, however, did not approve the application of Dans. There is also no showing that it
accepted the sum of P1,476.00, which DBP credited to its account with full knowledge that it
was payment for Dans‘s premium. There was, as a result, no perfected contract of insurance;
hence, the DBP MRI Pool cannot be held liable on a contract that does not exist.

2. The liability of DBP is another matter.

It was DBP, as a matter of policy and practice, that required Dans, the borrower, to secure MRI
coverage with DBP MRI Pool. In dealing with Dans, DBP was wearing two legal hats: the first as
a lender, and the second as an insurance agent.

As an insurance agent, DBP made Dans go through the motion of applying for said insurance,
thereby leading him and his family to believe that they had already fulfilled all the requirements
for the MRI. Under Article 1897 of the Civil Code of the Philippines, "the agent who acts as such
is not personally liable to the party with whom he contracts, unless he expressly binds himself or
exceeds the limits of his authority without giving such party sufficient notice of his powers."

The DBP is not authorized to accept applications for MRI when its clients are more than 60
years of age. The liability of an agent who exceeds the scope of his authority depends upon
whether the third person is aware of the limits of the agent‘s powers. There is no showing that
Dans knew of the limitation on DBP‘s authority to solicit applications for MRI.

Page 31 of 99
CASE NO. 24. EXCEPTION TO THE RULE: ETERNAL GARDENS MEMORIAL PARK V.
PHILAM LIFE, G.R. NO. 166245, APRIL 9, 2008 – CAWIS, AUDREY

FACTS:

Philamlife entered into an agreement denominated as Creditor Group Life Policy No. P-
19202 with petitioner Eternal Gardens Memorial Park Corporation (Eternal). Under the policy,
the clients of Eternal who purchased burial lots from it on installment basis would be insured by
Philamlife. The amount of insurance coverage depended upon the existing balance of the
purchased burial lots.

Eternal was required under the policy to submit to Philamlife a list of all new lot
purchasers, together with a copy of the application of each purchaser, and the amounts of the
respective unpaid balances of all insured lot purchasers. In relation to the instant petition,
Eternal complied by submitting a letter dated December 29, 1982,4 containing a list of insurable
balances of its lot buyers for October 1982. One of those included in the list as "new business"
was a certain John Chuang. His balance of payments was PhP 100,000. On August 2, 1984,
Chuang died.

Eternal sent a letter dated August 20, 19845 to Philamlife, which served as an insurance
claim for Chuang's death.

After more than a year, Philamlife had not furnished Eternal with any reply to the latter's
insurance claim. This prompted Eternal to demand from Philamlife the payment of the claim for
PhP 100,000 on April 25, 1986.

In response to Eternal's demand, Philamlife denied Eternal's insurance claim in a letter


dated May 20, 1986. Consequently, Eternal filed a case before the Makati City Regional Trial
Court (RTC).

ISSUE:

May the inaction of the insurer on the insurance application be considered as approval of
the application?

RULING:

YES. As earlier stated, Philamlife and Eternal entered into an agreement denominated
as Creditor Group Life Policy No. P-1920 dated December 10, 1980. In the policy, it is provided
that:

EFFECTIVE DATE OF BENEFIT.

The insurance of any eligible Lot Purchaser shall be effective on the date he contracts a
loan with the Assured. However, there shall be no insurance if the application of the Lot
Purchaser is not approved by the Company.

An examination of the above provision would show ambiguity between its two sentences. The
first sentence appears to state that the insurance coverage of the clients of Eternal already
became effective upon contracting a loan with Eternal while the second sentence appears to
require Philamlife to approve the insurance contract before the same can become effective.

Page 32 of 99
It must be remembered that an insurance contract is a contract of adhesion which must
be construed liberally in favor of the insured and strictly against the insurer in order to safeguard
the latter's interest.

The fact of the matter is, the letter dated December 29, 1982, which Philamlife stamped
as received, states that the insurance forms for the attached list of burial lot buyers were
attached to the letter. Such a stamp of receipt has the effect of acknowledging receipt of the
letter together with the attachments. Such receipt is an admission by Philamlife against its own
interest. The burden of evidence has shifted to Philamlife, which must prove that the letter did
not contain Chuang's insurance application.

However, Philamlife failed to do so; thus, Philamlife is deemed to have received


Chuang's insurance application. The seemingly conflicting provisions must be harmonized to
mean that upon a party's purchase of a memorial lot on installment from Eternal, an insurance
contract covering the lot purchaser is created and the same is effective, valid, and binding until
terminated by Philamlife by disapproving the insurance application. The second sentence of
Creditor Group Life Policy No. P-1920 on the Effective Date of Benefit is in the nature of a
resolutory condition which would lead to the cessation of the insurance contract.

Moreover, the mere inaction of the insurer on the insurance application must not work to
prejudice the insured; it cannot be interpreted as a termination of the insurance contract. The
termination of the insurance contract by the insurer must be explicit and unambiguous.

CASE NO. 25. FILIPINAS CIA DE SEGUROS VS. CHRISTERN, 89 PHIL 54 – RAGMA, MYKA
MARIE KHRISTINE

FACTS:

On October 1, 1941, the respondent corporation, Christern Huenefeld and Co., Inc., after
payment of corresponding premium, obtained from the petitioner, Filipinas Cia de Seguros fire
policy covering merchandise contained in a building located at Binondo, Manila. On February
27, 1942 or during the Japanese military occupation, the building and insured merchandise
were burned. In due time, the respondent submitted to the petitioner its claim under the policy.

The petitioner refused to pay the claim on the ground that the policy in favor of the
respondent that ceased to be a force on the date the United States declared war against
Germany, the respondent corporation (through organized under and by virtue of the laws of
Philippines) being controlled by German subjects and the petitioner being a company under
American jurisdiction when said policy was issued on October 1, 1941.

Filipinas Compañia refused to pay, alleging that Christern is a corporation whose


majority stockholders are Germans and that during the Japanese occupation, America declared
war against Germany, hence the insurance policy ceased to be effective because the insured
has become an enemy. Filipinas Compañia was eventually ordered to pay Christern as ordered
by the Japanese government.

ISSUE:

Whether the validity of the insurance contract ceased upon the declaration of war.

HELD:

Page 33 of 99
The respondent having become an enemy corporation on December 10, 1941, the
insurance policy issued in its favor on October 1, 1941, by the petitioner (a Philippine
corporation) had ceased to be valid and enforceable, and since the insured goods were burned
after December 10, 1941, and during the war, the respondent was not entitled to any indemnity
under said policy from the petitioner. However, elementary rules of justice (in the absence of
specific provision in the Insurance Law) require that the premium paid by the respondent for the
period covered by its policy from December 11, 1941, should be returned by the petitioner.

Majority of the stockholders of Christern were German subjects. This being so, SC ruled
that said corporation became an enemy corporation upon the war between the US and
Germany. The Phil Insurance Law in Sec. 8 provides that anyone except a public enemy may
be insured. It stands to reason that an insurance policy ceases to be allowable as soon as an
insured becomes a public enemy.

The purpose of the war is to cripple the power and exhaust the resources of the enemy,
and it is inconsistent that one country should destroy its enemy property and repay in insurance
the value of what has been so destroyed, or that it should in such manner increase the
resources of the enemy or render it aid.

All individuals who compose the belligerent powers, exist as to each other, in a state of
utter exclusion and are public enemies. Christern having become an enemy corporation on
Dec. 10. 1941, the insurance policy issued in his favor on Oct. 1, 1941 by Filipinas had ceased
to be valid and enforceable, and since the insured goods were burned after Dec. 10, 1941, and
during the war, Christern was NOT entitled to any indemnity under said policy from Filipinas.

CASE NO. 26. FRANCISCO DEL VAL, ET AL. VS. ANDRES DEL VAL, G.R. NO. L-9374
FEBRUARY 16, 1915 – GACAYAN, HAZEL FAYE

FACTS:

During the lifetime of Gregorio Nacianceno del Val, he took out an insurance on his life for
the sum of P40,000 and made it payable to Andres Del Val, the defendant, as sole beneficiary.
After the death of Gregorio, the defendant collected the face of the policy. The said policy was
then paid a sum of P18, 365.20 to redeem certain real estate which the decedent had sold to
third persons with a right to repurchase.

Moreover, the redemption of said premises was made by the attorney of the defendant in
the name of the plaintiff, Francisco Del Val, and the defendant as heirs of the deceased vendor.
Furthermore, the redemption of said premises, they have been used and benefited from, and
during that time, the plaintiff paid no taxes and made no repairs.

It further appears from the pleadings that the defendant, on the death of the deceased, took
possession of most of his personal property, which he still has in his possession, and that he
has also the balance on said insurance policy amounting to P21,634.80.law library

Plaintiffs contend that the amount of the insurance policy belonged to the estate of the
deceased and not to the defendant personally; that, therefore, they are entitled to a partition not
only of the real and personal property, but also of the P40,000 life insurance. The complaint
prays a partition of all the property, both real and personal, left by the deceased; that the
defendant account for P21,634.80, and that that sum be divided equally among the plaintiffs and
defendant along with the other property of deceased.

Page 34 of 99
The defendant denies the material allegations of the complaint and sets up as special
defense and counterclaim that the redemption of the real estate sold by his father was made in
the name of the plaintiffs and himself instead of in his name alone without his knowledge or
consent; and that it was not his intention to use the proceeds of the insurance policy for the
benefit of any person but himself, he alleging that he was and is the sole owner thereof and that
it is his individual property. He, therefore, asks that he be declared the owner of the real estate
redeemed by the payment of the P18,365.20, the owner of the remaining P21,634.80, the
balance of the insurance policy, and that the plaintiff's account for the use and occupation of the
premises so redeemed since the date of the redemption.

ISSUE:

Whether or not the proceeds belonged exclusively to the designated son, Andres Del Val,
the defendant, and not the estate of the insured.

HELD:

Yes. When a life insurance policy is made payable to one of the heirs of the person whose
life is insured, the proceeds of the policy or the death of the insured belong exclusively to the
beneficiary and not to the estate of the person whose life was insured; and such proceeds are
his individual property and not the property of the heirs of the person whose life was insured.

Article 1035 of the Civil Code provides that an heir by force of law surviving with others of
the same character to a succession must bring into the hereditary estate the property or
securities he may received from the deceased during the life of the same, by way of dowry, gift
or for any good consideration, in order to compute it in fixing the legal portions and in account of
the division, ―is not applicable to the proceeds of an insurance policy made payable to one of
the heirs of the insured by name, nor can proceeds of such policy be considered a gift under
Article 819 of the Civil Code.

The contract of life insurance policy is a special contract, and the destination of the
proceeds thereof is determined by the special laws which deal exclusively with that subject. The
Civil Code has no provisions which relate directly and specifically to life insurance contracts or
to the destination of life insurance proceeds. That subject is regulated exclusively by the Code
of Commerce, which provides for the terms of the contract, the relations of the parties and the
destination of the proceeds of the policy.

CASE NO. 27. HEIRS OF LORETO C. MARAMAG VS. EVA MARAMAG ET.AL G.R. NO.
181132, JUNE 5, 2009 – PASCUA, MA. BETINA GRACE

Facts:

Loreto Maramag designated as beneficiaries his concubine Eva de Guzman Maramag


together with his illegitimate children (respondents). Hence, petitioner Vicenta Maramag (legal
wife of Loreto) filed a petition against the respondents claiming for his insurance.

Vicenta alleges that Eva is disqualified from claiming any proceeds from his insurance
policies because she is a suspect in the killing of Loreto. Petitioner also asserts that illegitimate
children of Loreto —- Odessa, Karl Brian, and Trisha Angelie, were entitled only to one-half of
the legitime of the legitimate children, thus, the proceeds released to Odessa and those to be
released to Karl Brian and Trisha Angelie were inofficious and should be reduced.

Page 35 of 99
Issue:

Whether illegitimate children can be beneficiaries in an insurance contract.

Ruling:

Yes. Section 53 of the Insurance Code states that the insurance proceeds shall be
applied exclusively to the proper interest of the person in whose name or for whose benefit it is
made unless otherwise specified in the policy.

Pursuant thereto, it is obvious that the only persons entitled to claim the insurance
proceeds are either the insured, if still alive; or the beneficiary, if the insured is already
deceased, upon the maturation of the policy. The exception to this rule is a situation where the
insurance contract was intended to benefit third persons who are not parties to the same in the
form of favorable stipulations or indemnity. In such a case, third parties may directly sue and
claim from the insurer.

CASE NO. 28. PACIFIC TIMBER EXPORT CORPORATION V. THE HONORABLE COURT
OF APPEALS AND WORKMEN'S INSURANCE COMPANY, INC., G.R. NO. L-38613,
FEBRUARY 25, 1982 – LAUSAN, PENNELOPE

FACTS:

The plaintiff, Pacific Timber Export Corporation (Pacific), secured temporary insurance
from the defendant, Workmen's Insurance Company, Inc. (Workmen‘s), for its exportation of
1,250,000 board feet owned by Philippine Lauan and Apitong logs to be shipped from Quezon
Province to Okinawa and Tokyo, Japan. Workmen‘s issued a cover note insuring the cargo of
the Pacific subject to its terms and conditions. The two marine policies bore the numbers
53HO1032 and 53HO1033.

After the issuance of the cover note, but before the issuance of the two marine policies,
some of the logs intended to be exported were lost during loading operations in the Diapitan
Bay. While the logs were alongside the vessel, bad weather developed resulting in some logs
rafted together break loose from each other. Few were salvaged, but a number of pieces were
verified to have been lost or washed away as a result of the accident.

Pacific Timber informed Workmen‘s about the loss of pieces of logs during loading of SS
woodlock. Although dated April 4, 1963, the letter was received in the office of the Workmen‘s
only on April 15, 1963. The plaintiff claimed for insurance.

ISSUES:

1. Whether or not the cover not was without consideration, thus null and void.
2. Whether the Insurance Company was absolved from responsibility due to unreasonable
delay in giving notice of loss.

HELD:

Page 36 of 99
1. SC upheld Pacific‘s contention that said cover not was with consideration. The fact that
no separate premium was paid on the cover note before the loss was insured against
occurred does not militate against the validity of Pacific‘s contention, for no such
premium could have been paid, since by the nature of the cover note, it did not contain,
as all cover notes do not contain, particulars of the shipment that would serve as basis
for the computation of the premiums. As a logical consequence, no separate premiums
are required to be paid on a cover note.

If the note is to be treated as a separate policy instead of integrating it to the regular


policies subsequently issued, its purpose would be meaningless for it is in a real sense a
contract, not a mere application.

2. NO, the defense of delay can‘t be sustained. The facts show that instead of invoking the
ground of delay in objecting to petitioner's claim of recovery on the cover note, the
insurer never had this in its mind. It has a duty to inquire when the loss took place, so
that it could determine whether delay would be a valid ground of objection. There was
enough time for insurer to determine if petitioner was guilty of delay in communicating
the loss to respondent company. It never did in the Insurance Commission. Waiver can
be raised against it under Section 84 of the Insurance Act.

CASE NO. 29. VIOLETA R. LALICAN VS.THE INSULAR LIFE ASSURANCE COMPANY
LIMITED, AS REPRESENTED BY THE PRESIDENT VICENTE R. AVILON, G.R. NO. 183526,
AUGUST 25, 2009 – ANTONIO, PHOEBE ANTONETTE

FACTS:

Eulogio Lalican applied for an insurance policy with the Insular Life amounting to
P1,500,000. Under the terms of the policy, Eulogio was to pay the premiums on a quarterly
basis, having a grace period of 31 days, for the payment of each premium subsequent to the
first. If any premium was not paid on or before the due date, the policy would be in default and if
the premium remained unpaid until the end of the grace period, the policy would automatically
lapse and become void. Eulogio paid the premiums due on the first two succeeding payment
dates but failed to pay subsequent premiums even after the lapse of the grace period thereby
rendering the policy void. He submitted an application for reinstatement of policy through
Josephine Malaluan, an agent of Insular Life, together with the payment of the unpaid
premiums. However, the Insular Life notified him that his application could not be processed
because he failed to pay the overdue interest of the unpaid premiums.

On September 17, 1998, Eulogio submitted to Malaluan‘s house a second application for
reinstatement including the payment for the overdue interest as well as for the premiums due for
April and July of that year, which was received by Malaluan‘s husband on her behalf and was
thereby issued a receipt for the amount Eulogio deposited. However, on that same day, Eulogio
died of cardio-respiratory arrest secondary to electrocution. Violeta, Eulogio‘s widow filed with
the Insular Life a claim for payment of the full proceeds of the policy but the latter informed her
that the claim could not be granted since at the time of Eulogio‘s death, his policy has already
lapsed and he failed to reinstate the same. Violeta requested a reconsideration of her claim but
the same was also rejected.

Page 37 of 99
Therefore, she filed a complaint for death claim benefits with the RTC alleging the unfair
claim settlement practice of Insular Life and its deliberate failure to act with reasonable
promptness on her insurance claim. The trial court rendered a decision in favor of Insular Life
and after the former denied her motion for reconsideration, she directly elevated her case to the
Supreme Court via the petition for review on Certiorari.

ISSUE:

Whether Eulogio was able to reinstate the lapsed insurance policy on his life before his
death.

RULING:

The insurance policy had already lapsed is a fact beyond dispute.

To reinstate a policy means to restore the same to premium-paying status after it has
been permitted to lapse. Both the policy contract and application for reinstatement provide for
specific conditions for the reinstatement of a lapsed policy. According to the Application for
Reinstatement, the policy would only be considered reinstated upon the approval of the
application by Insular Life during the applicant‘s lifetime and good health and whatever amount
the application paid in connection was considered to be a deposit only until approval of said
application.

Eulogio‘s death rendered impossible full compliance with the conditions for
reinstatement of policy even though, before his death, he managed to file his application for
reinstatement and deposit the amount for payment of his overdue premiums and interest
thereon with Malaluan. As expressly provided on the policy contract, agents of Insular Life have
no authority to approve any application for reinstatement. They still had to turn over to Insular
Life the application for reinstatement and accompanying deposit, for processing and approval of
the latter.

CASE NO. 30. MALAYAN INSURANCE CO. V. PHILIPPINES FIRST INSURANCE CO., GR
NO. 184300, 2012-07-11 – CABALO, CASHMIR

FACTS:

Since 1989, Wyeth Philippines, Inc. (Wyeth) and respondent Reputable Forwarder Services,
Inc. (Reputable) had been annually executing a contract of carriage, whereby the latter
undertook to transport and deliver the former‘s products to its customers, dealers or salesmen.

On November 18, 1993, Wyeth procured Marine Policy No. MAR 13797 (Marine Policy) from
respondent Philippines First Insurance Co., Inc. (Philippines First) to secure its interest over its
own products. Philippines First then insured Wyeth‘s nutritional, pharmaceutical and other
products usual or incidental to the insured‘s business while the same were being transported or
shipped in the Philippines. The policy covers all risks of direct physical loss or damage from any
external cause, if by land, and provides a limit of P6, 000,000.00 per any one land vehicle.

On December 1, 1993, Wyeth executed its annual contract of carriage with Reputable was later
on discovered that was not signed by Wyeth‘s representative/s However, it was admittedly
signed by Reputable representatives, the terms thereof faithfully observed by the parties and, as
previously stated, the same contract of carriage had been annually executed by the parties

Page 38 of 99
every year since 1989. Under the contract, Reputable undertook to answer for ―all risks with
respect to the goods and shall be liable to the Company -Wyeth

Furthermore, the contract also required Reputable to secure an insurance policy on Wyeth‘s
goods. Thus, on February 11, 1994, Reputable signed a Special Risk Insurance Policy (SR
Policy) with Malayan (petitioner) for the amount of P1, 000,000.00.

On October 6, 1994, during the effectivity of the Marine Policy and SR Policy, Reputable
received from Wyeth 1,000 boxes of Promil infant formul amounting to P2, 357,582.70, to be
delivered by Reputable to Mercury Drug Corporation in Quezon City. Unfortunately, on the
same date, the truck carrying Wyeth‘s products was hijacked by about 10 armed men. Malayan
questions its liability based on sections 5 and 12 of the SR Policy.

ISSUE: Whether or not there is double insurance (with respect to Section 5 or Section 12 of the
SR Policy).

HELD:

No. Under Section 93 of the Insurance Code, double insurance exists where the same person is
insured by several insurers separately in respect to the same subject and interest. The
requisites in order for double insurance to arise are as follows:

The person insured is the same;

Two or more insurers insuring separately;

There is identity of subject matter;

There is identity of interest insured; and

There is identity of the risk or peril insured against.

In the case at bar, while it is true that the Marine Policy and the SR Policy were both issued over
the same subject matter, i.e. goods belonging to Wyeth, and both covered the same peril
insured against, it is, however, beyond cavil that the said policies were issued to two different
persons or entities. It is undisputed that Wyeth is the recognized insured of Philippines First
under its Marine Policy, while Reputable is the recognized insured of Malayan under the SR
Policy. The fact that Reputable procured Malayan‘s SR Policy over the goods of Wyeth pursuant
merely to the stipulated requirement under its contract of carriage with the latter does not make
Reputable a mere agent of Wyeth in obtaining the said SR Policy.

The interest of Wyeth over the property subject matter of both insurance contracts is also
different and distinct from that of Reputable. The policy issued by Philippines First was in
consideration of the legal and/or equitable interest of Wyeth over its own goods. On the other
hand, what was issued by Malayan to Reputable was over the latter‘s insurable interest over the
safety of the goods, which may become the basis of the latter‘s liability in case of loss or
damage to the property and falls within the contemplation of Section 15 of the Insurance Code.

Therefore, even though the two concerned insurance policies were issued over the same goods
and cover the same risk, there arises no double insurance since they were issued to two
different persons/entities having distinct insurable interests. Necessarily, over insurance by
double insurance CANNOT likewise exist. Hence, as correctly ruled by the RTC and CA, neither
Section 5 nor Section 12 of the SR Policy can be applied.

Page 39 of 99
CASE NO. 31. LOYOLA LIFE PLANS INCORPORATED (NOW LOYOLA PLANS
CONSOLIDATED INC.) AND ANGELITA D. LUMIQUED, VS. ATR PROFESSIONAL LIFE
ASSURANCE CORPORATION (NOW ASIAN LIFE AND GENERAL ASSURANCE
CORPORATION), G.R. NO. 228402, AUGUST 26, 2020 – GATTOC, SHEKINAH

FACTS:

Loyola Life Plans, Inc. (Loyola) is a pre-need company engaged in the business of
insuring the lives of its plan holders through its Timeplans (pension contracts) and Lifeplans
(memorial service contracts), which are covered by insurance benefits provided by several
insurance companies including the respondent. Dwight Lumiqed, husband of Angelita,
purchased a Timeplan from Loyola payable in 120 monthly installments. He, then, issued two
metrobank checks to pay the first monthly premium, and subsequently paid in cash, he was
then issued an official receipt. Belen Ganit , Loyola's Sales Operation Assistant, deposited on
the same day the two Metrobank checks while the cash payment was deposited to the account
of Loyola on May 2, 2000. But on May 1, 2000, Dwight died due to multiple stab wounds.
Thereafter, Angelita filed a claim to recover the proceeds of the insurance. However,
ATR denied the claim on the ground that the initial installment payment was not completed.
Loyola asked for a reconsideration, insisting that the Timeplan Dwight obtained was already in
full force and effect upon payment of the premium.
ATR also invalidated Dwight's application as his signature appearing therein was
allegedly forged. To bar Angelita from further pursuing any claim for the insurance benefits, ATR
instituted a complaint to declare the individual insurance coverage of Dwight void and of no
effect at the time of his death. ATR also prayed for the payment of attorney's fees, litigation
expenses, and costs of suit.
The RTC held that Dwight timely paid the premium of the policy. Since the agreement
and the official receipt state that the insurance coverage of a planholder shall take effect on the
date of initial payment and/or down payment on the Timeplan

ISSUE:
1. Whether an insurance contract was perfected between Dwight and ATR
2. Whether Dwight's Timeplan contract is entitled to the Group Creditors Life Insurance and
the Group Yearly Renewable Term Life benefits obtained by Loyola

RULING:
1. Yes. Dwight timely paid the initial monthly premium for the Timeplan on April 28, 2000 to
Loyola who is an agent of ATR. Hence, an insurance contract was perfected. A contract
of insurance is defined as an agreement whereby one undertakes for a consideration to
indemnify another against loss, damage, or liability arising from an unknown or
contingent event.
The fact that Dwight was only able to make an initial payment of the insurance premium
and that Loyola failed to immediately remit the cash portion of the initial payment to ATR
should not affect the validity of the perfected insurance contract. Furthermore, ATR
agreed to insure all present and future plan holders of Loyola.
It is important to clarify that Loyola is an agent of ATR. In a contract of agency, "a person
binds himself to render some service or to do something in representation or on behalf of
another, with the consent or authority of the latter." Therefore, a planholder's payment
made to Loyola has the same legal effect as payment made to ATR, even if Loyola failed
to immediately deposit the cash payment to its account.

Page 40 of 99
The contract between ATR and Loyola is a contract of adhesion as it was
prepared solely by ATR for Loyola and its planholders to conform to. Any ambiguity in a
contract of adhesion is construed strictly against the party that prepared it. In this case,
the obscure provision pertaining to the date of effectivity of the policy coverage should
be resolved in favor of Angelita.
Thus, the happening of any of the instances enumerated should suffice in giving
rise to the effectivity of the individual insurance. This interpretation is more consistent
with the other provisions of the policy such as the clause on the "EFFECTIVE DATE" of
the policy.

2. Yes. Dwight‘s Time plan contract entitles him to the benefits obtained by Loyola. The CA
committed serious error in deleting the award of actual damages comprising the
insurance benefits from the Group Creditors Life Insurance. The evidence on record and
the pleadings submitted by ATR all show that Loyola obtained a Group Creditors Life
Insurance from ATR, with supplementary Group Yearly Renewable Term Life and
Accidental Death benefits, for its present and future planholders. ATR never denied the
inclusion of Dwight's Timeplan in Master Policy No. GCL-878. Thus, the RTC was
correct in including the proceeds from those benefits in computing the award of actual
damages in favor of Angelita.

CASE NO. 32. PHILAM INSURANCE CO., INC., NOW CHARTIS PHILIPPINES INSURANCE,
INC., PETITIONER, VS. PARC CHATEAU CONDOMINIUM UNIT OWNERS ASSOCIATION,
INC., AND/OR EDUARDO B. COLET, RESPONDENTS., G.R. NO. 201116, MARCH 04, 2019
– ABRIGO, KAREN

FACTS:

Philam Insurance Co., Inc. (Philam) [now Chartis Philippines Insurance, Inc.] was
selected to provide the insurance requirements for Parc Chateau Condominium Unit Owners
Association, Inc. (Parc Association) as approved by the Associaton‘s board of directors. Upon
appraisal, Philam issued Fire and Lightning Insurance Policy No. 0601502995 for P900 million
and Comprehensive General Liability Insurance Policy No. 0301003155 for P1 Million, for the
period November 30, 2003 to November 30, 2004. The parties negotiated for a 90-day payment
term of the insurance premium, of which payment term was embodied in a Jumbo Risk
Provision, which further provided that the premium installment payments were due on
November 30, 2003, December 30, 2003, and January 30, 2004 (3 monthly installments) and
failure to pay any of the payments, the insurance contract shall be deemed cancelled.

Parc Association's board of directors found the terms unacceptable and did not pursue
the transaction. Since no premiums were paid, Philam made oral and written demands upon
Parc Association, who refused to do so alleging that the insurance agent had been informed of
its decision not to take up the insurance coverage. Philam sent demand letters with statement of
account claiming P363,215.21 unpaid premium based on Short Scale Rate Period and likewise
cancelled the policies. On June 3, 2005, Philam filed a complaint against Parc Association and
Colet for recovery of P363,215.21 unpaid premium, plus attorney's fees and costs of suit in the
Metropolitan Trial Court (MeTC) of Makati, Branch 65.

MeTC Ruling: Case Dismissed. The MeTC determined that since Philam admitted that
Parc Association did not pay its premium, one of the elements of an insurance contract was
lacking, that is, the insured must pay a premium. Philam appealed to the Regional Trial Court
(RTC) of Makati, Branch 137, which partly affirmed the MeTC decision which…

Page 41 of 99
RTC Ruling: Partly affirmed MeTC decision. The RTC pronounced that there was no
valid insurance contract between the parties because of non-payment of premium, and there
was no express waiver of full payment of premiums. Philam moved for reconsideration, which
the RTC denied. Philam elevated the case before the Court of Appeals (CA) through a petition
for review under Rule 42 of the Rules of Court.

CA Ruling: CA denied Philam's petition and affirmed the RTC Decision. The CA
discussed that based on Section 77 of Presidential Decree 612 or the Insurance Code of the
Philippines, the general rule is that no insurance contract issued by an insurance company is
valid and binding unless and until the premium has been paid. Although there are exceptions
laid down in UCPB General Insurance Co., Inc. v. Masagana Telamart, Inc.,the CA determined
that none of these exceptions were applicable to the case. The third exception, according to the
CA held in Makati Tuscany Condominium Corporation v. Court of Appeals, wherein the Court
ruled that the general rule in Section 77 may not apply if the parties agreed to the payment of
premium in installment and partial payment has been made at the time of loss. Here, the parties
agreed to a payment by installment, but no actual payment was made. Thus, the third exception
has no application in this case. Philam moved for reconsideration. CA denied. Philam filed a
Petition for Review on Certiorari under Rule 45 of the Rules of Court.

ISSUE:

Whether there was a valid insurance contract between the parties.

RULING:

Petition denied. There was no valid insurance contract between the parties. Rule 45 of
the Rules of Court, as amended, states that only questions of law shall be raised in a petition for
review on certiorari. While the rule has exceptions, they are irrelevant in this case, as Philam did
not properly plead and substantiate the applicability of the exceptions. Thus, the Court applies
the general rule.

In resolving whether the CA was correct in affirming the RTC decision, the Court
considered the following simplified alleged errors as presented by Philam:

1. Whether or not respondents‘ request for terms of payment of premium after the
policies were issued and the grant of said request by petitioner constitute the parties‘ intention
to be bound by the insurance contract;

2. Whether or not the fourth exception provided for under Section 77 of the Insurance
Code of the Philippines applies in the instant case; and

3. Whether or not the negotiations which the parties had were with respect to the terms
of payment of premium already agreed upon by the parties and not on the lowering of the
amount of premium as to negate the existence of a perfected contract of insurance. The first
and third alleged errors refer to the request for the terms of payment. Does Parc Association‘s
request and Philam‘s subsequent grant of the request constitute their intention to be bound by
the insurance contract? Does the negotiation refer to the terms of payment or to the lowering of
the premium? In arriving at the answers to the questions, the Court has to determine the
intention of the parties. In doing so, the Court has to read the transcript of stenographic notes of
the witnesses, and review the language or tenor of some of the documentary evidence, such as:
Philam‘s proposal on October 7, 2003, Colet‘s acceptance letter dated November 24, 2003, the
Jumbo Risk Provision, and the written communications between Philam and Parc Association.
In short, the Court has to re-evaluate the evidence on record. Evaluation of evidence is an

Page 42 of 99
indication that the question or issue posed before the Court is a question of fact or a factual
issue.

In Century Iron Works, Inc. v. Biñasthe Court differentiated between question of


law and question of fact. A question of law arises when there is doubt as to what the law is on
a certain state of facts, while there is a question of fact when the doubt arises as to the truth or
falsity of the alleged facts. For a question to be one of law, the question must not involve an
examination of the probative value of the evidence presented by the litigants or any of them.
The resolution of the issue must rest solely on what the law provides on the given set of
circumstances. Once it is clear that the issue invites a review of the evidence presented, the
question posed is one of fact.

Thus, the test of whether a question is one of law or of fact is not the appellation given to
such question by the party raising the same; rather, it is whether the appellate court can
determine the issue raised without reviewing or evaluating the evidence, in which case, it is a
question of law; otherwise it is a question of fact. Applying the test to this case, it is without a
doubt that the questions/issues presented before the Court are factual in nature, which are not
proper subjects of a petition for review on certiorari under Rule 45 of the Rules of Court, as
amended. It has been repeatedly pronounced that the Court is not a trier of facts. Evaluation of
evidence is the function of the trial court.

As for the second alleged error, Philam avers that this case falls under the fourth
exception as explained in the Makati Tuscany case. The Makati Tuscany case provides that if
the insurer has granted the insured a credit term for the payment of the premium, it is an
exception to the general rule that premium must first be paid before the effectivity of an
insurance contract. Philam argues that the 90-day payment term is a credit extension and
should be considered as an exception to the general rule.

However, the CA correctly determined that the Jumbo Risk Provision clearly indicates
that failure to pay in full any of the scheduled installments on or before the due date shall render
the insurance policy void and ineffective as of 4 p.m. of such date. Parc Association‘s failure to
pay on the first due date (November 30, 2003), resulted in a void and ineffective policy as of 4
p.m. of November 30, 2003. Hence, there is no credit extension to consider as the Jumbo Risk
Provision itself expressly cuts off the inception of the insurance policy in case of default. The
Court resolves to deny the petition after finding that the CA did not commit any reversible error
in the assailed decision and resolution. The CA had exhaustively explained the law and
jurisprudence, which are the bases of its decision and resolution. Both trial courts and the
appellate court are consistent in its findings of fact that there is no perfected insurance contract,
because of the absence of one of the elements, that is, payment of premium. As a
consequence, Philam cannot collect P363,215.21 unpaid premiums of void insurance policies

CASE NO. 33. PEDRO ARCE V. THE CAPITAL INSURANCE & SURETY CO., INC. G.R. NO.
L-28501, SEPTEMBER 30, 1982 – MANGAOANG, NICOLE JOYCE

FACTS:

Pedro Arce owned a house in Tondo, Manila, which had been insured with the Capital
Insurance Co., Inc. The COMPANY sent to the INSURED Renewal Certificate No. 47302 to
cover the period December 5, 1965 to December 5, 1966. The COMPANY also requested
payment of the corresponding premium.

Page 43 of 99
The premium was not paid on January 4, 1966. On January 8, 1966, the house of the
INSURED was totally destroyed by fire.

INSURED's wife presented a claim for indemnity to the COMPANY. She was told that no
indemnity was due because the premium on the policy was not paid. Nonetheless the
COMPANY tendered a check for P300.00 as financial aid. The COMPANY reiterated that the
check was given "not as an obligation, but as a concession" because the renewal premium had
not been paid, The INSURED cashed the check but then sued the COMPANY on the policy.

ISSUE:

Whether Capital Insurance and Surety Co., Inc., should pay the proceeds of a fire
insurance policy to Pedro Arce.

RULING: NO.

Sec. 72 of the Insurance Act, as amended by R.A. No. 3540 reads:

SEC. 72. An insurer is entitled to payment of premium as soon as the thing


insured is exposed to the perils insured against, unless there is clear agreement
to grant credit extension for the premium due. No policy issued by an insurance
company is valid and binding unless and until the premium thereof has been paid
" (Italics supplied.) (p. 11, Appellant's Brief.)

It is obvious from both the Insurance Act, as amended, and the stipulation of the parties that
time is of the essence in respect of the payment of the insurance premium so that if it is not paid
the contract does not take effect unless there is still another stipulation to the contrary. In the
instant case, the INSURED was given a grace period to pay the premium but the period having
expired with no payment made, he cannot insist that the COMPANY is nonetheless obligated to
him.

It is to be noted that Delgado was decided in the light of the Insurance Act before Sec. 72 was
amended by the addition of the underscored portion, supra, Prior to the amendment, an
insurance contract was effective even if the premium had not been paid so that an insurer was
obligated to pay indemnity in case of loss and correlatively he had also the right to sue for
payment of the premium. But the amendment to Sec. 72 has radically changed the legal regime
in that unless the premium is paid there is no insurance.

CASE NO. 34. UCPB GENERAL INSURANCE CO., INC. V. MASAGANA TELAMART, INC.,
G.R. NO. 137172, APRIL 4, 2001 – DELA CRUZ, JEREMY KIMBERLY

FACTS:
On April 15, 1991, petitioner issued five (5) insurance policies covering respondent‘s
various property described therein against fire, for the period from May 22, 1991 to May 22,
1992.

In March 1992, petitioner evaluated the policies and decided not to renew them upon
expiration of their terms on May 22, 1992. Petitioner advised respondent‘s broker, Zuellig
Insurance Brokers, Inc. of its intention not to renew the policies.

Page 44 of 99
On April 6, 1992, petitioner gave written notice to respondent of the non-renewal of the
policies at the address stated in the policies.

On June 13, 1992, fire razed respondent‘s property covered by three of the insurance
policies petitioner issued.

On July 13, 1992, respondent presented to petitioner‘s cashier at its head office five (5)
manager‘s checks in the total amount of P225,753.95, representing premium for the renewal of
the policies from May 22, 1992 to May 22, 1993. No notice of loss was filed by respondent
under the policies prior to July 14, 1992.

On July 14, 1992, respondent filed with petitioner its formal claim for indemnification of
the insured property razed by fire. On the same day, petitioner returned to respondent the five
(5) manager‘s checks that it tendered, and at the same time rejected respondent‘s claim for the
reasons (a) that the policies had expired and were not renewed, and (b) that the fire occurred on
June 13, 1992, before respondent‘s tender of premium payment.

On July 21, 1992, respondent filed with the Regional Trial Court, Branch 58, Makati City,
a civil complaint against petitioner for recovery of P18,645,000.00, representing the face value
of the policies covering respondent‘s insured property razed by fire, and for attorney‘s fees.

On October 23, 1992, after its motion to dismiss had been denied, petitioner filed an
answer to the complaint. It alleged that the complaint ―fails to state a cause of action‖; that
petitioner was not liable to respondent for insurance proceeds under the policies because at the
time of the loss of respondent‘s property due to fire, the policies had long expired and were not
renewed.

ISSUE:

Whether the fire insurance policies issued by petitioner to the respondent covering the
period May 22, 1991 to May 22, 1992, had expired on the latter date or had been extended or
renewed by an implied credit arrangement though actual payment of premium was tendered on
a latter date after the occurrence of the risk (fire) insured against.

HELD:

Yes. Upon a meticulous review of the records and reevaluation of the issues raised in
the motion for reconsideration and the pleadings filed thereafter by the parties, we resolved to
grant the motion for reconsideration.

The instant case has to rise or fall on the core issue of whether Section 77 of the
Insurance Code of 1978 (P.D. No. 1460) must be strictly applied to Petitioner‘s advantage
despite its practice of granting a 60- to 90-day credit term for the payment of premiums.

Page 45 of 99
Section 77 of the Insurance Code of 1978 provides:

SECTION 77. An insurer is entitled to payment of the premium as soon as the thing
insured is exposed to the peril insured against. Notwithstanding any agreement to the contrary,
no policy or contract of insurance issued by an insurance company is valid and binding unless
and until the premium thereof has been paid, except in the case of a life or an industrial life
policy whenever the grace period provision applies.

The first exception is provided by Section 77 itself, and that is, in case of a life or
industrial life policy whenever the grace period provision applies.

The second is that covered by Section 78 of the Insurance Code, which provides:

SECTION 78. Any acknowledgment in a policy or contract of insurance of the receipt of


premium is conclusive evidence of its payment, so far as to make the policy binding,
notwithstanding any stipulation therein that it shall not be binding until premium is actually paid.

A third exception was laid down in Makati Tuscany Condominium Corporation vs. Court
of Appeals, wherein we ruled that Section 77 may not apply if the parties have agreed to the
payment in installments of the premium and partial payment has been made at the time of loss.

By the approval of the aforequoted findings and conclusion of the Court of Appeals,
Tuscany has provided a fourth exception to Section 77, namely, that the insurer may grant
credit extension for the payment of the premium. This simply means that if the insurer has
granted the insured a credit term for the payment of the premium and loss occurs before the
expiration of the term, recovery on the policy should be allowed even though the premium is
paid after the loss but within the credit term.

Moreover, there is nothing in Section 77 which prohibits the parties in an insurance


contract to provide a credit term within which to pay the premiums. That agreement is not
against the law, morals, good customs, public order or public policy. The agreement binds the
parties. Article 1306 of the Civil Code provides:

ARTICLE 1306. The contracting parties may establish such stipulations clauses, terms
and conditions as they may deem convenient, provided they are not contrary to law, morals,
good customs, public order, or public policy.

Finally in the instant case, it would be unjust and inequitable if recovery on the policy
would not be permitted against Petitioner, which had consistently granted a 60- to 90-day credit
term for the payment of premiums despite its full awareness of Section 77. Estoppel bars it from
taking refuge under said Section, since Respondent relied in good faith on such practice.
Estoppel then is the fifth exception to Section 77.

Page 46 of 99
CASE NO. 35. JAIME T. GAISANO V. DEVELOPMENT INSURANCE AND SURETY
CORPORATION, G.R. NO. 190702, FEBRUARY 27, 2017 – CHAWI, DECIMAY

FACTS:

Gaisano entered into a contract of insurance with respondent Development insurance


forits 3 company vehicles, which covers the period fromSeptember 27, 1996 to September 26,
1997. OnSeptember 27, 1996, petitioner issued a check for the payment of the premiums, but
Trans-Pacific, the agent of respondent, was not able to collect the check. On the same night,
one of the vehicles got stolen. Gaisano filed a claim, but was rejected on the ground of non-
payment of premiums.

ISSUE: whether there is a binding insurance contract between petitioner and respondent

HELD:

Insurance is a contract whereby one undertakes for a consideration to indemnify another


against loss, damage or liability arising from an unknown or contingent event. Just like any other
contract, it requires a cause or consideration. The consideration is the premium, which must be
paid at the time and in the way and manner specified in the policy. If not so paid, the policy will
lapse and be forfeited by its own terms.

The law, however, limits the parties' autonomy as to when payment of premium may be
made for the contract to take effect. The general rule in insurance laws is that unless the
premium is paid, the insurance policy is not valid and binding.

There are, of course, exceptions to the rule that no insurance contract takes effect
unless premium is paid:

(1) in case of life or industrial life policy, whenever the grace period provision applies, as
expressly provided by Section 77 itself;

(2) where the insurer acknowledged in the policy or contract of insurance itself the
receipt of premium, even if premium has not been actually paid, as expressly provided
by Section 78 itself;

(3) where the parties agreed that premium payment shall be in installments and partial
payment has been made at the time of loss, as held in Makati Tuscany Condominium
Corp. v. Court of Appeals;

(4) where the insurer granted the insured a credit term for the payment of the premium,
and loss occurs before the expiration of the term, as held in Makati Tuscany
Condominium Corp.; and

(5) where the insurer is in estoppel as when it has consistently granted a 60 to 90-day
credit term for the payment of premiums.

Here, there is no dispute that the check was delivered to and was accepted by
respondent's agent, Trans-Pacific, only on September 28, 1996. No payment of premium had
thus been made at the time of the loss of the vehicle on September 27, 1996. While petitioner
claims that Trans-Pacific was informed that the check was ready for pick-up on September 27,

Page 47 of 99
1996, the notice of the availability of the check, by itself, does not produce the effect of payment
of the premium. Trans-Pacific could not be considered in delay in accepting the check because
when it informed petitioner that it will only be able to pick-up the check the next day, petitioner
did not protest to this, but instead allowed Trans-Pacific to do so. Thus, at the time of loss, there
was no payment of premium yet to make the insurance policy effective.

Even if there is a waiver of pre-payment of premiums, that in itself does not become an
exception to Section 77, unless the insured clearly gave a credit term or extension.

Thus, we find that petitioner is not entitled to the insurance proceeds because no
insurance policy became effective for lack of premium payment.

CASE NO. 36. GOVERNMENT SERVICE INSURANCE SYSTEM VS. PRUDENTIAL


GUARANTEE AND ASSURANCE, INC, G.R. NO. 176982, NOVEMBER 20, 2013 –
MENDOZA, JOYCE ALLYSA

FACTS:
PGAI filed a complaint for collection of a sum of money against GSIS when GSIS failed
to settle the 4th and last installment on the insurance premium with PGAI. In its Answer, GSIS
did not specifically deny the allegations of PGAI. PGAI filed a Motion for Judgment on the
Pleadings averring that GSIS essentially admitted the material allegations of the complaint, such
as the: existence of the MOA between NEA and GSIS; existence of the reinsurance binder
between GSIS and reinsurance binder between GSIS and PGAI; remittance by GSIS to PGAI of
the first three quarterly reinsurance premiums; and failure/refusal of GSIS to remit the fourth a
emit the fourth and last reinsurance premium. RTC granted PGAI‘s Motion for Judgment on the
Pleadings. It observed that the admissions of GSIS that it paid the first 3 quarterly reinsurance
premiums to PGAI affirmed the validity of the contract of reinsurance between them. As such,
GSIS cannot now renege on its obligation to remit the last and remaining quarterly reinsurance
premiums. PGAI subsequently filed a PGAI subsequently filed a Motion for Execution Pending
Appeal which the RTC granted. CA affirmed the RTC‘s ruling. GSIS elevated it to the SC.
ISSUE:
Whether or not the CA erred in ruling that only the Social Insurance Fund and not the
General Fund of the GSIS is exempt from a garnishment
HELD:
No. The declared policy of the State in Section 39 of the GSIS Charter granting GSIS an
exemption from tax, lien, attachment, levy, execution, and other legal processes should be read
together with the grant of power to the GSIS to invest its "excess funds" under Section 36 of the
same Act. Under Section 36, the GSIS is granted the ancillary power to invest in business and
other ventures for the benefit of the employees, by using its excess funds for investment
purposes. In the exercise of such function and power, the GSIS is allowed to assume a
character similar to a private corporation. Thus, it may sue and be sued, as also explicitly
granted by its charter.
Needless to say, where proper, under Section 36, the GSIS may be held liable for the
contracts it has entered into in the course of its business investments. For GSIS cannot claim a
special immunity from liability in regard to its business ventures under said Section.

Page 48 of 99
Nor can it deny contracting parties, in our view, the right of redress and the enforcement
of a claim, particularly as it arises from a purely contractual relationship of a private character
between an individual and the GSIS. Thus, the petition in G.R. No. 165585 is partly granted.
In this case, records disclose that in its Answer, GSIS admitted the material allegations
of PGAI‘s complaint warranting the grant of the relief prayed for. In particular, GSIS admitted
that: (a) it made a request for reinsurance cover which PGAI accepted in a reinsurance binder
effective for one year; (b) it remitted only the first three reinsurance premium payments to
PGAI; (c) it failed to pay PGAI the fourth and final reinsurance premium installment;96 and (d) it
received demand letters from PGAI.97 It also did not refute the allegation of PGAI that it settled
reinsurance claims during the reinsured period. On the basis of these admissions, the Court
finds that the CA did not err in affirming the propriety of a judgment on the pleadings.
The Court hold that the subject policies are valid even if the premiums were paid on
installments. The records clearly show that petitioner and private respondent intended subject
insurance policies to be binding and effective notwithstanding the staggered payment of the
premiums. The initial insurance contract entered into in 1982 was renewed in 1983, then in
1984. In those three (3) years, the insurer accepted all the installment payments. Such
acceptance of payments speaks loudly of the insurer‘s intention to honor the policies it issued to
petitioner. Certainly, basic principles of equity and fairness would not allow the insurer to
continue collecting and accepting the premiums, although paid on installments, and later deny
liability on the lame excuse that the premiums were not prepaid in full.
Moreover, as correctly observed by the appellate court, where the risk is entire and the
contract is indivisible, the insured is not entitled to a refund of the premiums paid if the insurer
was exposed to the risk insured for any period, however brief or momentary.
CASE NO. 37. PHILIPPINE PRYCE ASSURANCE CORPORATION V. COURT OF APPEALS,
ET AL, 230 SCRA 164 / G.R. NO. 107062, FEBRUARY 21, 1994 – SOMERA, MARIA MONICA

FACTS:

Gegroco, Inc. filed a collection suit against the petitioner, Interworld Assurance
Corporation (now Philippine Pryce Assurance Corporation). The complaint alleged that Phil
Pryce issued two surety bonds in behalf of its principal Sagum General Merchandise for 500k
and 1M, respectively.

Phil Pryce admitted having executed the said bonds, but denied liability because
allegedly 1) the checks which were to pay for the premiums bounced and were dishonored
hence there is no contract to speak of between petitioner and its supposed principal; and 2) that
the bonds were merely to guarantee payment of its principal's obligation, thus, excussion is
necessary.

Phil Pryce filed a "Motion with Leave to Admit Third-Party Complaint" with the Third-
Party Complaint attached. When the case was called for pre-trial conference on February 1,
1989, petitioner was again not presented by its officer or its counsel, despite being duly notified.
Hence, upon motion of respondent, petitioner was considered as in default and respondent was
allowed to present evidence ex-parte.

RTC ruled in favor of Gegroco Inc. and CA affirmed RTC. Hence, this appeal.

Page 49 of 99
ISSUE: Whether Phil Pryce should be liable for the surety bond that it issued as payment for the
premium.

RULING:

Yes. Phil Pryce did not and never attempted to pay the requisite docket fee and was not
present during the scheduled pre-trial so it is as if third-party complaint was never filed.

Sec. 177. The surety is entitled to payment of the premium as soon as the contract of suretyship
or bond is perfected and delivered to the obligor. No contract of suretyship or bonding shall be
valid and binding unless and until the premium therefor has been paid, except where the obligee
has accepted the bond, in which case the bond becomes valid and enforceable irrespective of
whether or not the premium has been paid by the obligor to the surety. (Insurance Code)

The above provision outrightly negates petitioner‘s first defense. In a desperate attempt
to escape liability, petitioner further asserts that the above provision is not applicable because
the respondent allegedly had not accepted the surety bond, hence could not have delivered the
goods to Sagum Enterprises. In the first place, petitioner, in its answer, admitted to have issued
the bonds subject matter of the original action. Secondly, the testimony of Mr. Leonardo T.
Guzman, witness for the respondent, reveals that 2 surety bonds where submitted by Sagum
General Merchandise. Likewise attached to the record are exhibits consisting of delivery
invoices addressed to Sagum General Merchandise proving that parts were purchased,
delivered and received.

On the other hand, Phil Pryce‘s defense that it did not have authority to issue a Surety
Bond when it did is an admission of fraud committed against Gegroco. No person can claim
benefit from the wrong he himself committed. A representation made is rendered conclusive
upon the person making it and cannot be denied or disproved as against the person relying
thereon.

CASE NO. 38. THE INSULAR ASSURANCE CO. LTD. V. THE HEIRS OF JOSE H.
ALVAREZ, GR NO. 207526, OCTOBER 03, 2018 – DE GUZMAN, YVONNE CHRISTELLE

Facts:

Alvarez applied for and was granted a housing loan by the Union bank. The loan was
secured by a promissory note, real estate mortgage and a mortgage redemption insurance
taken on the life of Alvarez with Union Bank as beneficiary. Alvarez was among the mortgagers
included in the list qualified debtors covered by Group Mortgage Redemption Insurance under
Insular Life.

Alvarez passed away in 1998 and Union Bank filed with Insular Life a death claim under
Alvarez'name.

Union Bank was asked to submit Alvarez‘s birth, marriage and death certificate. Herein,
the claim was denied after determining that Alvarez was not eligible for coverage because he
was supposedly to be more than 60 years of age at the time of loan's approval.

Page 50 of 99
The loan remained unpaid and the property was sold at public auction to Union bank.
Alvarez's heirs filed a complaint for the declaration of the nullity of the contract denying their
knowledge of any loan obtained by Alvarez.

On the other hand, Insular Life maintained that based on the documents submitted by
UnionBank, Alvarez was no longer eligible under the Group Mortgage Redemption Insurance
since he was more than 60 years old when his loan was approved

Issue: Whether or not petitioner Insular Life Assurance Co., Ltd. is obliged to pay Union Bank of
the Philippines the balance of Jose H. Alvarez‘s loan given the claim that he lied about his age
at the time of the approval of his loan

Held:

Petitioners Union Bank of the Philippines and The Insular Life Assurance Co., Ltd. are
ordered to comply with the insurance undertaking under Mortgage Redemption Insurance
Policy.

In insurance contracts, concealing material facts is inherently fraudulent: ―if a material


fact is actually known to the [insured], its concealment must of itself necessarily be a fraud.‖

What this case involves, instead, is an allegedly false representation. Section 44 of the
Insurance Code states, ―A representation is to be deemed false when the facts fail to
correspond with its assertions or stipulations.‖ If indeed Alvarez misdeclared his age such that
his assertion fails to correspond with his factual age, he made a false representation, not a
concealment.

Section 26 defines concealment as ―[a] neglect to communicate that which a party


knows and ought to communicate.‖

Here, as pointed out by the CA, Insular Life, failed to establish this defense. It only relied
on Alvarez‘s Health Statement Form where he wrote ―1942‖ as his birth year. This form alone
was insufficient to prove that he fraudulently intended to misrepresent his age because aside
from this, Alvarez had to fill out an application for insurance. This application would have
supported the conclusion that he consistently wrote ―1942‖ in all the documents that he had
submitted to Union Bank. However, the records made no reference to this document.

CASE NO. 39. SUN LIFE OF CANADA (PHILIPPINES), INC., V. MA. DAISY'S. SIBYA,
JESUS MANUEL S. SIBYA III, JAIME LUIS S. SIBYA, AND THE ESTATE OF THE
DECEASED ATTY. JESUS SIBYA, JR., GR NO. 211212, JUNE 08, 2016 – ALPINE
CHIWERAN

FACTS:

On January 10, 2001, Atty. Jesus Sibya, Jr. (Atty. Jesus Jr.) applied for life insurance with Sun
Life Canada (Philippines), Inc. In his Application for Insurance, he indicated that he had sought
advice for kidney problems. On February 5, 2001, Sun Life approved Atty. Jesus Jr.'s
application and issued Insurance Policy. The Policy indicated the respondents as beneficiaries
and entitles them to a death benefit of Pl,000,000.00 should Atty. Jesus Jr. dies on or before
February 5, 2021, or a sum of money if Atty. Jesus Jr. is still living on the endowment date.

Page 51 of 99
On May 11, 2001, Atty. Jesus Jr. died as a result of a gunshot wound in San Joaquin, Iloilo.
Thus, Ma. Daisy filed a Claimant's Statement with Sun Life to seek the death benefits indicated
in his insurance policy. Sun Life denied the claim on the ground that the details on Atty. Jesus
Jr. 's medical history were not disclosed in his application. Simultaneously, Sun Life tendered a
check representing the refund of the premiums paid by Atty. Jesus Jr. Despite demands of Ma.
Daisy, Sun Life refused to heed the requests and instead filed a Complaint for Rescission of
Atty. Jesus Jr. 's insurance policy. Sun Life alleges in the complaint that Atty. Jesus Jr. did not
disclose in his insurance application his previous medical treatment at the National Kidney
Transplant Institute in May and August of 1994. The undisclosed fact suggested that the
insured was in "renal failure" and at a high-risk medical condition. Consequently, had it known
such fact, it would not have issued the insurance policy in favor of Atty. Jesus Jr.

ISSUE:

Whether there was concealment or misrepresentation when Atty. Jesus Jr. submitted his
insurance application with Sun Life.

HELD:

No.

In Manila Bankers Life Insurance Corporation v. Aban, the Court held that if the insured dies
within the two-year contestability period, the insurer is bound to make good its obligation under
the policy, regardless of the presence or lack of concealment or misrepresentation.

In the present case, Sun Life issued Atty. Jesus Jr.'s policy on February 5, 2001. Thus, it has
two years from its issuance, to investigate and verify whether the policy was obtained by fraud,
concealment, or misrepresentation. Upon the death of Atty. Jesus Jr., however, on May 11,
2001, or a mere three months from the issuance of the policy, Sun Life loses its right to rescind
the policy. As discussed in Manila Bankers, the death of the insured within the two-year period
will render the right of the insurer to rescind the policy nugatory. As such, the incontestability
period will now set in.

Also, Atty. Jesus Jr. admitted in his application his medical treatment for kidney ailment.
Moreover, he executed an authorization in favor of Sun Life to conduct investigation in reference
with his medical history. It also appears that Atty. Jesus Jr. also signed the Authorization, which
gave Sun Life the opportunity to obtain information on the facts disclosed by Atty. Jesus Jr. in
his insurance application. Given the express language of the Authorization, it cannot be said
that Atty. Jesus Jr. concealed his medical history since Sun Life had the means of ascertaining
Atty. Jesus Jr.'s medical record. As to allegations of misrepresentation Atty. Jesus Jr. was not a
medical doctor, and his answer "no recurrence" may be construed as an honest opinion. Where
matters of opinion or judgment are called for, answers made in good faith and without intent to
deceive will not avoid a policy even though they are untrue. Indeed, the intent to defraud on the
part of the insured must be ascertained to merit rescission of the insurance contract.
Concealment as a defense for the insurer to avoid liability is an affirmative defense and the duty
to establish such defense by satisfactory and convincing evidence rests upon the provider or
insurer. In the present case, Sun Life failed to clearly and satisfactorily establish its allegations,
and is therefore liable to pay the proceeds of the insurance.

Page 52 of 99
CASE NO. 40. MANULIFE PHILIPPINES, INC. V. HERMENEGILDA YBAÑEZ, GR NO
204736, NOVEMBER 28, 2016 – PUA, JOMARIE CASSANDRA

Facts:

Manulife Philippines, Inc. (Manulife) instituted a Complaint for Rescission of Insurance


Contracts against Hermenegilda Ybañez (Hermenegilda) and the BPI Family Savings Bank (BPI
Family).

It is alleged in the complaint that the insurance which Manulife issued in favor of Dr.
Gumersindo Solidum Ybañez (insured), were void due to concealment or misrepresentation of
material facts in the latter's applications for life insurance and that Hermenegilda, wife of the
said insured, was revocably designated as beneficiary in the subject insurance policies and that
on November 17, 2003, when one of the subject insurance policies had been in force for only
one year and three months, while the other for only four months, the insured died; that Manulife
conducted an investigation into the circumstances leading to the said insured's death, in view of
the aforementioned entries in the said insured's Death Certificate; that Manulife thereafter
concluded that the insured misrepresented or concealed material facts at the time the subject
insurance policies were applied for; and that for this reason Manulife accordingly denied
Hermenegilda's death claims and refunded the premiums that the insured paid on the subject
insurance policies.

BPI Family filed a Manifestation praying that either it be dropped from the case or that
the case be dismissed with respect to it, since no objection was interposed to this prayer by
either Manulife or Hermenegilda. The RTC granted the prayer. Manulife presented its sole
witness Jessiebelle Victoriano (Victoriano), the Senior Manager of its Claims and Settlements
Department. The oral testimony of this witness chiefly involved identifying herself as the Senior
Manager of Manulife's Claims and Settlements Department and also identifying the evidence.

After due proceedings, the RTC dismissed Manulife's Complaint. The RTC found no
merit at all in Manulife's Complaint for rescission of the subject insurance policies because it
utterly failed to prove that the insured had committed the alleged misrepresentation/s or
concealment/s. The CA affirmed the decision of RTC.

Issue:

Whether the CA committed any reversible error in affirming the RTC Decision dismissing
Manulife's Complaint for rescission of insurance contracts for failure to prove concealment on
the part of the insured.

Held:

No. The RTC correctly held that the CDH‘s medical records that might have established
the insured‘s purported misrepresentation/s or concealment/s was inadmissible for being
hearsay, given the fact that Manulife failed to present the physician or any responsible official of
the CDH who could confirm or attest to the due execution and authenticity of the alleged
medical records.

Manulife's sole witness gave no evidence at all relative to the particulars of the purported
concealment or misrepresentation allegedly perpetrated by the insured. In fact, Victoriano
merely perfunctorily identified the documentary exhibits produced by Manulife. She never
testified in regard to the circumstances attending the execution of these documentary exhibits

Page 53 of 99
much less in regard to its contents. Of course, the mere mechanical act of identifying these
documentary exhibits, without the testimonies of the actual participating parties thereto, adds up
to nothing. These documentary exhibits did not automatically validate or explain themselves.

The fraudulent intent on the part of the insured must be established to entitle the insurer
to rescind the contract. Misrepresentation as a defense of the insurer to avoid liability is an
affirmative defense and the duty to establish such defense by satisfactory and convincing
evidence rests upon the insurer.

For failure of Manulife to prove intent to defraud on the part of the insured, it cannot
validly sue for rescission of insurance contracts.

CASE NO. 41. MANILA BANKERS LIFE INSURANCE CORPORATION V. CRESENCIA P.


ABAN, G.R. NO. 175666, JULY 29, 2013 – DE GUZMAN, YVONNE CHRISTELLE

Facts:

1. Delia Sotero took out a life insurance policy from Manila Bankers Life Insurance Corporation
(Bankers Life), designating respondent Cresencia P. Abanher niece,as her beneficiary.

2. Petitioner Banker‘s Life issued Insurance Policy on August 30, 1993.

3. On April 10 1996, more than two years and seven months after the policy had been in force,
Sotero died. Respondent Aban now claims the proceeds of the insurance policy.

4. Petitioner denied the claim on the basis of: Sotero did not personally apply for insurance
coverage, as she was illiterate; she was sickly since 1990; She did not have the financial
capability to pay the insurance premiums on Insurance Policy; She did not sign the July 3, 1993
application for insurance; and Respondent was the one who filed the insurance application,
designated herself as the beneficiary.

5. Petitioner Manila Banker‘s life filed a civil case for rescission and/or annulment of the policy

6. Respondent filed a Motion to Dismiss on ground of Section 48 of the Insurance Code:

Whenever a right to rescind a contract of insurance is given to the insurer by any provision of
this chapter, such right must be exercised previous to the commencement of an action on the
contract.

Issue: Whether the insurer can still rescind the contract

Held: Section 48 serves a noble purpose, as it regulates the actions of both the insurer and the
insured. Under the provision, an insurer is given two years – from the effectivity of a life
insurance contract and while the insured is alive — to discover or prove that the policy is void ab
initio or is rescindible by reason of the fraudulent concealment or misrepresentation of the
insured or his agent.

After the two-year period lapses, or when the insured dies within the period, the insurer
must make good on the policy, even though the policy was obtained by fraud, concealment, or
misrepresentation. This is not to say that insurance fraud must be rewarded, but that insurers
who recklessly and indiscriminately solicit and obtain business must be penalized, for such

Page 54 of 99
recklessness and lack of discrimination ultimately work to the detriment of bona fide takers of
insurance and the public in general.

CASE NO. 42. THELMA VDA. DE CANILANG V. COURT OF APPEALS, G.R. NO. 92492,
JUNE 17, 1993 – MENDOZA, JOYCE ALLYSA

FACTS:

On 18 June 1982, Jaime Canilang consulted Dr. Wilfredo B. Claudio and was diagnosed
as suffering from "sinus tachycardia." The doctor prescribed the following for him: Trazepam, a
tranquilizer; and Aptin, a beta-blocker drug. Mr. Canilang consulted the same doctor again on
August 03, 1982 and this time was found to have "acute bronchitis." On the next day, Jaime
applied for a "non-medical" insurance policy with respondent Great Pacific Life Assurance
Company ("Great Pacific") naming his wife, Thelma Canilang, as his beneficiary. Jaime was
issued ordinary life insurance, with the face value of P19,700, effective as of August 09, 1982.
The next year, Jaime died of "congestive heart failure," "anemia," and "chronic anemia."
Petitioner, widow and beneficiary of the insured, filed a claim with Great Pacific which the
insurer denied upon the ground that the insured had concealed material information from it.
Petitioner then filed a complaint against Great Pacific with the Insurance Commission for
recovery of the insurance proceeds.

ISSUE:

Whether or not the non-disclosure of certain facts about the insured‘s previous health
conditions is material to warrant the denial of the claims of Thelma Canilang

HELD:

Yes, the Court agree with the Court of Appeals that the information which Jaime
Canilang failed to disclose was material to the ability of Great Pacific to estimate the probable
risk he presented as a subject of life insurance. Had Canilang disclosed his visits to his doctor,
the diagnosis made and medicines prescribed by such doctor, in the insurance application, it
may be reasonably assumed that Great Pacific would have made further inquiries and would
have probably refused to issue a non-medical insurance policy or, at the very least, required a
higher premium for the same coverage. The materiality of the information withheld by Great
Pacific did not depend upon the state of mind of Jaime Canilang. A man's state of mind or
subjective belief is not capable of proof in our judicial process, except through proof of external
acts or failure to act from which inferences as to his subjective belief may be reasonably
drawn. Neither does materiality depend upon the actual or physical events which ensue.
Materiality relates rather to the "probable and reasonable influence of the facts" upon the party
to whom the communication should have been made, in assessing the risk involved in making
or omitting to make further inquiries and in accepting the application for insurance; that
"probable and reasonable influence of the facts" concealed must, of course, be determined
objectively, by the judge ultimately.

In any case, in the case at bar, the nature of the facts not conveyed to the insurer was
such that the failure to communicate must have been intentional rather than merely
inadvertent. For Jaime Canilang could not have been unaware that his heart beat would at
times rise to high and alarming levels and that he had consulted a doctor twice in the two (2)
months before applying for non-medical insurance. Indeed, the last medical consultation took

Page 55 of 99
place just the day before the insurance application was filed. In all probability, Jaime went to
visit his doctor precisely because of the discomfort and concern brought about by his
experiencing "sinus tachycardia."

The Court finds it difficult to take seriously the argument that Great Pacific had waived
inquiry into the concealment by issuing the insurance policy notwithstanding Canilang's failure
to set out answers to some of the questions in the insurance application. Such failure precisely
constituted concealment on the part of Canilang. Petitioner's argument, if accepted, would
obviously erase Section 27 from the Insurance Code of 1978.

CASE NO. 43. THE INSULAR LIFE ASSURANCE COMPANY, LTD, V. PAZ Y. KHU, FELIPE
Y. KHU, JR., AND FREDERICK Y. KHU, G.R. NO. 195176, APRIL 18, 2016 – PUA, JOMARIE
CASSANDRA

Facts:

In March 1997, Felipe Khu, Sr. (Felipe) applied for a life insurance policy with Insular Life
under the latter‘s Diamond Jubilee Insurance Plan. Felipe accomplished the required medical
questionnaire wherein he did not declare any illness or adverse medical condition. This took
effect on June 22, 1997.

In June 1999, Felipe‘s policy lapsed due to non-payment of the premium covering the
period from June 22, 1999 to June 23, 2000. In September 1999 Felipe applied for the
reinstatement of his policy and paid P25,020.00 as premium. Except for the change in his
occupation of being self-employed to being the Municipal Mayor of Binuangan, Misamis
Oriental, all the other information submitted by Felipe in his application for reinstatement was
virtually identical to those mentioned in his original policy. In Oct 1999, Insular Life advised
Felipe that his application for reinstatement may only be considered if he agreed to certain
conditions such as payment of additional premium and the cancellation of the riders pertaining
to premium waiver and accidental death benefits. Felipe agreed to these conditions and paid the
agreed additional premium.

In Jan 2000, Insular Life reinstated Felipe‘s policy and issued an Endorsement stating:
This certifies that as agreed by the Insured, the reinstatement of this policy has been approved
by the Company on the understanding that the following changes are made on the policy
effective June 22, 1999. In Sept 2001, Felipe died. Paz Y. Khu, Felipe Y. Khu, Jr. and Frederick
Y. Khu (Felipe‘s beneficiaries or respondents) filed with Insular Life a claim for benefit under the
reinstated policy. Insular Life denied the claim. It rescinded the reinstated policy on the grounds
of concealment and misrepresentation by Felipe since he did not disclose the ailments (Type 2
Diabetes Mellitus, Diabetes Nephropathy and Alcoholic Liver Cirrhosis with Ascites) that he
already had prior to his application for reinstatement of his insurance policy; and that it would
not have reinstated the insurance policy had Felipe disclosed the material information on his
adverse health condition. It contended that when Felipe died, the policy was still contestable.
Respondents instituted a complaint for specific performance with damages.

The RTC and CA ruled in favor of respondents.

Issue:

Whether Felipe‘s reinstated life insurance policy is already incontestable at the time of
his death.

Page 56 of 99
Held:
YES.

Sec 48 of the Insurance Code provides: After a policy of life insurance made payable on the
death of the insured shall have been in force during the lifetime of the insured for a period of two
years from the date of its issue or of its last reinstatement, the insurer cannot prove that the
policy is void ab initio or is rescindible by reason of the fraudulent concealment or
misrepresentation of the insured or his agent.

The insurer is deemed to have the necessary facilities to discover such fraudulent concealment
or misrepresentation within a period of two (2) years. It is not fair for the insurer to collect the
premiums as long as the insured is still alive, only to raise the issue of fraudulent concealment
or misrepresentation when the insured dies in order to defeat the right of the beneficiary to
recover under the policy. At least two (2) years from the issuance of the policy or its last
reinstatement, the beneficiary is given the stability to recover under the policy when the insured
dies. The provision also makes clear when the two-year period should commence in case the
policy should lapse and is reinstated, that is, from the date of the last reinstatement.

In the Letter of Acceptance, Khu declared that he was accepting the imposition of an
extra/additional premium of P5.00 a year per thousand of insurance effective June 22, 1999. It
is true that the phrase as used in this particular paragraph does not refer explicitly to the
effectivity of the reinstatement. But the Court notes that the reinstatement was conditioned upon
the payment of additional premium not only prospectively, that is, to cover the remainder of the
annual period of coverage, but also retroactively, that is for the period starting June 22, 1999.
Hence, by paying the amount of P3,054.50 on December 27, 1999 in addition to the P25,020.00
he had earlier paid on September 7, 1999, Khu had paid for the insurance coverage starting
June 22, 1999. At the very least, this circumstance has engendered a true lacuna. In the
Endorsement, the obscurity is patent. In the first sentence of the Endorsement, it is not entirely
clear whether the phrase "effective June 22, 1999" refers to the subject of the sentence, namely
"the reinstatement of this policy," or to the subsequent phrase "changes are made on the
policy." Given the obscurity of the language, the construction favorable to the insured will be
adopted by the courts.

Accordingly, the subject policy is deemed reinstated as of June 22, 1999. Thus, the period of
contestability has lapsed.

CASE NO. 44. MA. LOURDES FLORENDO V. PHILAM PLANS INC., PERLA ABCEDE,
CELESTE ABCEDE, G.R. NO. 186983 FEBRUARY 22, 2012 – ABRIGO, KAREN

FACTS:

On October 23, 1997 Manuel Florendo filed an application for comprehensive pension
plan with respondent Philam Plans, Inc. (Philam Plans) after some convincing by respondent
Perla Abcede. The plan had a pre-need price of ₱997,050.00, payable in 10 years, and had a
maturity value of ₱2,890,000.00 after 20 years. Manuel signed the application and left to Perla
the task of supplying the information needed in the application. Respondent Ma. Celeste
Abcede, Perla‘s daughter, signed the application as sales counselor.

Aside from pension benefits, the comprehensive pension plan also provided life
insurance coverage to Florendo. This was covered by a Group Master Policy that Philippine
American Life Insurance Company (Philam Life) issued to Philam Plans. Under the master

Page 57 of 99
policy, Philam Life was to automatically provide life insurance coverage, including accidental
death, to all who signed up for Philam Plans‘ comprehensive pension plan.

On September 15, 1998, Manuel died of blood poisoning prompting Lourdes to file a
claim with Philam Plans for the payment of the benefits under her husband‘s plan. However,
Philam declined Lourdes claim upon learning that Manuel was on maintenance medicine for his
heart and had an implanted pacemaker. Further, he suffered from diabetes mellitus and was
taking insulin. Lourdes renewed her demand for payment under the plan but Philam Plans
rejected, hence this present action before the Regional Trial Court (RTC) of Quezon City.

RTC Ruling: The RTC ruled that Manuel was not guilty of concealing the state of his
health from his pension plan application and ordered Philam Plans, Perla and Ma. Celeste,
solidarily, to pay Lourdes all the benefits from her husband‘s pension plan.

CA Ruling: (CA) reversed the RTC decision, holding that insurance policies are
traditionally contracts uberrimae fidae or contracts of utmost good faith. As such, it required
Manuel to disclose to Philam Plans conditions affecting the risk of which he was aware or
material facts that he knew or ought to know.

ISSUE:

Whether the policy is void for misrepresentation by Manuel.

RULING:

As already stated, Manuel had been taking medicine for his heart condition and diabetes
when he submitted his pension plan application. These clearly fell within the five-year period.
More, even if Perla‘s knowledge of Manuel‘s pacemaker may be applied to Philam Plans under
the theory of imputed knowledge, it is not claimed that Perla was aware of his two other
afflictions that needed medical treatments. Pursuant to Section 27 of the Insurance Code,
Manuel‘s concealment entitles Philam Plans to rescind its contract of insurance with him.

As the Court said in New Life Enterprises v. Court of Appeals, 207 SCRA 669
(1992):

It may be true that x x x insured persons may accept policies without reading
them, and that this is not negligence per se. But, this is not without any exception. It is
and was incumbent upon petitioner Sy to read the insurance contracts, and this can be
reasonably expected of him considering that he has been a businessman since 1965 and the
contract concerns indemnity in case of loss in his money-making trade of which important
consideration he could not have been unaware as it was precisely the reason for his procuring
the same. The same may be said of Manuel, a civil engineer and manager of a
construction company. He could be expected to know that one must read every
document, especially if it creates rights and obligations affecting him, before signing the
same. Manuel is not unschooled that the Court must come to his succor. It could
reasonably be expected that he would not trifle with something that would provide
additional trifle with something that would provide additional financial security to him
and to his wife in his twilight years. In a final attempt to defend her claim for benefits under
Manuel‘s pension plan, Lourdes points out that any defect or insufficiency in the information
provided by his pension plan application should be deemed waived after the same has been
approved, the policy has been issued, and the premiums have been collected. The Court cannot
agree. The comprehensive pension plan that Philam Plans issued contains a one-year
incontestability period. It states:

Page 58 of 99
VIII. INCONTESTABILITY After this Agreement has remained in force for one (1) year,
we can no longer contest for health reasons any claim for insurance under this Agreement,
except for the reason that installment has not been paid (lapsed), or that you are not insurable
at the time you bought this pension program by reason of age. If this Agreement lapses but is
reinstated afterwards, the one (1) year contestability period shall start again on the date of
approval of your request for reinstatement. The above incontestability clause precludes the
insurer from disowning liability under the policy it issued on the ground of concealment or
misrepresentation regarding the health of the insured after a year of its issuance. Since Manuel
died on the eleventh month following the issuance of his plan, the one year incontestability
period has not yet set in. Consequently, Philam Plans was not barred from questioning Lourdes‘
entitlement to the benefits of her husband‘s pension plan

CASE NO. 45. MALAYAN INSURANCE CO., INC. V. PAP CO., LTD, G.R. NO. 200784,
AUGUST 7, 2013, 716 PHIL 155-171 – MANGAOANG, NICOLE JOYCE

FACTS:

Malayan Insurance issued a fire insurance policy to PAP Co., Ltd. (PAP Co.) for the
latter‘s machineries and equipment located at Sanyo Precision Phils. Bldg., Phase III, Lot 4,
Block 15, PEZA, Rosario, Cavite (Sanyo Building). After the passage of almost a year but prior
to the expiration of the insurance coverage, PAP Co. renewed the policy on an "as is" basis.

On October 12, 1997 and during the subsistence of the renewal policy, the insured
machineries and equipment were totally lost by fire. Hence, PAP Co. filed a fire insurance claim
with Malayan in the amount insured.

In a letter, dated December 15, 1997, Malayan denied the claim upon the ground that, at
the time of the loss, the insured machineries and equipment were transferred by PAP Co. to a
location different from that indicated in the policy.

ISSUE/S:
1. Whether Malayan should be held liable for the loss of the insured properties under the fire
insurance policy.

2. Whether Malayan is entitled to the rescission of the contract


- Whether PAP committed concealment, misrepresentation and a breach of material warranty.

RULING:
1. NO.

The policy forbade the removal of the insured properties unless sanctioned by Malayan
Condition No. 9(c) of the renewal policy provides:
9. Under any of the following circumstances the insurance ceases to attach as regards the
property affected unless the insured, before the occurrence of any loss or damage, obtains the
sanction of the company signified by endorsement upon the policy, by or on behalf of the
Company:
xxx xxx xxx
(c) If property insured be removed to any building or place other than in that which is herein
stated to be insured.

Page 59 of 99
Evidently, by the clear and express condition in the renewal policy, the removal of the insured
property to any building or place required the consent of Malayan. Any transfer effected by the
insured, without the insurer‘s consent, would free the latter from any liability.

The respondent failed to notify, and to obtain the consent of, Malayan regarding the removal.

2. YES. The terms and conditions in the renewal policy provided, among others, that the
location of the risk insured against is at the Sanyo factory in PEZA. The subject insured
properties, however, were totally burned at the Pace Factory. There being an unconsented
removal, the transfer was at PAP‘s own risk.

It can also be said that with the transfer of the location of the subject properties, without notice
and without Malayan‘s consent, PAP clearly committed concealment, misrepresentation and a
breach of a material warranty. Section 26 of the Insurance Code provides:

Section 26. A neglect to communicate that which a party knows and ought to communicate, is
called a concealment.

Under Section 27 of the Insurance Code, "a concealment entitles the injured party to rescind a
contract of insurance."

Moreover, under Section 168 of the Insurance Code, the insurer is entitled to rescind the
insurance contract in case of an alteration in the use or condition of the thing insured. Section
168 of the Insurance Code provides, as follows:

Section 68. An alteration in the use or condition of a thing insured from that to which it is limited
by the policy made without the consent of the insurer, by means within the control of the
insured, and increasing the risks, entitles an insurer to rescind a contract of fire insurance.

Accordingly, an insurer can exercise its right to rescind an insurance contract when the following
conditions are present, to wit:

1) the policy limits the use or condition of the thing insured;

2) there is an alteration in said use or condition;

3) the alteration is without the consent of the insurer;

4) the alteration is made by means within the insured‘s control; and

5) the alteration increases the risk of loss.

In the case at bench, all these circumstances are present. It was clearly established that the
renewal policy stipulated that the insured properties were located at the Sanyo factory; that PAP
removed the properties without the consent of Malayan; and that the alteration of the location
increased the risk of loss.

Page 60 of 99
CASE NO. 46. GREAT PACIFIC LIFE ASSURANCE CORP. V. COURT OF APPEALS AND
MEDARDA V. LEUTERIO, G.R. NO. 113899, OCTOBER 13, 1999 – CAWIS, AUDREY

FACTS:

A contract of group life insurance was executed between petitioner Great Pacific and
Development Bank Grepalife agreed to insure the lives of eligible housing loan mortgagors of
DBP. Wilfredo Leuterio, a physician and a housing debtor of DBP, applied for membership in
the group life insurance plan. In an application form, Dr. Leuterio answered questions
concerning his health condition as follows:

“7. Have you ever had, or consulted, a physician for a heart condition, high blood
pressure, cancer, diabetes, lung, kidney or stomach disorder or any other
physical impairment?

8. Are you now, to the best of your knowledge, in good health?”

Grepalife issued a coverage to the value of P86,200.00 pesos.

Dr. Leuterio died due to ―massive cerebral hemorrhage.‖ DBP submitted a death claim to
Grepalife. Grepalife denied the claim alleging that Dr. Leuterio was not physically healthy when
he applied for insurance coverage.

Grepalife insisted that Dr. Leuterio did not disclose he had been suffering from hypertension,
which caused his death.

Allegedly, such non-disclosure constituted concealment that justified the denial of the claim. The
widow, respondent Medarda V. Leuterio, filed against Grepalife. The trial court rendered a
decision in favor of the respondent widow and against Grepalife.

The Court of Appeals sustained the trial court‘s decision.

ISSUES:

1. Whether the Court of Appeals erred in holding petitioner liable to DBP as beneficiary in a
group life insurance contract from a complaint filed by the widow of the decedent/mortgagor?

2. Whether the Court of Appeals erred in not finding that Dr. Leuterio concealed that he had
hypertension, which would vitiate the insurance contract?

3. Whether the Court of Appeals erred in holding Grepalife liable in the amount of eighty six
thousand, two hundred (P86,200.00) pesos without proof of the actual outstanding mortgage
payable by the mortgagor to DBP.

RULING:

1. NO. Petitioner alleges that the complaint was instituted by the widow of Dr. Leuterio, not the
real party in interest, hence the trial court acquired no jurisdiction over the case.

It argues that when the Court of Appeals affirmed the trial court‘s judgment, Grepalife
was held liable to pay the proceeds of insurance contract in favor of DBP, the indispensable
party who was not joined in the suit. The insured private respondent did not cede to the
mortgagee all his rights or interests in the insurance, the policy stating that: ―In the event of the
debtor‘s death before his indebtedness with the Creditor [DBP] shall have been fully paid, an

Page 61 of 99
amount to pay the outstanding indebtedness shall first be paid to the creditor and the balance of
sum assured, if there is any, shall then be paid to the beneficiary/ies designated by the debtor.‖

When DBP‘s claim was denied, it collected the debt from the mortgagor and took the
necessary action of foreclosure on the residential lot of private respondent. Since a policy of
insurance upon life or health may pass by transfer, will or succession to any person, whether he
has an insurable interest or not, and such person may recover it whatever the insured might
have recovered, the widow of the decedent Dr. Leuterio may file the suit against the insurer,
Grepalife.

2. NO. The medical findings were not conclusive because Dr. Mejia did not conduct an autopsy
on the body of the decedent.

The medical certificate stated that hypertension was ―the possible cause of death.‖
Hence, the statement of the physician was properly considered by the trial court as hearsay.
Contrary to appellant‘s allegations, there was no sufficient proof that the insured had suffered
from hypertension. Aside from the statement of the insured‘s widow who was not even sure if
the medicines taken by Dr. Leuterio were for hypertension, the appellant had not proven nor
produced any witness who could attest to Dr. Leuterio‘s medical history.

Appellant insurance company had failed to establish that there was concealment made
by the insured, hence, it cannot refuse payment of the claim.‖ The fraudulent intent on the part
of the insured must be established to entitle the insurer to rescind the contract.

Misrepresentation as a defense of the insurer to avoid liability is an affirmative defense


and the duty to establish such defense by satisfactory and convincing evidence rests upon the
insurer.

3. NO. A life insurance policy is a valued policy. Unless the interest of a person insured is
susceptible of exact pecuniary measurement, the measure of indemnity under a policy of
insurance upon life or health is the sum fixed in the policy.

The mortgagor paid the premium according to the coverage of his insurance. In the
event of the debtor‘s death before his indebtedness with the creditor shall have been fully paid,
an amount to pay the outstanding indebtedness shall first be paid to the creditor. DBP
foreclosed one of the deceased person‘s lots to satisfy the mortgage.

Hence, the insurance proceeds shall inure to the benefit of the heirs of the deceased
person or his beneficiaries.

CASE NO. 47. PIONEER INSURANCE AND SURETY CORPORATION V. OLIVA YAP, G.R.
NO. L-36232, DECEMBER 19, 1974 – DELA CRUZ, JEREMY KIMBERLY

FACTS:
Yap took out a Fire Insurance Policy with Pioneer for her two-storey store building. The
policy requires Yap to give Pioneer notice of co-insurers covering the same subject matter
under the condition that failure to give such notice shall lead to the forfeiture of the proceeds
under the policy. At the time of issuance of policy Yap had a P20k fire insurance with Great
American Insurance Co., this was noted in the policy with Pioneer as co-insurance. A
subsequent endorsement on the policy indicated Northwest Insurance as a co-insurer. Yap
furthermore took out another insurance from Federal Insurance Co. Inc. A fire then broke on
Yap‘s store. She filed a claim with Pioneer but the latter denied on ground of breach of condition

Page 62 of 99
on the policy. Yap filed a complaint for recovery of the face value of the insurance proceeds with
the CFI. The CFI ruled in favor of Yap. The CA affirmed. The SC reversed.

ISSUES:
1. Should Pioneer be absolved from liability on Fire Insurance on account of any violation by
Yap of the co-insurance clause therein?
2. Was the formal requirement of endorsing the policy of co-insurance waived by Pioneer ―since
there was absolutely no showing that it was not aware of said substitution and preferred to
continue the policy‖?

HELD:
a. YES. The SC held that there is no evidence to establish and prove such a substitution.
CA‘s finding to the effect that the policy issued by Federal was a substitution of the
policy issued by Great American was unsubstantiated, contrary to stipulation and
admission of Yap, and is grounded entirely on speculation, surmises or conjectures,
hence, not binding on the Supreme Court.

By the plain terms of the policy, other insurance without the consent of petitioner
would ipso facto avoid the contract. It required no affirmative act of election on the part
of the company to make operative the clause avoiding the contract, wherever the
specified conditions should occur. Its obligations ceased, unless, being informed of the
fact, it consented to the additional insurance.

A clause in a policy to the effect that the procurement of additional insurance


without the consent of the insurer renders the policy void is a valid provision. Upon
violation, the insurer may terminate the contract at any time, at its option, by giving
notice and refunding a ratable portion of the premium. (Milwaukee Mechanids‘ Lumber
Co. vs. Gibson)

The purpose of said Other Insurance Clauses in the insurance policies was to
reduce moral hazard, that is, to prevent over-insurance and thus avert the perpetration
of fraud. The public, as well as the insurer, is interested in preventing the situation in
which a fire would be profitable to the insured.

These conditions partake the nature, and must be deemed to be a warranty.


(General Insurance & Surety Corporation vs. Ng Hua)

b. NO. Yap has the burden of proof to show that Pioneer was aware of the substitution.
Section 1, Rule 131 of the Revised Rules of Court, requires each party to prove his own
allegations. The CA shifted to Pioneer, Yap‘s burden of proving her proposition that
Pioneer was aware of the alleged substitution, and with such knowledge preferred to
continue the policy. Furthermore, a waiver must be express. If it is to be implied from
conduct mainly, said conduct must be clearly indicative of a clear intent to waive such

Page 63 of 99
right. Nothing less than a clear, positive waiver, made with full knowledge of the
circumstances, must be required. Judgment reversed and set aside.

CASE NO. 48. GENERAL INSURANCE AND SURETY CORPORATION VS. NG HUA, G.R.
NO. L-14373 – CHAWI, DECIMAY

FACTS:

On April 15, 1952, the defendant General Insurance and Surety Corporation issued its
insurance Policy No. 471, insuring against fire, for one year, the stock in trade of the Central
Pomade Factory owned by Ng Hua, the court insured. The next day, the Pomade factory
building burned, resulting in destruction by fire of the insured properties. Ng Hua claimed
indemnity from the insurer. The policy covered damages up to P10,000.00; but after some
negotiations and upon suggestion of the Manila Adjustment Company, he reduced the claim of
P5,000.00. Nevertheless, the defendant insurer refused to pay for various reasons, namely (a)
action was not filed in time; (b) violation of warranty; (c) submission of fraudulent claim; and (f)
failure to pay the premium.

Ng Hua had obtained fire insurance on the same goods, for the same period of time, in
the amount of P20,000.00 from General Indemnity Co. However, the Court of Appeals referring
to the annotation and overruling the defense, held that there was no violation of the above
clause, inasmuch as "co-insurance exists when a condition of the policy requires the insured to
bear ratable proportion of the loss when the value of the insured property exceeds the face
value of the policy," hence there is no co-insurance here.

ISSUE: Whether there is warranty breach or concealment of the other insurance and/or violation
of the provision of the policy

HELD:

Other insurers of the same property against the same hazard are sometimes referred as
co-insurers and the ensuing combination as co-insurance. And considering the terms of the
policy which required the insured to declare other insurances, the statement in question must be
deemed to be a statement (warranty) binding on both insurer and insured, that there were no
other insurance on the property. Remember it runs "Co-Insurance declared"; emphasis on the
last word. If "Co-Insurance" means that the Court of Appeals says, the annotation served no
purpose. It would even be contrary to the policy itself, which made the insured a co-insurer for
the excess of the value of the property over the amount of the policy.

The annotation must be deemed to be a warranty that the property was not insured by
any other policy. Violation thereof entitles the insurer to rescind. (Sec. 69. Insurance Act) Such
misrepresentation is fatal. The materiality of non-disclosure of other insurance policies is not
open to doubt.

Page 64 of 99
Even if the annotations were overlooked, the defendant insurer would still be free from
liability because there is no question that the policy issued by General Indemnity had not been
stated in nor endorsed on Policy No. 471 of defendant. And as stipulated in the provisions of
such policy "all benefit under this policy shall be forfeited."

To avoid the dissastrous effect of the misrepresentation or concealment of the other


insurance policy, Ng Hua alleges "actual knowledge" on the part of General insurance of the
fact that he had taken out additional insurance with General Indemnity. He does not say when
such knowledge was acquired or imparted. If General Insurance know before issuing its policy
or before the fire, such knowledge might overcome the insurer's defense. However, the Court of
Appeals found no evidence of such knowledge. As to knowledge of General Insurance before
issuance of its policy or the fire, there was none.

The defendant insurer (herein petitioner) is acquitted from all the liability under the
policy.

CASE NO. 49. COMMUNICATION AND INFORMATION SYSTEMS CORPORATION V.


MARK SENSING AUSTRALIA PTY. LTD., MARK SENSING PHILIPPINES, INC. AND
OFELIA B. CAJIGAL, G.R. NO. 192159, JANUARY 25, 2017 – ALPINE CHIWERAN

FACTS:

MS APL and CISC entered into a MOA where CISC was appointed as the exclusive agent of
MS APL to PCSO. MS APL agreed to pay CISC a commission of 24.5% of future sales to
PCSO. Initially, MS APL was complying with its obligation until it stopped due to some
irregularities. Litigations between parties involved in PCSO transactions surfaced until
eventually settled by a compromised agreement. MS APL felt short changed by CISC‘s efforts
and, then, decided to withhold payment of commissions.

Due to MS APL‘s refusal to pay, CISC filed a complaint before the RTC. It reached the court of
Appeals. Thus, the present case faced the SC.

ISSUE

Whether the RTC committed grave abuse of discretion when it approved the attachment bond
whose face amount exceeded the retention limit of the surety.

HELD:

No.

Section 215 of the old Insurance Code, the law in force at the time Plaridel issued the
attachment bond, limits the amount of risk that insurance companies can retain to a maximum of
20% of its net worth. However, in computing the retention limit, risks that have been ceded to
authorized reinsurers are ipso jure deducted. In mathematical terms, the amount of retained risk
is computed by deducting ceded/reinsured risk from insurable risk. If the resulting amount is
below 20% of the insurer‘s net worth, then the retention limit is not breached. In this case, both
the RTC and CA determined that, based on Plaridel‘s financial statement that was attached to
its certificate of authority issued by the Insurance Commission, its net worth is P289,332,999.00.
Plaridel‘s retention limit is therefore P57,866,599.80, which is below the P113,197,309.10 face

Page 65 of 99
value of the attachment bond. However, it only retained an insurable risk of P17,377,938.19
because the remaining amount of P98,819,770.91 was ceded to 16 other insurance companies.
Thus, the risk retained by Plaridel is actually P40 Million below its maximum retention limit.
Therefore, the approval of the attachment bond by the RTC was in order. Contrary to MSAPL‘s
contention that the RTC acted with grave abuse of discretion, we find that the RTC not only
correctly applied the law but also acted judiciously when it required Plaridel to submit proof of its
reinsurance contracts after MSAPL questioned Plaridel‘s capacity to underwrite the attachment
bond. Apparently, MSAPL failed to appreciate that by dividing the risk through reinsurance,
Plaridel‘s attachment bond actually became more reliable — as it is no longer dependent on the
financial stability of one company — and, therefore, more beneficial to MSAPL. In cancelling
Plaridel‘s insurance bond, the Court of Appeals (CA) also found that because the reinsurance
contracts were issued in favor of Plaridel, and not MSAPL, these failed to comply with the
requirement of Section 4, Rule 57 of the Rules of Court requiring the bond to be executed to the
adverse party. This led the CA to conclude that ―the bond has been improperly and insufficiently
posted.‖ We reverse the CA and so hold that the reinsurance contracts were correctly issued in
favor of Plaridel.

A contract of reinsurance is one by which an insurer (the ―direct insurer‖ or ―cedant‖) procures a
third person (the ―reinsurer‖) to insure him against loss or liability by reason of such original
insurance. It is a separate and distinct arrangement from the original contract of insurance,
whose contracted risk is insured in the reinsurance agreement. The reinsurer‘s contractual
relationship is with the direct insurer, not the original insured, and the latter has no interest in
and is generally not privy to the contract of reinsurance. Put simply, reinsurance is the
―insurance of an insurance.‖

CASE NO. 50. INDUSTRIAL PERSONNEL AND MANAGEMENT SERVICES, INC.,


PETITIONER, V. COUNTRY BANKERS INSURANCE CORPORATION, RESPONDENT. G.R.
NO. 194126, OCTOBER 17, 2018 – RAGMA, MYKA MARIE KHRISTINE

FACTS:

Industrial Personnel and Management Services, Inc. (IPAMS) began recruiting


registered nurses for work deployment in the United States of America (U.S.). It takes eighteen
(18) to twenty four (24) months for the entire immigration process to complete. As the process
requires huge amounts of money, such amounts are advanced [to] the nurse applicants.

Because of the advances made to the nurse applicants, the latter were required to post
a surety bond. The purpose of the bond is to guarantee the following during its validity period:
(a) that they will comply with the entire immigration process, (b) that they will complete the
documents required, and (c) that they will pass all the qualifying examinations for the issuance
of immigration visa. The Country Bankers Insurance Corporation (Country Bankers) and IPAMS
agreed to provide bonds for the said nurses.

Under the agreement of IPAMS and Country Bankers, the latter will provide surety
bonds and the premiums thereof were paid by IPAMS on behalf of the nurse applicants.

A Memorandum of Agreement (MOA) was executed by the said parties on February 1,


2002 which stipulated the various requirements for collecting claims from Country Bankers.
Among the requirements for collecting claims, the official receipts of the expenses incurred for
the application of nurses are not included.

Page 66 of 99
Country Bankers was not able to pay some of the claims of IPAMS. Country Bakers
acknowledged its obligations, apologized for the delay in the payment of claims, and proposed
to amortize the settlement of claims by paying a semi-monthly amount of P850,000.00. In
addition, Country Bankers promised to pay future claims within a ninety (90)-day period.

Later on, Country Bankers started to oppose the payment of claims and insisted on the
production of official receipts of IPAMS on the expenses it incurred for the application of nurses.
IPAMS opposed this, saying that the Country Bankers' insistence on the production of official
receipts was contrary to, and not contemplated in, the MOA and was an impossible condition
considering that the U.S. authorities did not issue official receipts. In lieu of official receipts,
IPAMS submitted statements of accounts, as provided in the MOA.

ISSUE:

Whether Country Bankers can refuse payment of the claims on the ground of non-
presentation of official receipts.

HELD:

Under Section 92 of the Insurance Code all defects in the proof of loss, which the
insured might remedy, are waived as grounds for objection when the insurer omits to specify to
him without unnecessary delay. It is the duty of the insurer to indicate the defects on the proofs
of loss given, so that the deficiencies may be supplied by the insured. When the insurer
recognizes his liability to pay the claim, there is waiver by the insurer of any defect in the proof
of loss.

In the instant case, it must be emphasized that Country Bankers readily acknowledged
the obligations of Country Bankers under the surety agreement, apologized for the delay in the
payment of claims, and proposed to amortize the settlement of claims by paying a semi-
monthly amount of P850,000.00. In addition, Country Bankers promised to pay future claims
within a 90-day period.

CASE NO. 51. FINMAN GENERAL ASSURANCE CORPORATION VS. COURT OF


APPEALS AND USIPHIL INCORPORATED, G.R. NO. 138737, JULY 12, 2001 – ANTONIO,
PHOEBE ANTONETTE

FACTS:

UsiPhil obtained a fire insurance policy from Finman General Assurance Corporation
(FGAC) covering certain properties, e.g., office, furniture, fixtures, shop machinery and other
trade equipment; FGAC undertook to indemnify UsiPhil for any damage to or loss of said
properties arising from fire. Sometime in 1982, UsiPhil filed with FGAC an insurance claim
amounting to P987,126 for the loss of the insured properties due to fire. UsiPhil likewise
submitted Proof of Loss signed by its Accounting Manager Pedro Palallos and countersigned by
H.H. Bayne‘s Adjuster F.C. Medina. Palallos personally followed-up UsiPhil‘s claim with FGAC‘s
President Joaquin Ortega. During their meeting, Ortega instructed their Finance Manager,
Rosauro Maghirang, to reconcile the records. Thereafter, Maghirang and Palallos signed a
Statement/Agreement, dated February 28, 1985, which indicated that the amount due
respondent was P842,683.40. Despite repeated demands by UsiPhil, petitioner refused to pay
the insurance claim.

Page 67 of 99
Thus, UsiPhil was constrained to file a complaint against petitioner for the unpaid
insurance claim. In its Answer, petitioner maintained that the claim of UsiPhil could not be
allowed because it failed to comply with Policy Condition No. 13 regarding the submission of
certain documents to prove the loss.

ISSUE:

Whether Finman General Assurance Corporation should indemnify Usiphil under the
Fire Insurance policy.

RULING:

Both the trial court and the CA concur in holding that UsiPhil had substantially complied
with Policy Condition No. 13 which reads: ―the insured shall give immediate written notice to the
Company of any loss, protect the property from further damage, forthwith separate the damaged
and undamaged personal property, put it in the best possible order, furnish a complete inventory
of the destroyed, damaged, and undamaged property, showing in detail quantities, costs, actual
cash value and the amount of loss claimed; and within sixty days after the loss, unless such
time is extended in writing by the company the insured shall render to the company a proof of
loss, signed and sworn to by the insured, stating the knowledge and belief of the insured.‖

A perusal of the records shows that UsiPhil, after the occurrence of the fire, immediately
notified petitioner thereof. Thereafter, UsiPhil submitted the following documents: (1) Sworn
Statement of Loss and Formal Claim and; (2) Proof of Loss. The submission of these
documents, to the Court‘s mind, constitutes substantial compliance with the above provision.
Indeed, as regards the submission of documents to prove loss, substantial, not strict as urged
by petitioner, compliance with the requirements will always be deemed sufficient. In any case,
petitioner itself acknowledged its liability when through its Finance Manager, Rosauro
Maghirang, it signed the document indicating that the amount due UsiPhil is P842,683.40.

The policy itself obliges petitioner to pay the insurance claim within thirty days after proof
of loss and ascertainment of the loss made in an agreement between and petitioner. In this
case, as found by the CA, petitioner and private respondent signed the agreement indicating
that the amount due private respondent was P842,683.40 on April 2, 1985. Petitioner thus had
until May 2, 1985 to pay private respondent‘s insurance.

CASE NO. 52. INTEGRATED MICRO ELECTRONICS, INC., VS. STANDARD INSURANCE
CO., INC., G.R. NO. 210302, AUGUST 27, 2020 – VILLAMAR, JOHN OLIVER DUKE

FACTS:

In March 2009, a panel of insurers composed of Standard Insurance, United United


Coconut Planters Bank (UCPB) General Insurance, Co. Inc., Pioneer Insurance, Surety
Corporation, Bank of Philippine Islands (BPI) M/S Insurance Corporation and Malayan
Inusrance Co., Inc., issued Policy No. HOF09FD-FAR086036 in favor of Integrated Micro
Electronics, Inc. insuring all of its properties against ―all risk of physical loss, destruction of, or
damage, including fire, for the period of March 31, 2009 to March 31, 2010.

In May 2009, A fire broke out at Integrated Micro's building causing damage to its
production equipment and machineries. Subsequently, Integrated Micro filed a claim for
indemnity from Standard Insurance but was rejected on the ground that the cause of the loss

Page 68 of 99
was an excluded peril to which Integrated Micro sought reconsideration. A year after, Integrated
Micro filed a complaint for specific performance and damages against Standard Insurance
before the RTC. The latter denied the motion to dismiss and directed them to file a responsive
pleading. Standard Insurance filed a petition for certiorari with the CA to which was granted and
ruled that Integrated Micro's cause of action had prescribed and that the summons was
improperly served

ISSUE:

Whether or not the petition is meritorious

HELD:

The petition is unmeritorious.

Contracts of insurance must be construed according to the sense and meaning of the
terms which the parties themselves have used. If the provisions are clear and unambiguous,
they must be taken and understood in their plain, ordinary and popular sense. This is consistent
with the cardinal rule of interpretation that "[i]f the terms of a contract are clear and leave no
doubt upon the intention of the contracting parties, the literal meaning of its stipulations shall
control. Here, the insurance policy provides the manner and period of filing of a claim.

If a claim be made and rejected and an action or suit be not commenced either in the
Insurance Commission or any Court of competent jurisdiction within twelve (12) months from
receipt of notice of such rejection, or in case of arbitration taking place as provided herein within
twelve (12) months after due notice of the award made by the arbitrator or arbitrators or umpire,
then the claim shall for all purposes be deemed to have been abandoned and shall not
thereafter be recoverable hereunder

It is explicit that if a claim is made and rejected, an action or suit should be commenced
within a period of 12 months. There is no qualification nor distinction whether it is the insurer's
initial or final rejection. The parties did not agree that the insurer should first deny any request
for reconsideration before a suit for indemnity may be filed. Thus, based on the plain and
ordinary context of the agreement, the parties contemplated that the cause of action for loss or
damages arising from the insurance contract shall accrue from rejection of the claim at the first
instance.

CASE NO. 53. SUMMIT GUARANTY & INSURANCE COMPANY V. ARNALDO, 158 SCRA
332, FEBRUARY 29, 1988 – CAPULE, STELLA MARIE

FACTS:

These three consolidated cases arose from three different complaints filed against
Summit Guaranty and Insurance Company, Inc. for the payment of insurance on insurance
policies issued by the company. Private respondents Jose Ledesma, Geronima Pulmano and
Amelia Generao were insured with Summit Guaranty and Insurance Company for purposes of
Third Party Liability. They all filed, in separate cases, notice of claim with Summit Guaranty.
However, the petitioner failed to act on their claims.

Page 69 of 99
Thereafter, Ledesa and Pulmano filed a complaint before the Insurance Commission.
Summit Guaranty claims that the complaints of private respondents, having been filed beyond
the one-year period provided in Section 384 of the Insurance Code, can no longer prosper.

Summit Guaranty argues that under this law, even if the notice of claim was timely filed
with the insurance company within the six-month period, as what happened in the three cases,
the action or suit that follows, if filed beyond the one-year period should necessarily be
dismissed on the ground of prescription.

ISSUE:

Whether the causes of action of private respondents have already prescribed?

RULING:

NO. The Court held that these three cases do not fall within the meaning of ‗proper
cases‘ as contemplated in Section 384 of the Insurance Code. To hold otherwise would enable
the petitioner company to evade its responsibility through a clever scheme it had contrived."

The plaintiff's cause of action did not accrue until his claim was finally rejected by the
insurance

company. This is because, before such final rejection, there was no real necessity for bringing
suit. Since a "cause of action" requires, as essential elements, not only a legal right of the
plaintiff and a correlative obligation of the defendant but also "an act or omission of the
defendant in violation of said legal right," the cause of action does not accrue until the party
obligated refuses, expressly or impliedly, to comply with its duty.

In the cases at bar, no denial of the claims was ever made and on the contrary, private
respondents were made to believe that they will be paid by petitioner company. The alleged
delay was not caused by herein private respondents but by the petitioner company itself.

The one-year period should instead be counted from the date of rejection by the insurer
as this is the time when the cause of action accrues. Since in these cases there has yet been no
accrual of cause of action, the Court holds that prescription has not yet set in.

CASE NO. 54. KEIHIN-EVERETT FORWARDING CO V. TOKIO MARINE MALAYAN


INSURANCE CO., G.R. NO. 212107, JANUARY 28, 2019 – ANTONIO, PHOEBE
ANTONETTE

FACTS:

Honda Trading ordered 80 bundles of Aluminum Alloy Ingots from Indonesia. Honda
Trading insured the entire shipment with TKMNFI under Policy No. 83-00143689. It also
engaged the services of Keihin-Everett to transport and deliver the same to its warehouse in
Laguna. Meanwhile, Keihin-Everett had an Accreditation Agreement with Sunfreight Forwarders
whereby the latter undertook to render common carrier services for the former within the
Philippines. When the shipment arrived in Manila, Keihin-Everett turned over it to Sunfreight
Forwarders for delivery to Honda Trading. En route to the latter‘s warehouse, the truck carrying
the containers was hijacked. As a consequence, Honda Trading suffered losses. Claiming to
have paid Honda Trading‘s insurance claim for the loss it suffered, ―Tokio Marine‖ (not TKMNFI)
filed a suit for damages against Keihin-Everett. It maintained that it had been subrogated to all
the rights and causes of action pertaining to Honda Trading.

Page 70 of 99
The RTC rendered a Decision finding Keihin-Everett and Sunfreight Forwarders jointly
and severally liable to pay Tokio Marine. The RTC found the driver of Sunfreight Forwarders as
the cause of the evil caused. Under Article 2180 of the Civil Code, it provides: ―Employers shall
be liable for the damages caused by their employees and household helpers acting within the
scope of their assigned tasks, even though the former are not engaged in any business or
industry.‖ Thus, Sunfreight Forwarders is hereby held liable for the loss of the subject cargoes
with Keihin-Everett, being a common carrier. Keihin-Everett moved for reconsideration of the
foregoing RTC Decision. However, its motion was denied for lack of merit. Hence, Keihin-
Everett filed an appeal with the Court of Appeals.

The CA modified the ruling of the RTC insofar as the solidary liability of Keihin-Everett
and Sunfreight Forwarders is concerned. The CA went to rule that solidarity is never presumed.
There is solidary liability when the obligation so states, or when the law or the nature of the
obligation requires the same. Thus, because of the lack of privity between Honda Trading and
Sunfreight Forwarders, the latter cannot simply be held jointly and severally liable with Keihin-
Everett for Tokio Marine‘s claim as subrogee. Dissatisfied with the CA Decision, Keihin-Everett
went to the Supreme Court (this Court).

ISSUES:

1. Whether the case should have been dismissed for failure of Tokio Marine to
attach or state in the Complaint the actionable document or the insurance policy
between the insurer and the insured?

2. Whether Tokio Marine is not the insurer but TMNFIC.

a. Whether Tokio Marine is entitled to be reimbursed of what it had paid to


Honda Trading.

3. Whether or not there is legal subrogation in this case.

RULING:

1. Keihin-Everett argued that the case should have been dismissed for failure of Tokio
Marine to attach or state in the Complaint the actionable document or the insurance
policy between the insurer and the insured, in clear violation of Section 7, Rule 8 of the
1997 Rules of Court. It bears to stress that failure of Tokio Marine to attach in the
Complaint the contract of insurance between the insurer (TKMNFI) and the insured
(Honda Trading) is not fatal to its cause of action. In the case of Malayan Insurance Co.,
Inc. vs. Regis Brokerage Corp., relied upon by Keihin-Everett, the Court makes it
imperative for the plaintiff (whose action is predicated upon his right as a subrogee) to
attach the insurance contract in the complaint in accordance with Section 7, Rule 8 of
the 1997 Rules of Court, just so in order to establish the legal basis of the right to
subrogation.

However, in the aforesaid case, the Court did not suggest an outright dismissal of a
complaint in case of failure to attach the insurance contract in the complaint. Promoting
a reasonable construction of the rules so as not to work injustice, the Court makes it
clear that failure to comply with the rules does not preclude the plaintiff to offer it as
evidence. Thus, it may be that there is no specific provision in the Rules of Court which
prohibits the admission in evidence of an actionable document in the event a party fails

Page 71 of 99
to comply with the requirement of the rule on actionable documents under Section 7,
Rule 8. Unfortunately, in the Malayan case cited by Keihin-Everett, Malayan not only
failed to attach or set forth in the complaint the insurance policy, it likewise did not
present the same as evidence before the trial court or even in the CA.

The instant case cannot be dismissed just like that. Unlike in the Malayan case, Tokio
Marine presented as evidence, not only the Insurance Policy, but also the Subrogation
Receipt evidencing that it paid Honda Trading the sum of US$38,855.83 in full
settlement of the latter‘s claim under the said policy. During the trial, Keihin-Everett even
had the opportunity to examine the said documents and conducted a cross-examination
of the said Contract of Insurance. By presenting the insurance policy constitutive of the
insurance relationship of the parties, Tokio Marine was able to confirm its legal right to
recover as subrogee of Honda Trading.

2. Keihin-Everett pointed out that the Insurance Policy shows in its face that Honda
Trading procured the insurance from TMNFIC and not from Tokio Marine. Hence, it
argued that Tokio Marine has no right to institute the present action.

While this assertion is true, the Insurance Policy itself expressly made Tokio Marine as
the party liable to pay the insurance claim of Honda Trading pursuant to the Agency
Agreement entered into by and between Tokio Marine and TMNFIC. As properly
appreciated by the RTC and the CA, the Agency Agreement shows that TMNFIC had
subsequently changed its name to that of Tokio Marine. By agreeing to this stipulation in
the Insurance Policy, Honda Trading binds itself to file its claim from Tokio Marine and
thereafter to accept payment from it.

At any rate, even if we consider Tokio Marine as a third person who voluntarily paid the
insurance claims of Honda Trading, it is still entitled to be reimbursed of what it had paid.
As held by this Court in the case of Pan Malayan Insurance Corp. vs. Court of Appeals,
the insurer who may have no rights of subrogation due to ―voluntary‖ payment may
nevertheless recover from the third party responsible for the damage to the insured
property under Art. 1236 of the Civil Code which provides that the creditor is not bound
to accept payment or performance by a third person who has no interest in the fulfillment
of the obligation, unless there is a stipulation to the contrary. Whoever pays for another
may demand from the debtor what he has paid, except that if he paid without the
knowledge or against the will of the debtor, he can recover only insofar as the payment
has been beneficial to the debtor.

3. Since the insurance claim for the loss sustained by the insured shipment was paid
by Tokio Marine as proven by the Subrogation Receipt – showing the amount paid and
the acceptance made by Honda Trading, it is inevitable that it is entitled, as a matter of
course, to exercise its legal right to subrogation as provided under Art. 2207 of the Civil
Code.

It must be stressed that the Subrogation Receipt only proves the fact of payment. This
fact of payment grants Tokio Marine subrogatory right which enables it to exercise legal
remedies that would otherwise be available to Honda Trading as owner of the hijacked
cargoes as against the common carrier (Keihin-Everett). In other words, the right of
subrogation accrues simply upon payment by the insurance company of the insurance
claim. The payment by the insurer to the insured operates as an equitable assignment to
the insurer of all the remedies which the insured may have against the third party whose

Page 72 of 99
negligence or wrongful act caused the loss. The right of subrogation is not dependent
upon, nor does it grow out of any privity of contract or upon payment by the insurance
company of the insurance claim. It accrues simply upon payment by the insurance
company of the insurance claim.

Consequently, the payment made by Tokio Marine to Honda Trading operates as an


equitable assignment to the former of all the remedies which the latter may have against
Keihin-Everett.

CASE NO. 55. PAN MALAYAN INSURANCE CORPORATION VS. COURT OF APPEALS,
ERLINDA FABIE AND HER UNKNOWN DRIVER, 184 SCRA 54, APRIL 3, 1990 – SOMERA,
MARIA MONICA

FACTS:

On December 10, 1985, PANMALAY filed a complaint for damages with the RTC of
Makati against private respondents Erlinda Fabie and her driver. PANMALAY averred the
following: that it insured a Mitsubishi Colt Lancer car with plate No. DDZ-431 and registered in
the name of Canlubang Automotive Resources Corporation [CANLUBANG]; that on May 26,
1985, due to the ―carelessness, recklessness, and imprudence‖ of the unknown driver of a pick-
up with plate no. PCR-220, the insured car was hit and suffered damages in the amount of
P42,052.00; that PANMALAY defrayed the cost of repair of the insured car and, therefore, was
subrogated to the rights of CANLUBANG against the driver of the pick-up and his employer,
Erlinda Fabie; and that, despite repeated demands, defendants, failed and refused to pay the
claim of PANMALAY.

Private respondents, thereafter, filed a Motion for Bill of Particulars and a supplemental
motion thereto. In compliance therewith, PANMALAY clarified, among others, that the damage
caused to the insured car was settled under the ―own damage‖ coverage of the insurance
policy, and that the driver of the insured car was, at the time of the accident, an authorized
driver duly licensed to drive the vehicle. PANMALAY also submitted a copy of the insurance
policy and the Release of Claim and Subrogation Receipt executed by CANLUBANG in favor of
PANMALAY.

On February 12, 1986, private respondents filed a Motion to Dismiss alleging that
PANMALAY had no cause of action against them. They argued that payment under the ―own
damage‖ clause of the insurance policy precluded subrogation under Article 2207 of the Civil
Code, since indemnification thereunder was made on the assumption that there was no
wrongdoer or no third party at fault.

After hearings conducted on the motion, opposition thereto, reply and rejoinder, the RTC
issued an order dated June 16, 1986 dismissing PANMALAY‘s complaint for no cause of action.
On August 19, 1986, the RTC denied PANMALAY‘s motion for reconsideration.

On appeal taken by PANMALAY, these orders were upheld by the Court of Appeals on
November 27, 1987. Consequently, PANMALAY filed the present petition for review. After
private respondents filed its comment to the petition, and petitioner filed its reply, the Court
considered the issues joined and the case submitted for decision.

Page 73 of 99
ISSUE:

Whether PANMALAY is subrogated to the rights of CANLUBANG upon payment of the


former to the latter.

RULING:

Yes. Article 2207 of the Civil Code is founded on the well-settled principle of
subrogation. If the insured property is destroyed or damaged through the fault or negligence of
a party other than the assured, then the insurer, upon payment to the assured, will be
subrogated to the rights of the assured to recover from the wrongdoer to the extent that the
insurer has been obligated to pay. Payment by the insurer to the assured operates as an
equitable assignment to the former of all remedies which the latter may have against the third
party whose negligence or wrongful act caused the loss. The right of subrogation is not
dependent upon, nor does it grow out of, any privity of contract or upon written assignment of
claim. It accrues simply upon payment of the insurance claim by the insurer.

There are a few recognized exceptions to this rule. For instance, if the assured by his
own act releases the wrongdoer or third party liable for the loss or damage, from liability, the
insurer‘s right of subrogation is defeated. Similarly, where the insurer pays the assured the
value of the lost goods without notifying the carrier who has in good faith settled the assured‘s
claim for loss, the settlement is binding on both the assured and the insurer, and the latter
cannot bring an action against the carrier on his right of subrogation And where the insurer pays
the assured for a loss which is not a risk covered by the policy, thereby effecting ―voluntary
payment‖, the former has no right of subrogation against the third party liable for the loss. None
of the exceptions are availing in the present case.

CASE NO. 56. EQUITABLE INSURANCE CORPORATION V. TRANSMODAL


INTERNATIONAL, INC., G.R. NO. 223592, AUGUST 7, 2017 – GATTOC, SHEKINAH

FACTS:
Sytengco Enterprises Corporation (Sytengco) hired respondent Transmodal
International, Inc. (Transmodal) to clear from the customs authorities and withdraw, transport,
and deliver to its warehouse, cargoes consisting of 200 cartons of gum Arabic. The said
cargoes arrived in Manila and were brought to Ocean Links Container Terminal Center, Inc.
pending their release by the Bureau of Customs (BOC). Respondent then withdrew the same
cargoes and delivered them to Sytengco's warehouse. It was noted in the delivery receipt that
all the containers were wet. A preliminary survey was conducted by Elite Surveyors and it was
found that 187 cartons had watermarks and the contents of the 13 wet cartons were partly
hardened. In its final report, Elite Surveyor fixed the computed loss after adjustment of 50% loss
allowance. Sytengco demanded from respondent Transmodal the payment as compensation
for total loss of shipment; and on the same date, petitioner Equitable Insurance, as insurer of
the cargoes, paid Sytengco's claim. Sytengco then signed a subrogation receipt and loss receipt
in favor of petitioner Equitable Insurance.

Page 74 of 99
As such, petitioner demanded reimbursement of the payment given to Sytengco.
Thereafter, petitioner filed a complaint for damages invoking its right as subrogee after paying
Sytengco's insurance claim.
Respondent denied knowledge of an insurance policy and claimed that petitioner has no
cause of action against it because the damages to the cargoes were not due to its fault or gross
negligence.
RTC ruled in favor of the petitioner finding that petitioner Equitable Insurance was able
to prove by substantial evidence its right to institute an action as subrogee of Sytengco. CA
reversed the ruling of RTC stating that there was no proof of insurance of the cargoes at the
time of the loss and that the subrogation was improper. According to the CA, the insurance
contract was neither attached in the complaint nor offered in evidence for the perusal and
appreciation of the RTC, and what was presented was just the marine risk note.

ISSUE:
Whether or not petitioner‘s subrogation right is improper for its failure to prove that there exists a
contract of insurance covering the wet goods in question.

RULING:
No. A perusal of the records would show that petitioner is correct in its claim that the
marine insurance policy was offered as evidence; and constitutes as evidence before it may
recover the insured value of the lost/damaged cargo in the exercise of its subrogatory right.
Under the law, the principle of subrogation is a normal incident of indemnity property
insurance as a legal effect of payment; it inures to the insurer without any formal assignment or
any express stipulation to that effect in the policy. Said right is not dependent upon nor does it
grow out of any privity of contract. Payment to the insured makes the insurer an assignee in
equity. Subrogation is the substitution of one person in the place of another with reference to a
lawful claim or right, so that he who is substituted succeeds to the rights of the other in relation
to a debt or claim, including its remedies or securities.
The right of subrogation springs from Article 2207 of the Civil Code. In this case, it was
well established that petitioner has the right to step into the shoes of the insured who has a
direct cause of action against the respondent on account of the damages sustained by the
cargoes. The records further show that petitioner was able to accomplish its obligation under the
insurance policy as it has paid the assured of its insurance claim in the amount of P728,712,00
as evidenced by, among others, the Subrogation Receipt, Loss Receipt, Check Voucher, and
Equitable PCI Bank.
The payment by the insurer to the insured operates as an equitable assignment to the
insurer of all the remedies which the insured may have against the third party whose negligence
or wrongful act caused the loss. The right of subrogation is not dependent upon, nor does it
grow out of any privity of contract or upon payment by the insurance company of the insurance
claim. It accrues simply upon payment by the insurance company of the insurance claim.
Therefore, petitioner was able to present as evidence the marine open policy that vested
upon it, its rights as a subrogee. The court has ordered respondent Transmodal International,

Page 75 of 99
Inc., to pay Equitable Insurance Corp. the P728,712 insurance cost over its mishandling of a
shipment of gum Arabic that got damaged by water.

CASE NO. 57. 2100 CUSTOMS BROKERS, INC., VS. PHILAM INSURANCE COMPANY
[NOW AIG PHILIPPINES INSURANCE INC.], G.R. NO. 223377, JUNE 10, 2020, DECEMBER
1, 2020 – GACAYAN, HAZEL FAYE

FACTS:

On February 27, 2001, Ablestik Laboratories (Ablestik) placed two (2) cardboard boxes
containing 63 jars of Ablebond Adhesive on board Japan Airlines (JAL) Flight No. JL 5261 in
Los Angeles, California, United States of America covered by Airway Bill No. 131-66081842 for
consignee (TSPIC). After transshipment in Japan, the goods were expected to arrive in Manila
aboard JAL Flight No. JL 745 on March 1, 2001. Ablestik issued a handling instruction
addressed to its freight forwarding agent, U-Freight America Inc., stating the following:

“SHIPMENTS CONTAINING DRY ICE ARE

PERISHABLE AND MUST DELIVER TO OUR

CUSTOMER WITHIN 72 HOURS. DO NOT DELAY.”

xxx

5. Frozen products must maintain temperatures of -40F.

6. If transit is to be longer than 72 hours[,] total shipment must be


reiced [sic] in transit or at broker's import destination, depending
on flight schedule.

7. Shipment must be stored upon arrival in the destination of


broker's freezer with temperatures of 32F or colder.

The goods were insured with respondent Philam Insurance Company (Philam; now AIG
PHILIPPINES INSURANCE, INC.) against all risks per Marine Cargo Certificate 0801012154
and Open Policy Number 9595292.

At 1:30 a.m. on March 1, 2001 (Thursday), the goods arrived at the Ninoy Aquino
International Airport (NAIA) and were subsequently stored at the Paircargo warehouse located
in NAIA Complex, Parañaque City.

At 2:47 p.m. on March 2, 2001 (Friday), TSPIC notified 2100 CBI that the shipment had
arrived. TSPIC allegedly forwarded to 2100 CBI the Packing List from Ablestik indicating "1 Year
@-40C or colder/ Dry ice shipment" and the Shipment Handling Instructions from Ablestik
stating "SHIPMENTS CONTAINING DRY ICE ARE PERISHABLE AND MUST DELIVER TO
OUR CUSTOMER WITHIN 72 HOURS. DO NOT DELAY." It is further stated in the Shipment
Handling Instructions that:

―Frozen products must maintain temperatures of-40F.

Page 76 of 99
If transit is to be longer than 72 hours total shipment

must be re-iced in transit or at broker's import

destination, depending on flight schedule.‖

Shipment must be stored upon arrival in destination

broker's freezer with temperatures of 32F or colder.cral

TSPIC also sent an extra copy of Airway Bill No. 131-66081842 with "freight collect"
stamped on its face which meant that freight charges must be paid to JAL before it could
release the original copy of Airway Bill No. 131-66081842. This is required to process the
discharge of the shipment from the custody of the Bureau of Customs (BOC). TSPIC informed
2100 CBI that the latter will advance the necessary funds for the freight charges in the amount
of P14,672.00. Since it was already past 3 p.m. on a Friday, the banks were already closed, and
there were no available signatories to sign the checks. The freight charges were only settled on
March 5, 2001.

At around 2:00 a.m. on March 6, 2001 (Tuesday) or five (5) days after the date of arrival
of the shipment in Manila, 2100 CBI delivered the cargo to TSPIC. Upon receipt of the goods,
TSPIC's representatives found that the dry ice stuffed inside the boxes have melted due to the
delay in the delivery as shown in the Damage Report and photographs taken by the Manila
Adjusters Surveyors Company (MASCO).

TSPIC filed a claim against 2100 CBI for the value of the shipment but the latter refused
to pay. 2100 CBI contended that the delay in the delivery of the goods was due to TSPIC's
failure to give pre-alerts as to the expected arrival thereof and TSPIC's failure to pay the freight
charges on time.

TSPIC then filed a formal claim for the recovery of the value of the damaged goods
against Philam. After the survey conducted by the MASCO, payment in the amount of
P391,917.69 was recommended. Philam paid the insurance claim of TSPIC. On July 30, 2001,
a subrogation receipt for Claim No. 200140080A was executed certifying that Philam paid the
insurance claim of TSPIC.

Thereafter, Philam filed a claim for reimbursement against 2100 CBI but its claim was
denied. Hence, Philam filed a complaint for damages docketed as Civil Case No. 78072 in the
Metropolitan Trial Court of Makati City (MeTC).

On June 6, 2013, the MeTC ordered 2100 CBI to pay Philam the following: (1) P391,917.69
as actual damages; (2) P10,000.00 as attorney's fees; and (3) costs of suit. The MeTC held
that, as customs broker, 2100 CBI is regarded as a common carrier because transportation of
goods is an integral part of its business. It is mandated by law to exercise extraordinary
diligence in handling TSPIC's shipment.

On May 23, 2014, the Regional Trial Court (RTC) affirmed the ruling of the MeTC. In
sustaining the ruling of the MeTC, the RTC found that the cargo deteriorated while inside the
Paircargo warehouse because of the delay in the release and withdrawal to TSPIC, as stated in
the Certificate of Survey and Material Status Report.

Page 77 of 99
On October 12, 2015, the CA denied the petition of 2100 CBI and affirmed the ruling of the RTC
ordering 2100 CBI to pay P391,917.69 as actual damages, P10,000.00 as attorney's fees, and
costs of suit.

ISSUES:

(1) Whether the insurance policy must be presented to establish the liability of the common
carrier to Philam; and

(2) Whether 2100 CBI was negligent in handling the shipment of TSPIC, thus making it liable for
damages.

HELD:

(1) Rule 130, Section 3, of the Rules states:

Sec. 3. Original document must be produced; exceptions. -When the subject of


inquiry is the contents of a document, no evidence shall be admissible other than
the original document itself, except in the following cases:

(a) When the original has been lost or destroyed, or cannot be


produced in court, without bad faith on the part of the offeror;

(b) When the original is in custody or under the control of the party
against whom the evidence is offered, and the latter fails to
produce it after reasonable notice;

(c) When the original consists of numerous accounts or other


documents which cannot be examined in court without great loss
of time and the fact sought to be established from them is only the
general result of the whole; and

(d) When the original is a public record in the custody of a public


officer or is recorded in a public office.

The original copy of the insurance policy is the best proof of its contents. The contract of
insurance must be presented in evidence to indicate the extent of its coverage. At most, Marine
Cargo Certificate No. 0801012154 and the subrogation receipt may be used to establish the
relationship between the insurer and the consignee and the amount paid to settle the claim. The
subrogation receipt, by itself, is not sufficient to prove a claim holding an insurer liable for
damage sustained by an insured item. These documents are not sufficient to prove that the
damage to the cargo is compensable under the insurance policy chargeable against 2100 CBI.

In addition, Section 7, Rule 8 of the Rules provides:

Sec. 7. Action or defense based on document. - Whenever an action or defense


is based upon a written instrument or document, the substance of such
instrument or document shall be set forth in the pleading, and the original or a
copy thereof shall be attached to the pleading as an exhibit, which shall be
deemed to be a part of the pleading, or said copy may with like effect be set forth
in the pleading.

Page 78 of 99
As an actionable document, the insurance policy must be presented in order to determine
whether the damage sustained by the cargo of TSPIC is caused by a peril or risk covered by the
policy.

In the absence of proof of the contents of the policy confirming that the damage to the
cargo is covered by the insurance policy chargeable against 2100 CBI, Philam cannot hold 2100
CBI responsible for the damage to the cargo. Philam's failure to present the original copy, which
was presumably in its possession, or even a copy of it, for unknown reasons, is fatal to its claim
against 2100 CBI as this document is the primary basis for its claim of right to subrogation. Had
a copy of the insurance policy been presented by Philam, it would have clearly delineated the
scope of its coverage. We cannot ignore the possibility that the insurance policy did not cover all
phases of handling the shipment.

(2) Assuming arguendo that the risk or peril that caused the damage to the cargo is covered by
the insurance policy, We find that 2100 CBI was not negligent in handling the shipment of
TSPIC.

It must be pointed out that the arrangement for the payment of the freight charges is on
a "Freight Collect" basis which means that the consignee or receiver of the goods will be
responsible for paying the freight and other charges in the total amount of P14,672.00.

It is clear that there is no need to rely on the presumption of the law that a common carrier
is presumed to have been at fault or have acted negligently in case of damaged goods. This is
because the delay in the release of the goods was through no fault of 2100 CBI. The damage
was caused by the late payment of the funds needed for the release of the goods from the
custody of BOC which was originally TSPIC's responsibility. It must be noted that while waiting
for the freight charges to be settled, 2100 CBI did not have custody over the shipment.

The pro-forma stipulation in DR No. 659556 that TSPIC received the cargo in good order
and condition from 2100 CBI does not disprove the claim of 2100 CBI that the cargo may have
been damaged while it was in the possession of BOC. It is important to note that at the time the
cargo was released to 2100 CBI from BOC and delivered to TSPIC, the cargo remained sealed.
Thus, said pro-forma stipulation did not accurately describe the condition of the cargo at the
time delivery was made to TSPIC and cannot be used as basis for holding 2100 CBI
accountable for the damaged goods.

It is clear that the only handling instruction 2100 CBI received was to "PLS. PUT INTO
COOL ROOM UPON ARRIVAL," which was stated in Airway Bill No. 131-66081842. 2100 CBI
could not have undertaken precautionary measures nor implement handling instructions
because it did not have possession of the cargo until 2:00 a.m. of March 6, 2001 - when the
goods were released by the BOC. It must be emphasized that, until the freight charges are paid,
JAL cannot release the original copy of Airway Bill No. 131-66081842 and the goods to 2100
CBI. Payment of the freight charges is required to process the release of the goods in the
custody of the BOC. At 2:47 p.m. on March 2, 2001, 2100 CBI only received a duplicate copy of
Airway Bill No. 131-66081842.Therefore, without the original copy of the Airway Bill No. 131-
66081842, the goods remained in the possession of the BOC and were not released to 2100
CBI.

Moreover, 2100 CBI may only be expected to implement the handling instructions when
the shipment was in the Paircargo warehouse which was under the control of the BOC. It would
be physically impossible and unreasonable for 2100 CBI to implement any control or handling

Page 79 of 99
instructions over goods not in its custody. Based on the evidence presented, Philam failed to
establish that negligence in the handling of the shipment could be attributed to 2100 CBI from
the time the BOC released the goods to the custody of 2100 CBI at 2:00 a.m. on March 6, 2001
until they were delivered to TSPIC in good order and condition at 3:44 a.m. on March 6, 2001.

Accordingly, as an insurer who pays the insured for loss or liability not proven to be
compensable under the subject policy, Philam is not subrogated to the rights of TSPIC.

CASE NO. 58. ISABELA ROQUE, DOING BUSINES UNDER THE NAME AND STYLE OF
ISABELA ROQUE TIMBER ENTERPRISES AND ONG CHIONG V. HON. INTERMEDIATE
APPELATE COURT AND PIONEER INSURANCE AND SURETY CORPORATION, G.R. NO.
L-66935, NOVEMBER 11, 1985 – CABALO, CASHMIR

FACTS:

On February 19, 1972 Manila Bay Lighterage Corporation (Manila Bay), a common carrier,
entered into a contract with the petitioners whereby the former would load and carry on board its
barge Mable 10 about 422.18 cubic meters of logs from Malampaya Sound, Palawan to North
Harbor, Manila. Moreover the petitioners – Isabela Roque insured the logs against loss for
P100, 000.00 with respondent Pioneer Insurance and Surety Corporation.

On February 29, 1972 Isabela Roque loaded on the barge, but the shipment never reached its
destination because Mable 10 sank with the 811 pieces of logs somewhere off Cabuli Point in
Palawan on its way to Manila.

On March 8, 1972 the petitioners wrote a letter to Manila Bay demanding payment of P150,
000.00 for the loss of the shipment plus P100, 000.00 as unrealized profits but the latter ignored
the demand. Another letter was sent to respondent Pioneer claiming the full amount of P100,
000.00 under the insurance policy but respondent refused to pay. Therefore, petitioners
commenced Civil Case No. 86599 against Manila Bay and respondent Pioneer.

The Trial Court ruled in favor of the petitioners and the respondent Pioneer appealed, while
Manila Bay already rest its case as the transportation company is no longer doing business and
is already without funds. The appellate court modified the trial court's decision and absolved
Pioneer from liability after finding that there was a breach of implied warranty of seaworthiness
on the part of the petitioners and that the loss of the insured cargo was caused by the "perils of
the ship" and not by the "perils of the sea".

It was then ruled that the loss is NOT covered by the marine insurance policy.

ISSUE: Whether or not the loss is covered by the marine insurance policy.

RULING:

The court ruled in the negative. The petitioners state that a mere shipper of cargo, having no
control over the ship, has nothing to do with its seaworthiness. This argument has no merit. The
liability of the insurance company is governed by law.

Section 113 of the Insurance Code provides: In every marine insurance upon a ship or freight,
or freightage, or upon anything which is the subject of marine insurance, a warranty is implied
that the ship is seaworthy.

Page 80 of 99
Section 99 of the same Code also provides in part. Marine insurance includes: (1) Insurance
against loss of or damage to: (a) Vessels, craft, aircraft, vehicles, goods, freights, cargoes,
merchandise, ...

From the above-quoted provisions, there can be no mistaking the fact that the term "cargo" can
be the subject of marine insurance and that once it is so made, the implied warranty of
seaworthiness immediately attaches to whoever is insuring the cargo whether he be the ship-
owner or not. The fact that the unseaworthiness of the ship was unknown to the insured is
immaterial in ordinary marine insurance and may not be used by him as a defense in order to
recover on the marine insurance policy. Since the law provides for an implied warranty of
seaworthiness in every contract of ordinary marine insurance, it becomes the obligation of a
cargo owner to look for a reliable common carrier which keeps its vessels in seaworthy
condition. The shipper of cargo may have no control over the vessel but he has full control in the
choice of the common carrier that will transport his goods. Or the cargo owner may enter into a
contract of insurance which specifically provides that the insurer answers not only for the perils
of the sea but also provides for coverage of perils of the ship

CASE NO. 59. LA RAZON SOCIAL "GO TIAOCO Y HERMANOS VS. UNION INSURANCE
SOCIETY OF CANTON, LTD., G.R. NO. 13983, SEPTEMBER 1, 1919 – PASCUA, MA.
BETINA GRACE

Facts:

This is an action on a policy of marine insurance issued by the Union Insurance Society
of Canton, Ltd., upon a cargo of rice belonging to the plaintiffs, Go Tiaoco Brothers, which was
transported on the steamship Hondagua from the port of Saigon to Cebu.

It was discovered that one thousand four hundred seventy-three sacks of rice had been
damaged by sea water while being discharged from one of the compartments.

The trial court found that the inflow of the sea water during the voyage was due to a
defect in one of the drain pipes of the ship and concluded that the loss was not covered by the
policy of insurance.

The court found in effect that the opening above described had resulted in course of time
from ordinary wear and tear and not from the straining of the ship in rough weather on that
voyage. It was also found that the repairs that had been made on the pipe were slovenly and
defective and that, by reason of the condition of this pipe, the ship was not properly equipped to
receive the rice at the time the voyage was begun. For this reason the court held that the ship
was unseaworthy.

Issue:

Whether the insurer is liable to the plaintiff due to the loss.

Ruling:

No. Maritime insurance purports to insure the cargo from the following among other
risks: "Perils of the seas, men of war, fire, enemies, pirates, rovers, thieves, jettisons, barratry of
the master and mariners, and of all other perils, losses, and misfortunes that have or shall come
to the hurt, detriment, or damage of the said goods and merchandise or any part thereof."

Page 81 of 99
A loss which, in the ordinary course of events, results from the natural and inevitable
action of the sea, from the ordinary wear and tear of the ship, or from the negligent failure of the
ship's owner to provide the vessel with proper equipment to convey the cargo under ordinary
conditions, is considered peril of the ship rather than a peril of the sea.

In essence, the insurer undertakes to insure against perils of the sea and similar
perils, but not against perils of the ship.

CASE NO. 60. CALTEX, INC. VS. SULPICIO LINES, 315 SCRA 709/GR NO. 131166,
SEPTEMBER 30, 1999 – LAUSAN, PENNELOPE

FACTS:

On December 19, 1987, motor tanker MT Vector, owned and operated by Vector
Shipping Corporation, engaged in the business of transporting fuel products such as gasoline,
kerosene, diesel and crude oil, traveled enroute to Masbate. During that particular voyage, the
MT Vector carried on board 8,800 barrels of petroleum products shipped by Caltex by virtue of a
charter contract between them. On December 20, 1987, the passenger ship MV Doña Paz,
owned and operated by Sulpicio Lines, Inc. (Sulpicio), left the port of Tacloban headed for
Manila with a complement of 59 crew members including the master and his officers, and
passengers totaling 1,493 as indicated in the Coast Guard Clearance.

At about 10:30 p.m. of December 20, 1987, the two vessels collided in the open sea. All
the crewmembers of MV Doña Paz died, while the two survivors from MT Vector claimed that
they were sleeping at the time of the incident. The MV Doña Paz carried an estimated 4,000
passengers; many indeed, were not in the passenger manifest. Only 24 survived the tragedy
after having been rescued. On March 22, 1988, the board of marine inquiry after investigation
found that the MT Vector, its registered operator Soriano, and its owner and actual operator
Vector Shipping Corporation, were at fault and responsible for its collision with MV Doña Paz.

Victim‘s relatives filed with the Regional Trial Court a complaint for ―Damages Arising
from Breach of Contract of Carriage‖ against Sulpicio. Sulpicio, in turn, filed a third party
complaint against Soriano, Vector Shipping Corporation and Caltex (Philippines), Inc. Sulpicio
alleged that Caltex chartered MT Vector with gross and evident bad faith knowing fully well that
MT Vector was improperly manned, ill-equipped, unseaworthy and a hazard to safe navigation;
as a result, it rammed against MV Doña Paz in the open sea setting MT Vector‘s highly
flammable cargo ablaze.

ISSUE:

Whether the charterer/ shipper is liable for breach of warranty of seaworthiness.

HELD:

NO, the charterer has no liability for damages under Philippine Maritime laws.

Page 82 of 99
Clatex and Vector entered into a contract of affreightment, also known as a voyage
charter. A charter party is a contract by which an entire ship, or some principal part thereof, is let
by the owner to another person for a specified time or use; a contract of affreightment is one by
which the owner of a ship or other vessel lets the whole or part of her to a merchant or other
person for the conveyance of goods, on a particular voyage, in consideration of the payment of
freight. The charter-party provides for the hire of the vessel only, either for a determinate period
of time or for a single or consecutive voyage, the ship owner to supply the ship‘s store, pay for
the wages of the master of the crew, and defray the expenses for the maintenance of the ship. If
the charter is a contract of affreightment, which leaves the general owner in possession of the
ship as owner for the voyage, the rights and the responsibilities of ownership rest on the owner.
The charterer is free from liability to third persons in respect of the ship.

Carriers of goods by sea are deemed to warrant impliedly the seaworthiness of the ship.
For a vessel to be seaworthy, it must be adequately equipped for the voyage and manned with
a sufficient number of competent officers and crew. The failure of a common carrier to maintain
in seaworthy condition the vessel involved in its contract of carriage is a clear breach of its duty
prescribed in Article 1755 of the Civil Code. The charterer of a vessel has no obligation before
transporting its cargo to ensure that the vessel it chartered complied with all legal requirements.
The duty rests upon the common carrier simply for being engaged in "public service." The
relationship between the parties in this case is governed by special laws. Because of the implied
warranty of seaworthiness, shippers of goods, when transacting with common carriers, are not
expected to inquire into the vessel‘s seaworthiness, genuineness of its licenses and compliance
with all maritime laws. To demand more from shippers and hold them liable in case of failure
exhibits nothing but the futility of our maritime laws insofar as the protection of the public in
general is concerned.

CASE NO. 61. DELSAN TRANPORT LINES, INC., VS CA AND AMERICAN HOME
ASSURANCE CORPORATION, G.R. NO. 127897 NOVEMBER 15, 2001 – PUA, JOMARIE
CASSANDRA

Facts:

Caltex Philippines entered into a contract of affreightment with the petitioner, Delsan
Transport Lines, Inc., for a period of 1 year whereby the said common carrier agreed to
transport Caltex‘s industrial fuel oil from the Batangas-Bataan Refinery to different parts of the
country. Under the contract, petitioner took on board its vessel, MT Maysun, industrial fuel oil of
Caltex to be delivered to the Caltex Oil Terminal in Zamboanga City.

The shipment was insured with the private respondent, American Home Assurance
Corporation. The vessel sank taking with it the entire cargo of fuel oil. Private respondent paid
Caltex the sum of P5,096,635.57 representing the insured value of the lost cargo. Exercising its
right of subrogation the private respondent demanded of the petitioner the same amount it paid
to Caltex. Due to its failure to collect from the petitioner despite prior demand, private
respondent filed a complaint with the RTC for collection of a sum of money.

Page 83 of 99
The trial court dismissed the complaint against the petitioner. The trial court found that
the vessel, MT Maysun, was seaworthy to undertake the voyage as determined by the
Philippine Coast Guard per Survey Certificate Report No. M5-016-MH upon inspection during its
annual dry-docking and that the incident was caused by unexpected inclement weather
condition or force majeure, thus exempting the common carrier (herein petitioner) from liability
for the loss of its cargo.

The decision of the trial court was reversed by the Court of Appeals. In the absence of
any explanation as to what may have caused the sinking of the vessel coupled with the finding
that the same was improperly manned, the appellate court ruled that the petitioner is liable on its
obligation as common carrier to herein private respondent insurance company as subrogee of
Caltex.

Issue:

Whether the payment made by the private respondent to Caltex for the insured value of
the lost cargo amounted to an admission that the vessel was seaworthy, thus precluding any
action for recovery against the petitioner.

Held:
The payment made by the private respondent for the insured value of the lost cargo
operates as waiver of its right to enforce the term of the implied warranty against Caltex under
the marine insurance policy. However, the same cannot be validly interpreted as an automatic
admission of the vessel‘s seaworthiness by the private respondent as to foreclose recourse
against the petitioner for any liability under its contractual obligation as a common carrier. The
fact of payment grants the private respondent subrogatory right which enables it to exercise
legal remedies that would otherwise be available to Caltex as owner of the lost cargo against
the petitioner common carrier.

The right of subrogation has its roots in equity. It is designed to promote and to
accomplish justice and is the mode which equity adopts to compel the ultimate payment of a
debt by one who in justice and good conscience ought to pay. It is not dependent upon, nor
does it grow out of, any privity of contract or upon written assignment of claim. It accrues simply
upon payment by the insurance company of the insurance claim. Consequently, the payment
made by the private respondent (insurer) to Caltex (assured) operates as an equitable
assignment to the former of all the remedies which the latter may have against the petitioner.

Neither may petitioner escape liability by presenting in evidence certificates that tend to
show that at the time of dry-docking and inspection by the Philippine Coast Guard, the vessel
MT Maysun, was fit for voyage. These pieces of evidence do not necessarily take into account

Page 84 of 99
the actual condition of the vessel at the time of the commencement of the voyage. As correctly
observed by the Court of appeals:

At the time of dry-docking and inspection, the ship may have appeared fit. The
certificates issued, however, do not negate the presumption of unseaworthiness triggered by an
unexplained sinking. Of certificates issued in this regard, authorities are likewise clear as to their
probative value. Seaworthiness relates to a vessel‘s actual condition. Neither the granting of
classification or the issuance of certificates establishes seaworthiness.

CASE NO. 62. DEVELOPMENT INSURANCE CORPORATION VS. INTERMEDIATE


APPELLATE COURT, 143 SCRA 62, JUNE 16, 1986 – DELA CRUZ, JEREMY KIMBERLY

FACTS:
A fire occurred in the building of the private respondent and it sued for recovery of
damages from the petitioner on the basis of an insurance contract between them. The petitioner
allegedly failed to answer on time and was declared in default by the trial court. A judgment of
default was subsequently rendered on the strength of the evidence submitted ex parte by the
private respondent, which was allowed full recovery of its claimed damages. On learning of this
decision, the petitioner moved to lift the order of default, invoking excusable neglect, and to
vacate the judgment by default. Its motion was denied. It then went to the respondent court,
which affirmed the decision of the trial court in toto. Hence, the petition.

ISSUE:
Whether the lower court erred in its decision.

HELD:

NO. The petitioner's claim that the insurance covered only the building and not the
elevators is absurd, to say the least. This Court has little patience with puerile arguments that
affront common sense, let alone basic legal principles with which even law students are familiar.
The circumstance that the building insured is seven stories high and so had to be provided with
elevators- a legal requirement known to the petitioner as an insurance company-makes its
contention all the more ridiculous.

No less preposterous is the petitioner's claim that the elevators were insured after the
occurrence of the fire, a case of shutting the barn door after the horse had escaped, so to
speak. This pretense merits scant attention. Equally undeserving of serious consideration is its
submission that the elevators were not damaged by the fire, against the report of The arson
investigators of the INP and, indeed, its own expressed admission in its answer6 where it
affirmed that the fire "damaged or destroyed a portion of the 7th floor of the insured building and
more particularly a Hitachi elevator control panel."

The petitioner argues that since at the time of the fire the building insured was worth
P5,800,000.00, the private respondent should be considered its own insurer for the difference
between that amount and the face value of the policy and should share pro rata in the loss
sustained. Accordingly, the private respondent is entitled to an indemnity of only P67,629.31,
the rest of the loss to be shouldered by it alone. In support of this contention, the petitioner cites
Condition 17 of the policy, which provides:

Page 85 of 99
If the property hereby insured shall, at the breaking out of any fire, be collectively of
greater value than the sum insured thereon then the insured shall be considered as being his
own insurer for the difference, and shall bear a ratable proportion of the loss accordingly. Every
item, if more than one, of the policy shall be separately subject to this condition.

However, there is no evidence on record that the building was worth P5,800,000.00 at
the time of the loss; only the petitioner says so and it does not back up its self-serving estimate
with any independent corroboration. On the contrary, the building was insured at P2,500,000.00,
and this must be considered, by agreement of the insurer and the insured, the actual value of
the property insured on the day the fire occurred. This valuation becomes even more believable
if it is remembered that at the time the building was burned it was still under construction and
not yet completed.

As defined in the aforestated provision, which is now Section 60 of the Insurance Code,
"an open policy is one in which the value of the thing insured is not agreed upon but is left to be
ascertained in case of loss. " This means that the actual loss, as determined, will represent the
total indemnity due the insured from the insurer except only that the total indemnity shall not
exceed the face value of the policy.

The actual loss has been ascertained in this case and, to repeat, this Court will respect
such factual determination in the absence of proof that it was arrived at arbitrarily. There is no
such showing. Hence, applying the open policy clause as expressly agreed upon by the parties
in their contract, we hold that the private respondent is entitled to the payment of indemnity
under the said contract in the total amount of P508,867.00.

CASE NO. 63. NOEL F. MANANKIL, LIBERATO P LAUS, GLORIA C. MAGTOTO,


EVANGELINE G. TEJADA, ALIZAIDO F. PARAS AND PHILIP JOSE B. PANLILIO, VS.
COMMISSION ON AUDIT, G.R. NO. 217342, OCTOBER 13, 2020 – RAGMA, MYKA MARIE
KHRISTINE

FACTS:

Clark Development Corporation (CDC) is a subsidiary corporation of the Bases


Conversion and Development Authority (BCDA). It was established through Executive Order
No. (EO) 804 in 1993. The CDC is empowered by law to "make contracts, lease, own or
otherwise dispose of personal and real property; sue and be sued; and otherwise do and
perform any and all things that may be necessary or proper" to carry out the BCDA's purpose
and objectives.

In 1995, DC entered into a 25-year Lease Agreement with Amari Duty Free, Inc. (Amari)
to rent out a 1.70-hectare parcel of land located along Dyess Highway, CSEZ, Pampanga
(leased property). Amari shall use the leased property for its "duty free store/commercial
shopping" and "fast food/cafeteria" operations.

In their Lease Agreement, it is expressly agreed and understood that the insurance
coverage herein stipulated shall be secured from the Government Service Insurance System
(GSIS) only not later than two (2) months after construction/rehabilitation of facilities, in which
the LESSOR shall be designated as its beneficiary.

However, for moveable properties, insurance coverage may be secured from any
insurance company duly authorized by the lessor. It is further agreed, that in case of loss or

Page 86 of 99
damage to the Leased Property during the term of this Contract, the lessor shall reconstruct or
restore the lost or damaged property to its original condition using the proceeds from the
insurance for the continued lease and use by the lessee.

In the event that the insurance proceeds are insufficient for purpose of reconstruction or
restoration as herein required, then LESSEE shall provide the necessary funds to augment the
insurance proceeds.

A fire in 2005 razed the structure of Amari, which changed its name to Grand Duty Free,
and was forced to close their business and stop their operations. During further negotiations,
Grand Duty Free wanted to pre terminate the contract. CDC refuses, on the ground that their
lease contract is actually an insurance contract.

ISSUE: Whether the Insurance Code applies in the Lease Agreement between CDC and
Amari/Grand Duty Free.

HELD:

No. The Insurance Code does not apply.

Verily, the Insurance Code states that the proceeds shall be applied to the designated
recipient's exclusive benefit. However, once the insurer releases the proceeds in full to the
designated recipient, the obligations under a contract of insurance will have been fully
performed and, thus, extinguished. Upon such time, a contract of insurance's terms and the
Insurance Code's provisions may no longer control the manner by which the proceeds are
thereafter used or otherwise disposed of.

To recall, the CDC's 50% Release was pursuant to the terms and conditions attached to
the Lease Agreement's pre termination. Thus, the payment and its possible disallowance must
be evaluated based on these terms and conditions, not on the insurance contract or the
Insurance Code.

As earlier discussed, the CDC merely held the insurance proceeds in trust, in view of its
impending duty to rebuild the original structure. To recall, when fire razed the original structure
on December 29, 2005, 10 years into the lease's 25-year term, Grand Duty Free remained to
have full ownership over the property.

In addition, Grand Duty Free: (1) obtained the insurance to protect its property (the
original structure, and not the land on which it stood) against damage caused by fire, and (2)
paid all the required premiums. It is clear that the government did not contribute any capital to
obtain the insurance. In other words, the government collected from an insurance policy
constituted over private property, the premium payments for which it did not even fund.

CASE NO. 64. FIRST INTEGRATED BONDING & INSURANCE CO., INC. VS. HERNANDO,
ET. AL, G.R. NO. L-51221, JULY 31, 1991 – ALPINE CHIWERAN

FACTS:

Silverio Blanco was the owner of a passenger jeepney which he insured against liabilities for
death and injuries to third persons with First Integrated Bonding and Insurance Company, Inc.
(First Insurance) under Motor Vehicle Policy No. V-05-63751 with the face value of P30,000.00.

Page 87 of 99
On November 25, 1976, the said jeepney driven by Blanco himself bumped a five-year old child,
Deogracias Advincula, causing the latter‘s death.

A complaint for damages was brought by the child‘s parents, the Advincula spouses, against
Silverio Blanco. First Insurance was also impleaded in the complaint as the insurer. Summons
were served on Silverio Blanco and First Insurance. However, only Blanco filed an answer.
Upon motion of the Advincula spouses, First Insurance was declared in default (p. 45, Rollo) on
January 19, 1978.

The insured argued that the injured have no cause of action against the petitioner for not being
a party to the contract of insurance.

ISSUE:

Whether First Insurance is laible.

HELD:

Yes.

It is settled that where the insurance contract provides for indemnity against liability to a third
party, such third party can directly sue the insurer (Caguia vs. Fieldman‘s Insurance Co., Inc.
G.R. No. 23276, November 29, 1968, 26 SCRA 178). The liability of the insurer to such third
person is based on contract while the liability of the insured to the third party is based on tort
(Malayan Insurance Co., Inc. vs. CA, L-36413, September 26, 1988, 165 SCRA 536).

This rule was explained in the case of Shafer vs. Judge, RTC of Olongapo City, which provides
that the injured for whom the contract of insurance is intended can sue directly the insurer. The
general purpose of statutes enabling an injured person to proceed directly against the insurer is
to protect injured persons against the insolvency of the insured who causes such injury, and to
give such injured person a certain beneficial interest in the proceeds of the policy, and statutes
are to be liberally construed so that their intended purpose may be accomplished. Not even a
‗no action‘ clause under the policy which requires that a final judgment be first obtained against
the insured and that only thereafter can the person insured recover on the policy can prevail
over the Rules of Court provisions aimed at avoiding multiplicity of suits.

First Insurance cannot evade its liability as insurer by hiding under the cloak of the insured. Its
liability is primary and not dependent on the recovery of judgment from the insured.

CASE NO. 65. GIUNGON V DEL MONTE, 20 SCRA 1043 – SOMERA, MARIA MONICA

FACTS:

Julio Aguilar, owner and operator of several jeepneys, insured them with Capital
Insurance & Surety Co., Inc. On February 20, 1961, along the intersection of Juan Luna and
Moro streets, City of Manila, the jeepneys operated by Aguilar driven by Iluminado del Monte
and Gervacio Guingon bumped and Guingon died some days after. Iluminado del Monte was
charged with homicide thru reckless imprudence and was penalized 4 months imprisonment.

The heirs of Gervacio Guingon filed an action for damages praying that P82,771.80 be
paid to them jointly and severally by the driver del Monte, owner and operator Aguilar, and the
Capital Insurance & Surety Co., Inc.

Page 88 of 99
CFI: Iluminado del Monte and Julio Aguilar jointly and severally to pay plaintiffs the sum
of P8,572.95 as damages for the death of their father, plus P1,000.00 for attorney's fees plus
costs. Capital Insurance and Surety Co., Inc. is hereby sentenced to pay P5,000 plus P500 as
attorney's fees and costs to be applied in partial satisfaction of the judgment rendered against
Iluminado del Monte and Julio Aguilar in this case.

ISSUES:
1) Whether the plaintiffs can sue the insurer. (Yes)
2) Whether the heirs can sue the insurer jointly with the insured. (Yes)

RULING:

1) The policy in the present case, as aforequoted, is one whereby the insurer agreed to
indemnify the insured "against all sums . . . which the Insured shall become legally liable to pay
in respect of: a. death of or bodily injury to any person . . . ." Clearly, therefore, it is one for
indemnity against liability; from the fact then that the insured is liable to the third person, such
third person is entitled to sue the insurer.

The right of the person injured to sue the insurer of the party at fault (insured), depends
on whether the contract of insurance is intended to benefit third persons also or only the
insured. And the test applied has been this: Where the contract provides for indemnity against
liability to third persons, then third persons to whom the insured is liable, can sue the insurer.
Where the contract is for indemnity against actual loss or payment, then third persons cannot
proceed against the insurer, the contract being solely to reimburse the insured for liability
actually discharged by him thru payment to third persons, said third persons' recourse being
thus limited to the insured alone.

2) The policy requires, as afore-stated, that suit and final judgment be first obtained against the
insured; that only "thereafter" can the person injured recover on the policy; it expressly disallows
suing the insurer as a co-defendant of the insured in a suit to determine the latter's liability.

The "no action" clause in the policy of insurance cannot prevail over the Rules of Court
provision aimed at avoiding multiplicity of suits.

Here, Sec. 5 of Rule 2 on "Joinder of causes of action" and Sec. 6 of Rule 3 on


"Permissive joinder of parties" cannot be superseded, at least with respect to third persons not a
party to the contract, as herein, by a "no action" clause in the contract of insurance.

CASE NO. 66. SHAFER V. HON. JUDGE, RTC OF OLONGAPO CITY, G.R. NO. 78848,
NOVEMBER 14, 1988 – CHAWI, DECIMAY

FACTS:

Petitioner Sherman Shafer obtained a private car policy, GA No. 0889, over his Ford
Laser car from Makati Insurance Company, Inc., for third party liability (TPL).<äre||anº•1àw>
During the effectivity of the policy, an information for reckless imprudence resulting in damage
to property and serious physical injuries was filed against petitioner.

Page 89 of 99
The owner of the damaged Volkswagen car filed a separate civil action against petitioner
for damages, while Jovencio Poblete, Sr., who was a passenger in the Volkswagen car when
allegedly hit and bumped by the car driven by petitioner, did not reserve his right to file a
separate civil action for damages. Instead, in the course of the trial in the criminal case, Poblete,
Sr. testified on his claim for damages for the serious physical injuries which he claimed to have
sustained as a result of the accident.

ISSUE: Whether the accused, also the third party plaintiff, has a cause of action against the
third party defendant for the enforcement of its third party liability (TPL) under the insurance
contract.

HELD:

Compulsory Motor Vehicle Liability Insurance (third party liability, or TPL) is primarily
intended to provide compensation for the death or bodily injuries suffered by innocent third
parties or passengers as a result of a negligent operation and use of motor vehicles. The victims
and/or their dependents are assured of immediate financial assistance, regardless of the
financial capacity of motor vehicle owners.

The liability of the insurance company under the Compulsory Motor Vehicle Liability
Insurance is for loss or damage. Where an insurance policy insures directly against liability, the
insurer's liability accrues immediately upon the occurrence of the injury or event upon which the
liability depends, and does not depend on the recovery of judgment by the injured party against
the insured.

The injured for whom the contract of insurance is intended can sue directly the insurer.
The general purpose of statutes enabling an injured person to proceed directly against the
insurer is to protect injured persons against the insolvency of the insured who causes such
injury, and to give such injured person a certain beneficial interest in the proceeds of the policy,
and statutes are to be liberally construed so that their intended purpose may be accomplished.
It has even been held that such a provision creates a contractual relation which inures to the
benefit of any and every person who may be negligently injured by the named insured as if such
injured person were specifically named in the policy.

In the event that the injured fails or refuses to include the insurer as party defendant in
his claim for indemnity against the insured, the latter is not prevented by law to avail of the
procedural rules intended to avoid multiplicity of suits. Not even a "no action" clause under the
policy-which requires that a final judgment be first obtained against the insured and that only
thereafter can the person insured recover on the policy can prevail over the Rules of Court
provisions aimed at avoiding multiplicity of suits.

There is no need on the part of the insured to wait for the decision of the trial court
finding him guilty of reckless imprudence. The occurrence of the injury to the third party
immediately gave rise to the liability of the insurer under its policy.

Page 90 of 99
The claim of petitioner for payment of indemnity to the injured third party, under the
insurance policy, for the alleged bodily injuries caused to said third party, arose from the offense
charged in the criminal case, from which the injured (Jovencio Poblete, Sr.) has sought to
recover civil damages. Hence, such claim of petitioner against the insurance company cannot
be regarded as not related to the criminal action.

CASE NO. 67. TRAVELLER’S INSURANCE AND SURETY CORP. VS. CA AND VICENTE
MENDOZA, GR 82036, MAY 22. 1997 – ABRIGO, KAREN

FACTS:

Feliza Vineza de Mendoza, a 78-year old woman was on her way to hear mass at the
Tayuman Cathedral. While walking along Tayuman, she was bumped by a taxi that was running
fast. Several persons witnessed the accident, among whom were Rolando Marvilla, Ernesto
Lopez and Eulogio Tabalno. After the bumping, the old woman was seen sprawled on the
pavement. Marvilla, who was at the scene ran towards the old woman and held her on his lap to
inquire from her what had happened, but Feliza was already in shock and could not talk. At this
moment, a private jeep stopped. Marvilla and the jeepney driver helped board the old woman on
the jeep and brought her to the Mary Johnston Hospital in Tondo, and after which, she was
brought to the U.S.T. Hospital where she died due to a traumatic shock as a result of the severe
injuries she sustained. Private respondent filed a complaint for damages against Armando
Abellon as the owner of the Lady Love Taxi and Rodrigo Dumlao as the driver of the Lady Love
taxicab that bumped private respondent's mother. Subsequently, private respondent amended
his complaint to include Travellers Insurance as the compulsory insurer of the said taxicab.

ISSUE:

Whether Travellers Insurance can be held solidarily liable to pay private respondent
Mendoza under an insurance contract.

RTC Ruling: The trial court in it‘s decision held Travellers Insurance to be solidarily
liable against private respondent with the taxicab driver and operator.

CA Ruling: Petitioner appealed the decision to the CA. CA affirmed RTC decision.
Hence, this petition.

Petitioner contends that it did not issue an insurance policy as compulsory


insurer of the Lady Love Taxi and that, assuming arguendo that it had indeed covered
said taxicab for third-party liability insurance, private respondent failed to file a written
notice of claim with petitioner as required by Section 384 of P.D. No. 612, otherwise
known as the Insurance Code.

RULING:

NO. Travellers Insurance cannot be held solidarily liable to pay private respondent
Mendoza under an insurance contract.

The right of the person injured to sue the insurer of the party at fault (insured), depends
on whether the contract of insurance is intended to benefit third persons also or on the insured.
And the test applied has been this: Where the contract provides for indemnity against liability to
third persons, then third persons to whom the insured is liable can sue the insurer. Where the

Page 91 of 99
contract is for indemnity against actual loss or payment, then third persons cannot proceed
against the insurer, the contract being solely to reimburse the insured for liability actually
discharged by him thru payment to third persons, said third person‘s recourse being thus limited
to the insured alone.

While it is true that where the insurance contract provides for indemnity against liability
to third persons, such third persons can directly sue the insurer, however, the direct liability of
the insurer under indemnity contracts against third party liability does not mean that the insurer
can be held solidarily liable with the insured and/or the other parties found at fault. The liability
of the insurer is based on contract; that of the insured is based on tort.

We have certainly ruled with consistency that the prescriptive period to bring suit in court
under an insurance policy, begins to run from the date of the insurer's rejection of the claim filed
by the insured, the beneficiary or any person claiming under an insurance contract. This ruling
is premised upon the compliance by the persons suing under an insurance contract, with
the indispensable requirement of having filed the written claim mandated by Section 384
of the insurance Code before and after its amendment. Absent such written claim filed by
the person suing under an insurance contract, no cause of action accrues under such
insurance contract, considering that it is the rejection of that claim that triggers the
running of the one-year prescriptive period to bring suit in court, and there can be no
opportunity for the insurer to even reject a claim if none has been filed in the first place,
as in the instant case.

When petitioner asseverates, thus, that no written claim was filed by private respondent
and rejected by petitioner, and private respondent does not dispute such asseveration through a
denial in his pleadings, we are constrained to rule that respondent appellate court committed
reversible error in finding petitioner liable under an insurance contract the existence of which
had not at all been proven in court. Even if there were such a contract, private respondent's
cause of action cannot prevail because he failed to file the written claim mandated by
Section 384 of the Insurance Code. He is deemed, under this legal provision, to have
waived his rights as against petitioner-insurer.

CASE NO. 68. JEWEL VILLACORTA VS. THE INSURANCE COMMISSION AND EMPIRE
INSURANCE COMPANY, G.R. NO. L-54171, OCTOBER 28, 1980 – ANTONIO, PHOEBE
ANTONETTE

FACTS:

Petitioner was the owner of a Colt Lancer car insured with respondent company for
P35,000 — Own Damage; P30,000 — Theft; and P30,000 — Third Party Liability. While the
vehicle was in the custody of the Sunday Machine Works, for general check-up and repairs, the
car was allegedly taken by 6 persons and driven out to Montalban, Rizal. While travelling, the
car figured in an accident, hitting and bumping a gravel and sand truck parked at the right side
of the road. Complainant, thereafter, filed a claim for total loss with the respondent company but
claim was denied. Hence, complainant was compelled to institute the present action.

Respondent insurance commission dismissed petitioner's complaint for recovery of the


total loss of the vehicle against private respondent, sustaining respondent insurer's contention
that the accident did not fall within the provisions of the policy either for the Own Damage or
Theft coverage, invoking the policy provision on "Authorized Driver" clause. Respondent
commission upheld private respondent's contention on the "Authorized Driver" clause in this
wise: ―it must be observed that under the above-quoted provisions, the policy limits the use of

Page 92 of 99
the insured vehicle to 2 persons only, namely: the insured himself or any person on his
(insured's) permission.‖ Respondent commission likewise upheld private respondent's assertion
that the car was not stolen and therefore not covered by the Theft clause. According to
respondent commission, the fact that the car was taken by one of the residents of the Sunday
Machine Works, and the withholding of the same, for a joy ride should not be construed to mean
'taking' under Art. 308 of the RPC.

ISSUE:

Whether the ―authorized driver‖ clause bars petitioner from claiming against private
respondent. NO

RULING:

The main purpose of the "authorized driver" clause, is that a person other than the
insured owner, who drives the car on the insured's order, such as his regular driver, or with his
permission, such as a friend or member of the family or the employees of a car service or repair
shop must be duly licensed drivers and have no disqualification to drive a motor vehicle. A car
owner who entrusts his car to an established car service and repair shop necessarily entrusts
his car key to the shop owner and employees who are presumed to have the insured's
permission to drive the car for legitimate purposes of checking or road-testing the car. The mere
happenstance that the employees of the shop owner diverts the use of the car to his own illicit
or unauthorized purpose in violation of the trust reposed in the shop by the insured car owner
does not mean that the "authorized driver" clause has been violated such as to bar recovery,
provided that such employee is duly qualified to drive under a valid driver's license.

Where a car is admittedly as in this case unlawfully and wrongfully taken by some
people, be the employees of the car shop or not to whom it had been entrusted, and taken on a
long trip to Montalban without the owner's consent or knowledge, such taking constitutes or
partakes of the nature of theft as defined in Art. 308 of the RPC.

The evidence does not warrant respondent commission's findings that it was a mere "joy
ride". From the very investigator's report cited in its comment, the police found from the waist of
the car driver Benito Mabasa Bartolome who smashed the car and was found dead right after
the incident "one cal. 45 Colt. and one apple type grenade," hardly the materials one would
bring along on a "joy ride". Then, again, it is equally evident that the taking proved to be quite
permanent rather than temporary, for the car was totally smashed in the fatal accident and was
never returned in serviceable and useful condition to petitioner-owner.

CASE NO. 69. AGAPITO GUTIERREZ VS. CAPITAL INSURANCE & SURETY CO., INC.,
G.R. NO. L-26827, JUNE 29, 1984 – CAPULE, STELLA MARIE

FACTS:

Capital Insurance & Surety Co., Inc. insured on December 7, 1961 for one year the
jeepney of Agapito Gutierrez against passenger and third-party liability. The passenger liability
would not exceed P5,000 for any one person.

The policy provides in item 13 that the authorized driver must be the holder of a valid
and subsisting professional driver‘s license. Item 13 is part of the "declarations" which formed
part of the policy and had a promissory nature and effect and constituted "the basis of the
policy".

Page 93 of 99
On May 29, 1962, the insured jeepney figured in an accident at Buendia Avenue, Makati,
Rizal. As a result, a passenger named Agatonico Ballega fell off the vehicle and died. However,
at the time of the accident, Teofilo Ventura, the jeepney driver, did not have a license though he
was duly licensed for the years 1962 and 1963. Instead, he had a carbon copy of a traffic
violation report issued by the policeman on Feb. 22, 1962. The TVR, however, already expired
because it only served as a temporary operator‘s permit for 15 days from receipt.

Gutierrez paid P4,000 to the passenger‘s widow by reason of her husband‘s death. As
Capital Insurance refused to make any reimbursement, he filed on October 14, 1963 in the city
court of Manila an action for specific performance and damages.

The CFI held that Gutierrez‘s Exhibits B and B-1 prove that he paid the widow of
Ballega P4,099.95 and that his driver, Ventura, was an authorized driver because his TVR was
"coextensive with the" two-year term of his confiscated license. It ordered the insurance
company to pay the said amount. The insurance company appealed to this Court.

ISSUE:

Whether an insurance covers a jeepney whose driver's traffic violation report or


temporary operator's permit had already expired.

RULING:

NO. The insurance does not cover a jeepney whose driver‘s traffic violation report or
temporary operator‘s permit had already expired.

In liability insurance, "the parties are bound by the terms of the policy and the right of
insured to recover is governed thereby".

It may be that for purposes of the Motor Vehicle Law the TVR is coterminous with the
confiscated license. That is why the Acting Administrator of the Motor Vehicles Office and the
Manila deputy chief of police ventured the opinion that a TVR does not suspend the erring
driver‘s license, that it serves as a temporary license and that it may be renewed but should in
no case extend beyond the expiration date of the original license.

But the instant case deals with an insurance policy which definitively fixed the meaning
of "authorized driver." That stipulation cannot be disregarded or rendered meaningless. It is
binding on the insured.

It means that to be entitled to recovery the insured should see to it that his driver is
authorized as envisaged in paragraph 13 of the policy which is the law between the parties. The
rights of the parties flow from the insurance contract.

CASE NO. 70. CCC INSURANCE CORPORATION VS. CA AND CARLOS F. ROBES, G.R.
NO. L-25920, JANUARY 30, 1970 – GACAYAN, HAZEL FAYE

FACTS:

On 1 March 1961, Carlos F. Robes took an insurance, with the CCC Insurance Corporation,
on his Dodge Kingsway car against loss or damage through accident for an amount not
exceeding P8,000.00 (Policy No. M1156). On 25 June 1961, and during the effectivity of the
policy, the insured vehicle, while being driven by the owner's driver, became involved in a

Page 94 of 99
vehicular collision along Rizal Avenue Extension, Potrero, Malabon, Rizal. The car was
damaged, and the repair was estimated to cost P5,300.00.

As the insurance company refused either to pay for the repair or to cause the restoration of
the car to its original condition, Robes instituted Civil Case No. Q-6063 in the Court of First
Instance of Rizal for recovery not only of the amount necessary for the repair of the insured car
but also of actual and moral damages, attorneys' fees and costs. Resisting plaintiff's claim, the
insurance company disclaimed liability for payment, alleging that there had been violation of the
insurance contract because the one driving the car at the time of the incident was not an
"authorized driver."

After due hearing, judgment was rendered for the plaintiff, and defendant insurer was
ordered to pay unto the former the cost of repair of the car in the sum of P5,031.28; the sum of
P150.00, for the hauling and impounding of the car at the repair shop; P2,000.00 as actual
damages; and P1,000.00 as attorneys' fees, plus costs.

The insurance company went to the Court of Appeals, raising inter alia the questions of the
qualification of plaintiff's driver to operate the insured vehicle and the correctness of the trial
court's award to plaintiff of the amount of P5,013.28 as cost of repairs, and of actual damages
and attorneys' fees. In its decision of 31 January 1966, the Court of Appeals affirmed the ruling
of the lower court except the award of actual damages in the sum of P2,000.00, which was
eliminated on the ground that it was too speculative.

ISSUE:

1. Whether the plaintiff‘s driver is an authorized driver.

2. Whether or not the damage to the insured car was not covered by the insurance policy
because at the time of the accident it was being driven by one who was not an authorized
driver.

HELD:

A. Yes. The fatal flaw in appellant's argument is that it studiously ignores the provisions of
law existing at the time of the mishap. Under Section 24 of the Revised Motor Vehicles Law, Act
3992 of the Philippine Legislature, as amended by Republic Acts Nos. 587, 1204 and 2863,

An examination or demonstration to show any applicant's ability to operate motor vehicles


may also be required in the discretion of the Chief, Motor Vehicles Office or his deputies.

and reinforcing such discretion, Section 26 of the Act prescribes further:

SEC. 26. Issuance of chauffeur's license; professional badge: If, after examination, or without
the same, the Chief, Motor Vehicles Office or his deputies, believe the applicant to possess the
necessary qualifications and knowledge, they shall issue to such applicant a license to operate
as chauffeur.

It is thus clear that the issuance of a driving license without previous examination does not
necessarily imply that the license issued is invalid. As the law stood in 1961, when the claim

Page 95 of 99
arose, the examinations could be dispensed with in the discretion of the Motor Vehicles Office
official officials. Whether discretion was abused in issuing the license without examination is not
a proper subject of inquiry in these proceedings, though, as a matter of legislative policy, the
discretion should be eliminated. There is no proof that the owner of the automobile knew that
the circumstance surrounding such issuance showed that it was irregular.

The issuance of the license is proof that the Motor Vehicles Office official considered
Reyes, the driver of the insured- appellee, qualified to operate motor vehicles, and the insured
was entitled to rely upon such license. In this connection, it should be observed that the
chauffeur, Reyes, had been driving since 1957, and without mishap, for all the record shows.
Considering that, as pointed out by the Court of Appeals, the weight of authority is in favor of a
liberal interpretation of the insurance policy for the benefit of the party insured, and strictly
against the insurer, We find no reason to diverge from the conclusion reached by the Court of
Appeals that no breach was committed of the above-quoted provision of the policy.

A. The Supreme Court finds no cause sufficient to invalidate the proceedings had in the trial
court. We note that this issue was brought up by the appellant insurance company or the first
time only in its motion for reconsideration filed in the Court of Appeals. It was not raised in the
trial court, where the defect could still be remedied. This circumstance precludes ventilation of
the issue of validity of the hearing at this stage; for, if such irregularity is to vitiate the
proceeding, the question should have been seasonably raised, either before the parties
proceeded with the hearing or before the court handed down its ruling. It is a procedural point
that can be waived by consent of the parties, express or implied.

For the same reason, appellant cannot insist now on the annulment of the proceeding on
the basis of alleged lack of written consent of the parties to the commission, or of an order
appointing the clerk as commissioner, or of notice of the submission of his report to the court.
Furthermore, appellant has presented no proof that the clerk of court committed any mistake or
abuse in the performance of the task entrusted to him, or that the trial court was not able to
properly appreciate the evidence in the case because it was received by another person. If
indeed there were errors at all, they would be non-prejudicial and could not justify the holding of
a new trial, as urged by herein petitioner.

CASE NO. 71. FCP CREDIT CORPORATION, VS.CA, HERMINIO LIM AND EVELYN LIM,
G.R. NO. 96493 (MAY 7, 1992) – PASCUA, MA. BETINA GRACE

Facts:

Private respondents spouses Herminio and Evelyn Lim executed a promissory note in
favor Supercars, Inc. payable in monthly installments and secured by a chattel mortgage over a
brand new car. On the same date, Supercars, Inc., with notice to private respondents spouses,
assigned to petitioner FCP Credit Corporation (FCP for brevity) its rights, title and interest on
said promissory note and chattel mortgage as shown by the Deed of Assignment.

However, said vehicle was carnapped while parked.

Hence, private respondents filed a claim for loss with Perla who denied such claim on
the ground that Lim drove the vehicle with an expired license before it was carnapped.

Private respondents also requested FCP for a suspension of payment on the monthly
amortization agreed upon due to the loss of the vehicle and, since the carnapped vehicle

Page 96 of 99
insured with petitioner Perla, said insurance company should be made to pay the remaining
balance of the promissory note and the chattel mortgage contract.

Issue:

Whether the loss of the collateral exempted the debtor from his admitted obligations
under the promissory note particularly the payment of interest, litigation expenses and attorney's
fees.

Ruling:

No. Private respondents (debtors) are not relieved of their obligation to pay FCP the
installments due on the promissory note on account of the loss of the automobile.

However, this does not mean that private respondents are bound to pay the interest,
litigation expenses and attorney's fees stipulated in the promissory note.

The chattel mortgage earlier instituted by private respondents required private


respondents to insure the automobile and to make the proceeds thereof payable to Supercars,
Inc. The promissory note and chattel mortgage were assigned by Supercars, Inc. to petitioner
FCP, with the knowledge of private respondents. Private respondents, therefore, were able to
secure an insurance policy from petitioner Perla, and the same was made specifically payable
to petitioner FCP.

In view of the foregoing, Perla is ordered to pay FCP the unpaid instalments plus legal
interest.

CASE NO. 72. PARAMOUNT INSURANCE V. REMONDEULAZ, G.R. NO. 173773,


NOVEMBER 28, 2012 – MANGAOANG, NICOLE JOYCE

FACTS:

On May 26, 1994, respondents insured with petitioner their 1994 Toyota Corolla sedan
under a comprehensive motor vehicle insurance policy for one year. The car was then stolen
during the effectivity of the insurance policy and they immediately reported the incident. In their
complaint, they alleged that Ricardo Sales (Sales) took possession of the subject vehicle to add
accessories and improvements thereon, however, Sales failed to return the subject vehicle
within the agreed three-day period.

As a result, respondents notified petitioners to claim for the reimbursement of their lost
vehicle. However, the petitioner refused to pay.

Respondents then filed a complaint for a sum of money against the petitioner before the
RTC of Makati praying for the payment of the insured value of their car plus damages on April
21, 1995.

ISSUE:
Whether petitioner is liable under the insurance policy for the loss of respondents‘
vehicle.

RULING: YES.

Page 97 of 99
Adverse to petitioner‘s claim, respondents‘ policy clearly undertook to indemnify the insured
against loss of or damage to the scheduled vehicle when caused by theft, as embodied in
Section III of the Insurance Policy.

Furthermore, the loss of respondents‘ vehicle falls within the concept of the "theft clause" under
the insurance policy. In Malayan Insurance Co., Inc. v. Court of Appeals, this Court held that the
taking of a vehicle by another person without the permission or authority from the owner thereof
is sufficient to place it within the ambit of the word theft as contemplated in the policy, and is
therefore, compensable.

Sales did not have juridical possession over the vehicle. Hence, it is apparent that the taking of
respondents‘ vehicle by Sales is without any consent or authority from the former.

Records would show that respondents entrusted possession of their vehicle only to the extent
that Sales will introduce repairs and improvements thereon, and not to permanently deprive
them of possession thereof. Since Theft can also be committed through misappropriation, the
fact that Sales failed to return the subject vehicle to respondents constitutes Qualified Theft.
Hence, since respondent‘s car is undeniably covered by a Comprehensive Motor Vehicle
Insurance Policy that allows for recovery in cases of theft, petitioner is liable under the policy for
the loss of respondents‘ vehicle under the "theft clause."

Page 98 of 99

You might also like